Combo with "Evolve: Maternity "

Ace your homework & exams now with Quizwiz!

Which position does the nurse teach the client to avoid when she experiences back pain during labor?

Supine position

An infant born with hydrocephalus is to be discharged after insertion of a ventriculoperitoneal shunt. Which common complication should the nurse instruct the parents to report if it occurs at home?

Fever accompanied by decreased responsiveness

After an unexpected emergency cesarean birth the client tells the nurse, "I failed natural childbirth." Which postpartum phase of adjustment does this statement most closely typify?

Taking-in

A newborn's total body response to noise or movement is often distressing to the parents. What should the nurse explain about this response?

This reflexive response is an expected part of development.

The clinic nurse is providing home care instructions for a client with pelvic inflammatory disease. What resting position should be recommended by the nurse?

Fowler

One minute after birth a nurse assesses a newborn and auscultates a heart rate of 90 beats/min. The newborn has a strong, loud cry; moves all extremities well; and has acrocyanosis but is otherwise pink. What is this neonate's Apgar score?

8

A primigravida at 8 weeks' gestation is visiting the prenatal clinic for the first time. What should an examination reveal at this time?

Goodell's sign

A laboring client has asked the nurse help her to use a nonpharmacological strategy for pain management. Name the sensory simulation strategy.

Selecting a focal point and beginning breathing techniques

The most appropriate method for a nurse to evaluate the effects of the maternal blood glucose level in the infant of a diabetic mother (IDM) is by performing a heel stick blood test on the newborn. What does this test determine?

Serum glucose level

A nurse is obtaining a health history from a client with newly diagnosed cervical cancer. What aspect of the client's life is most important for the nurse to explore at this time?

Support system

A client at 28 weeks' gestation with previously diagnosed mitral valve stenosis is being evaluated in the clinic. Which sign or symptom indicates that the client is experiencing cardiac difficulties?

Syncope on exertion

The nurse is caring for a couple after the birth of their first child. What should the nurse tell the family to do when their infant is exhibiting the behavior demonstrated in the picture?

"This is the time when the baby is likely to be most responsive to you."

When a client who had a mastectomy sees her incision for the first time, she exclaims, "I look horrible! Will it ever look better?" What is the nurse's best response?

"You seem shocked by the way you look now."

A nurse is providing dietary counseling to a client at 14 weeks' gestation. The client is a recent immigrant from Asia, and the nurse explores the foods that the client usually eats. Which foods should the nurse counsel the client to avoid during pregnancy? (Select all that apply.)

4 Raw shellfish 5 Herbal supplements 6 Soft-scrambled eggs

A nurse is teaching a woman how to perform breast self-examination. Which statement indicates that the client needs further education?

"My breasts are so tender right before my period that I hate doing it."

What instruction should a nurse include when teaching about the correct use of a female condom?

"Remove the condom before standing up."

At 9 pm visiting hours are officially over, but the sister of a newly admitted postpartum client remains at the bedside. What is the most appropriate nursing intervention?

Encouraging the sister to participate in care as much as the client wishes

A 47-year-old client comes to the clinic for a Papanicolaou (Pap) smear. She tells the nurse that she has been experiencing hot flashes and that her periods have been occurring at longer, less regular intervals, with a scanty flow. What does the nurse conclude is the most likely cause of these changes?

Expected menopausal changes

What should the nurse's initial discussion include to best help new parents understand tA nurse is caring for a newborn with a myelomeningocele. What should immediate nursing care for this infant include? he unique characteristics of a newborn?

Expected movements and behaviors

A primigravida is admitted with a ruptured fallopian tube resulting from a tubal pregnancy and surgery is performed to remove the fallopian tube. What should postoperative nursing care include?

Explaining that the client may still be capable of becoming pregnant

A primigravida, unsure of the date of her last menstrual period, is told by the nurse that she appears to be at 22 weeks' gestation. What data support this conclusion?

Fundus at the umbilicus

A primigravida client gave birth in a vaginal delivery 24 hours ago. Which findings would be considered normal?

Fundus firm at the umbilicus; moderate lochia rubra; voiding quantity sufficient; colostrum present

A pregnant woman at 34 weeks' gestation is being seen at the clinic. The client's blood pressure is 166/100 mm Hg and her urine is +3 for protein. She states that she has a severe headache and occasional blurred vision. Her baseline blood pressure was 100/62 mm Hg. What is the priority nursing action?

Arranging transportation to the hospital

A client at 9 weeks' gestation asks the nurse in the prenatal clinic whether she may have chorionic villi sampling (CVS) performed during this visit. What should the nurse keep in mind as the optimal time for CVS while formulating a response?

At 10 weeks but no later than 12 weeks

A client has a total hysterectomy with bilateral salpingo-oophorectomy for cancer of the ovary. In addition to encouraging ambulation, what exercise should the nurse instruct the client to perform to help prevent postoperative deep vein thrombosis (DVT)?

B

A woman in labor with her third child is dilated 7 cm, and the fetal head is at station +1. The client's membranes rupture. What should the nurse do first?

Check the fetal heart rate while observing the color of the amniotic fluid

A client has chosen not to have her son circumcised. What instruction should be included in discharge teaching for the care of an uncircumcised neonate?

Clean the penis with warm water at each diaper change.

A neonate is tested for phenylketonuria (PKU) after formula feedings are initiated. The nurse explains to the parents that this is done to prevent:

Cognitive Impairment

What recommendation should a nurse give to a client with fluid retention during pregnancy?

Elevate the lower extremities.

After treatment for a bladder Infection; a client asks whether there is anything she can do to prevent cystitis in the future. What is the best response by the nurse?

Increase daily fluid consumption.

Where is the best area for the nurse to determine adequate tissue oxygenation in a neonate born of black parents?

Mucous membranes of the mouth

A client gives birth to a baby weighing 7 lb 2 oz and decides to breastfeed. The nurse is instructing the client regarding breastfeeding. What should the nurse tell the client to expect?

Lochial flow will be increased.

A client who has had a cesarean birth seems upset. She has been having difficulty breastfeeding for 2 days and now asks the nurse to bring her a bottle of formula. What is the nurse's initial action?

Observing the client's breastfeeding technique -The nurse should assess the client to determine why she is having difficulty with breastfeeding. She may be uncomfortable or in need of assistance with her breastfeeding technique. Immediately providing the formula without assessing the situation does not meet the client's needs at this time.

At a client's first visit to the prenatal clinic, the nurse asks the client when she had her last menstrual period so the estimated date of birth (EDB) can be determined. The client responds, "January 21." According to Nägele's rule, what is the month and day of the client's EDB?

October 28

What should the nurse evaluate before continuing the administration of IV magnesium sulfate therapy to a client with preeclampsia?

Patellar reflexes and urinary output -Adequate urinary output, an indicator of effective renal function, is necessary to prevent toxicity because magnesium sulfate is excreted by the kidneys

A newborn male is admitted to the nursery. He weighs 10 lb 2 oz, which is 2 lb more than the birthweight of any of his siblings. What should the nurse do in relation to the baby's weight?

Perform serial glucose readings

A left modified radical mastectomy is performed on a client with breast cancer. What is the most important measure to be included in the care plan for the first postoperative day?

Placing the client in the semi-Fowler position with the left arm and hand elevated

A nurse plans to weigh a newborn. What is the most appropriate way to obtain the newborn's weight?

Placing the naked infant on the scale

A 7-lb newborn is admitted to the nursery with a prescription for intramuscular phytonadione (vitamin K, Aquamephyton) 1 mg. The nurse explains to the parents that this vitamin is administered to:

Promote clotting of the blood

What nursing action is the priority for a client in the second stage of labor?

Promote effective pushing by the client

When teaching a client about using a diaphragm as a form of contraception, the nurse should tell her that the diaphragm:

Should remain in place for at least 6 hours after intercourse

A client with mild preeclampsia is being treated on an outpatient basis. Three days of bedrest is prescribed. What position should the nurse encourage the client to maintain while in bed?

Side-lying -The side-lying position improves venous return to the heart and increases stroke volume and cardiac output.

Which finding indicates the development of a complication resulting from the presence of bilateral cephalhematomas?

Skin color

A client had a cesarean birth 3 days ago. Where should the nurse, while palpating her fundus, expect to locate the fundus if each line represents 1 cm?

c

A nurse is providing nutritional counseling to a low-income pregnant client who has iron-deficiency anemia. What food should the nurse encourage the client to include in her diet each day to best address this problem?

½ cup of red kidney beans

Assign an Apgar score to this infant: heart rate 110, crying vigorously, moves all extremities, cries when suctioned, blue extremities with pink body. Record your answer using a whole number. ______

9

A client at 32 weeks' gestation is admitted to the prenatal unit in preterm labor. An infusion of magnesium sulfate is started. What physiological response indicates to the nurse that the magnesium sulfate is having a therapeutic effect?

A decrease in frequency and duration of contractions -The purpose of administering magnesium sulfate is to stop preterm labor. It is a tocolytic agent that relaxes uterine smooth muscle. Labor is progressing if dilation of the cervix continues

Which parts of a newborn's body are usually affected by the rash erythema toxicum neonatorum? Select all that apply.

A face D trunk E buttocks

A nurse on the postpartum unit is assessing several clients. Which clinical finding requires immediate investigation?

A slow trickle of blood from the vagina

A client is scheduled for a vacuum aspiration abortion to terminate an unwanted pregnancy. What information should the nurse's teaching plan include?

A temperature of 100.4° F (38° C) or higher should be reported immediately.

Which client is at risk for a postpartum infection?

A woman who required catheterization after voiding less than 75 mL

A nurse is giving discharge instructions to a client who has undergone anterior colporrhaphy. The nurse knows the teaching has been understood if the client says:

" For discomfort I can try using warm compresses on my abdomen."

A client at 31 weeks' gestation is admitted in preterm labor. She asks the nurse whether there is any medication that can stop the contractions. What is the nurse's response?

"A beta-adrenergic."

A mother asks the neonatal nurse why her infant must be monitored for hypoglycemia when her type 1 diabetes was in excellent control during her pregnancy. How should the nurse respond?

"Babies of mothers with diabetes have a higher-than-average insulin level because of the excess glucose received from the mothers during pregnancy, so the glucose level may drop."

A client has a child with Tay-Sachs disease and wants to become pregnant again. She tells the nurse, "I'm worried it will happen again." How should the nurse respond?

"Have you considered the option of genetic counseling?" -Asking the client whether she has considered genetic counseling ensures that the client is informed of the need for genetic counseling and gives her an option for decision-making.

What statement helps the nurse determine that a woman with genital herpes (HSV-2) understands her self-care in regard to this infection?

"I must be careful when I have sex because herpes is a lifelong problem." -Genital herpes (HSV-2) is characterized by remissions and exacerbations; it cannot be cured. Most pregnant women with HSV-2 have cesarean deliveries to prevent their newborns from contracting the disease while passing through the vagina. Clients should abstain from sex until 10 days after the lesions heal. Herpes can be controlled, not cured.

After the birth of her baby a client tells the nurse, "I'm so cold, and I can't stop shaking." How should the nurse respond?

"I'll get you some warm blankets to help make the chill go away."

A pregnant client uses a computer almost continuously during her working hours. This has implications for her plan of care during pregnancy. What should the nurse recommend?

"Try to walk around every few hours during the workday."

A client whose weight was average for her height before becoming pregnant is concerned because she has gained 15 lb (6.9 kg) after only 23 weeks of pregnancy. What is the nurse's most appropriate response?

"Your weight is expected for someone at 23 weeks' gestation. Continue the pregnancy diet."

Vitamin K 0.5 mg is prescribed for a newborn. The vial on hand is labeled "1 mL = 2 mg." How many milliliters should the nurse administer? Include a leading zero if applicable. Record your answer using two decimal places. _____ mL

0.25

A nurse is caring for a new mother who has a chlamydial infection. Which complications are associated with chlamydial infections in neonates? (Select all that apply.)

1 Pneumonia 2 Preterm birth 4 Conjunctivitis

The nurse discusses the recommended weight gain during pregnancy with a newly pregnant client who is 5 feet 3 inches tall and weighs 125 lb. The nurse explains that with the recommended weight gain, at term the client should weigh about:

1. 150 lb -A weight of 150 lb would put the client within the recommended weight gain of at least 25 lb for a woman who was of average weight for her height before pregnancy. A weight of 140 lb is less than the recommended weight gain for a woman of average weight for height before pregnancy, as are 135 lb and 130 lb.

A nurse caring for a pregnant client and her partner suspects domestic violence. Which observations support this suspicion? (Select all that apply.)

1. The woman has injuries to the breasts and abdomen. 3. The partner answers questions that are asked of the woman. 4. The woman has visited the clinic several times in the last month.

A negative home pregnancy test may result if the woman performs the test:

10 days after intercourse took place

A client who menstruates regularly every 30 days asks a nurse on what day she is most likely to ovulate. Because the client's last menses started on January 1, the nurse should tell her that ovulation should occur on which day in January?

16

A woman at 36 weeks' gestation is admitted to the hospital to receive a tocolytic medication in an attempt to stop labor. In addition, betamethasone (Celestone) is prescribed to enhance fetal lung maturity. The prescription reads: "Administer betamethasone 12 mg IM daily for 3 days." The betamethasone comes in a vial labeled "6 mg/mL." How many milliliters should the nurse administer each day? Record your answer using a whole number. ____ mL

2

A client attending a prenatal class about nutrition tells the nurse that she is a strict vegetarian (vegan). What should the nurse encourage the client to eat that includes all the essential amino acids?

2. Whole-grain cereals and nuts

What type of respirations does the nurse expect to identify in a healthy newborn?

Abdominal and irregular

A nurse is checking the external fetal monitor of a client in active labor. Which fetal heart pattern indicates cord compression?

Abrupt decreases in fetal heart rate that are unrelated to the contractions

A client is found to have gestational hypertension in the 22nd week of gestation. What is a major complication of hypertensive disease associated with pregnancy that the nurse should anticipate?

Abruptio placentae

A 16-year-old girl who has become sexually active asks the nurse, "What's the most effective way to prevent a pregnancy?" Which method of preventing pregnancy should the nurse tell her is most effective?

Abstaining from sexual intercourse

A nurse is performing the Ortolani test on a newborn. Which finding indicates a positive result?

An audible click on abduction

On a return visit to the fertility clinic a couple requests fertility drugs because, despite having a 28-day menstrual cycle and temperature readings that demonstrate an ovulatory pattern, the woman has been unable to conceive. What should the nurse explain to the couple?

An examination of semen will be needed.

A female client came to the clinic with suspected primary syphilis. What sign of primary syphilis does the nurse expect the client to exhibit?

An indurated painless nodule on the vulva that is draining

A newborn has a diagnosis of Erb palsy (Erb-Duchenne paralysis). What does a nurse identify as the cause of this complication?

An injury to the brachial plexus during birth

What should a nurse teach a non-nursing mother to help relieve the discomfort of engorgement?

Apply cold packs to the breasts frequently.

A newborn male is being discharged 4 hours after having had a circumcision. What should the nurse instruct the mother to do?

Apply the diaper loosely for several days

What is the best nursing intervention to minimize perineal edema after an episiotomy?

Applying ice packs

A client comes to the clinic for a 6-week postpartum checkup. She confides that she is experiencing exhaustion that is not relieved by sleep and feelings of failure as a mother because the infant "cries all of the time." When asked whether she has a support system, she replies that she lives alone. Which response would provide the most accurate information?

Asking the client questions, using a postpartum depression scale

How does a nurse identify possible developmental dysplasia of the hip (DDH) during a newborn assessment?

Asymmetry of the gluteal folds

A client seeking family planning information asks the nurse during which phase of the menstrual cycle an intrauterine device (IUD) should be inserted. Before responding the nurse recalls that the insertion usually is done:

Between the first and fourth days of the cycle

A client whose membranes have ruptured is admitted to the birthing unit. Her cervix is dilated 3 cm and 50% effaced. The amniotic fluid is clear and the fetal heart rate is stable. What does the nurse anticipate?

Birth of the fetus within a day

What does a nurse expect to find when checking the vital signs of a client in the early postpartum period?

Bradycardia with no change in respirations

A nurse teaches a woman who is planning to breastfeed how to relieve breast engorgement. The nurse determines that further teaching is necessary when the woman states that she will:

Breastfeed the infant less frequently

A nurse is assessing a newborn. Which sign should the nurse report?

Breathing pattern with recurrent sternal retractions

A nurse is caring for a client with preeclampsia who is receiving intravenous magnesium sulfate therapy. What antidote should the nurse have readily available?

Calcium gluconate

A client in labor is admitted to the birthing unit. Assessment reveals that the fetus is in a footling breech presentation. What should the nurse consider about breech presentations when caring for this client?

Cesarean birth probably will be necessary

A practitioner orders doxycycline (Vibramycin) for a sexually active woman with a history of a mucopurulent discharge and bleeding associated with cervical dysplasia, dysuria, and dyspareunia. With which sexually transmitted infection are these clinical findings and medication therapy commonly associated?

Chlamydial infection -The signs and symptoms listed and the treatment ordered (doxycycline or azithromycin) indicate that the client has a chlamydial infection. Painful blisters on the genitalia, fever, malaise, dysuria, and dyspareunia are signs of herpes simplex virus 2 infection. Chancre formation is a sign of primary syphilis; a symmetrical rash accompanied by malaise, fever, anorexia, and headache is indicative of secondary syphilis. Dysuria, heavy greenish-yellow purulent discharge, and swollen Bartholin glands are signs of gonorrhea.

A nurse in the postpartum unit must complete several interventions before a client's discharge from the hospital. The nurse plans to delegate some of the tasks to the nursing assistant. Which activity must be performed by the nurse?

Comparing the identification bands of mother and infant

A nurse is teaching a pregnant client with sickle cell anemia about the importance of taking supplemental folic acid. Folic acid is important for this client because it:

Compensates for a rapid turnover of red blood cells

A client at 11 weeks' gestation reports having to urinate more often. The nurse explains that urinary frequency often occurs because bladder capacity during pregnancy is diminished by:

Compression by the enlarging uterus

A pregnant client tells the nurse that her husband is a chain smoker. What information should the nurse's teaching include?

Continued exposure to secondhand smoke is related to fetal growth restriction.

Three weeks after a client gives birth, a deep vein thrombophlebitis develops in her left leg and she is admitted to the hospital for bedrest and anticoagulant therapy. Which anticoagulant does the nurse expect to administer?

Continuous infusion of heparin

A nurse is assessing a postpartum client for signs of an impending hemorrhage resulting from laceration of the cervix. Besides monitoring the client for a firm uterus, what other assessment is important?

Continuous trickling of blood

A 1-day-old newborn has just expelled a thick, greenish-black stool. The nurse determines that this is the first stool. What should the nurse do next?

Document the stool in the infant's record

A client at 16 weeks' gestation arrives at the prenatal clinic for a routine visit. During the examination the nurse notes bruises on the client's face and abdomen. There are no bruises on her legs and arms. Further assessment is required to confirm:

Domestic abuse

A client at 36 hours' postpartum is being treated with subcutaneous enoxaparin (Lovenox) for deep vein thrombosis of the left calf. Which client adaptation is of most concern to the nurse who is monitoring the client?

Dyspnea

A client in her 37th week of gestation calls a nurse in the prenatal clinic and reports, "My ankles are swollen." What should the nurse recommend?

Elevating her legs more frequently during the day

Hydramnios is diagnosed in a primigravida at 35 weeks' gestation. What condition of the newborn is associated with hydramnios?

Esophageal atresia

The nurse is caring for a pregnant client who is undergoing an ultrasound examination during the first trimester. The nurse explains that an ultrasound during the first trimester is used to:

Estimate fetal age

A client with a benign ovarian tumor undergoes laparoscopic surgery. What should the nurse include in the postoperative teaching?

Expect shoulder pain for 12 to 24 hours.

A client at 10 weeks' gestation tells the nurse in the maternity clinic that she is worried because she is voiding frequently. How should the nurse respond?

Explaining why this is expected in early pregnancy

A client is admitted in active labor to the birthing center. She is 100% effaced, dilated 3 cm, and at +1 station. What stage of labor does the nurse identify?

First

What does the nurse conclude is related directly to an infant's survival in the neonatal period?

Gestational age and birth weight

A nurse is caring for the newborn of a drug-addicted mother with suspected cytomegalovirus disease. What does the nurse suspect was the cause of the disease?

Having sex with many partners

Because of the high discomfort level during the transition phase of labor, nursing care should be directed toward:

Helping the client maintain control

A nurse is planning for the discharge of a crack-addicted 17-year-old mother and her newborn. What is the most appropriate referral to meet the mother's and infant's needs?

Home health nurse

A nurse identifies a right cephalhematoma on an otherwise healthy 1-day-old newborn. What should the nurse teach the parents at the time of discharge?

How to monitor their child for signs of jaundice

During a routine prenatal visit, a client tells a nurse that she gets leg cramps. What condition does the nurse suspect, and what suggestion is made to correct the problem?

Hypocalcemia; increase her intake of milk

A small-for-gestational-age (SGA) newborn who has just been admitted to the nursery has a high-pitched cry, appears jittery, and exhibits irregular respirations. What complication does the nurse suspect?

Hypoglycemia

A neonate has phenylketonuria (PKU). What information should the nurse include in a discussion with the parents when explaining what caused their infant's problem?

Inborn error of metabolism

A client who wishes to postpone having children until she and her husband are financially sound tells the nurse she has been taking oral contraceptive pills for several years. What finding indicates a potential risk in regard to continued use of birth control pills?

Increased blood pressure

A nurse in the prenatal clinic reviews second-trimester physiological changes in the hematological system before explaining them to a client. What change should the nurse identify?

Increased blood volume

A client arrives at the fertility clinic for a diagnostic workup and is told by the nurse to prepare for a Papanicolaou (Pap) test. The client states, "I do not want this test. I want to speak to the person in charge." How should the nurse respond to this statement?

Inform the health care provider of the client's request

A woman in the family planning clinic has decided to use the diaphragm for contraception. What should the nurse teach her about using a diaphragm?

Insert the diaphragm before intercourse and leave it in at least 6 hours after intercourse to kill all the sperm.

A thin older adult client is found to have osteoporosis. What should the nurse include in the discharge plan for this client?

Instructions relative to diet and exercise

The nurse is counseling a pregnant client with type 1 diabetes about medication changes as pregnancy progresses. Which medication will be needed in increased dosages during the second half of her pregnancy?

Insulin

A nurse has learned that infants born to very young mothers are at risk for neglect or abuse primarily because an adolescent characteristically:

Is involved in seeking her own identity -Adolescent parents are still involved in the developmental stage of resolving their own self-identity; they have not sequentially matured to intimacy and generativity, making nurturing of another difficult.

A nurse is writing a teaching plan about osteoporosis. How should the nurse explain what osteoporosis is?

It involves a decrease in bone substance.

When a mother sees her newborn son assume a fencing position as she turns his head, she becomes concerned. What should the nurse explain about this reflex?

It is expected in the healthy newborn.

A client has a modified radical mastectomy because of a malignant tumor of the breast. What does the nurse plan to teach the client during the early postoperative period?

Keep the arm in an elevated position.

A parent of a preterm infant in the neonatal intensive care unit, asks a nurse why the baby is in a bed with a radiant warmer. The nurse explains that preterm infants are at increased risk for hypothermia because they:

Lack the subcutaneous fat that usually provides insulation

What should the nurse discuss with new parents to help them prepare for infant care?

Learning specific behaviors involving states of wakefulness to promote positive interactions

A newborn has small, whitish, pinpoint spots over the nose that are caused by retained sebaceous secretions. When documenting this observation, a nurse identifies them as:

Milia -Milia are common, are not indicative of illness, and eventually disappear. Lanugo is fine, downy hair.

What is the priority nursing care after an amniocentesis?

Monitoring for signs of uterine contractions -It is possible that stimulation of the uterus resulting from the amniocentesis will cause uterine contractions. Perineal care is not necessary because an amniocentesis is not done by way of the vagina.

At 37 weeks' gestation a client's membranes spontaneously rupture but she does not have contractions. What action is most important in the nursing plan of care for this client?

Monitoring for the presence of fever

A 42-year-old client undergoes amniocentesis during the 16th week of gestation because of concern about Down syndrome. What additional information about the fetus will examination of the amniotic fluid reveal at this time?

Neural tube defect

Which sign indicates to the nurse that a neonate is preterm?

Numerous superficial veins

During the postpartum period a nurse determines that a client's rubella titer is negative. What action should the nurse plan to take?

Obtaining a prescription for immunization at discharge

In specific situations gloves are used to handle newborns whether or not they are HIV positive. When is it unnecessary for the nurse to wear gloves while caring for a newborn?

Offering a feeding

A female client with Hodgkin's disease is to start chemotherapy. She and her husband have been trying to have a child and are quite concerned when they learn that sterility may result. On what information should the nurse base the reply?

Ova can be harvested and frozen for future use.

A 37-year-old client with endometriosis visits the women's health clinic because she has dysmenorrhea and dyspareunia. What is a description of dysmenorrhea?

Pain with menses

A histogram (hysterosalpingography [HSG]) is performed to determine whether a client has a tubal obstruction. The nurse explains to the client that infertility caused by a defect in the tube is most often related to:

Past infection

A nurse is teaching a client to care for her episiotomy after discharge. What priority instruction should the nurse include?

Perform perineal care after toileting until healing occurs.

A client in labor is admitted with a suspected breech presentation. For what occurrence should the nurse be prepared?

Prolapsed cord

Sitz baths are ordered for a client with an episiotomy during the postpartum period. A nurse encourages her to take the sitz baths because they aid the healing process by:

Promoting vasodilation

Which action involving client needs may a nurse delegate to a nursing assistant?

Providing ice chips to a primigravida in early labor per order

A pregnant woman tells a nurse, "I think I can feel the baby move now. It feels like butterflies in my stomach. My friend calls it feeling life." What term should the nurse include when discussing fetal movement with the woman?

Quickening

An 18-year-old primigravida at 36 weeks' gestation is admitted with a diagnosis of mild preeclampsia. What is the nurse's most important goal for this client?

Reducing her blood pressure

When a preterm newborn requires oxygen, the nurse in the neonatal intensive care unit monitors and adjusts the oxygen concentration. What complication do these adjustments attempt to prevent?

Retinopathy of prematurity

A nurse is planning to administer Rhogam (Rh immune globulin). Which situation requires the administration of this medication?

Rh-negative woman who has had an amniocentesis

A pregnant client who has a history of cardiac disease asks how she can relieve her occasional heartburn. The nurse should instruct the client to avoid antacids containing:

Sodium

A nurse assesses a new mother who is breastfeeding. The client asks how to care for her nipples. What should the nurse recommend?

Spreading breast milk on the nipples after the feeding and allowing them to air dry

A health care provider tells a mother that her newborn has multiple visible birth defects. The mother seems composed and asks to see her baby. What nursing action will be most helpful in easing the mother's stress when she sees her child for the first time?

Staying with her after bringing the infant to help her verbalize her feelings.

A client who is visiting the prenatal clinic for the first time has a serology test for toxoplasmosis. What information about the client's activities in the history indicates to the nurse that there is a need for this test?

The client takes care of a cat.

A 36-year-old primigravida, accompanied by her husband, is admitted to the birthing unit at 39 weeks' gestation. External fetal monitoring is instituted. What should the nurse consider when a fetus is being monitored?

The machinery may be frightening to a laboring couple.

A client who has had a postpartum hemorrhage is to receive 1 unit of packed red blood cells (RBCs). The nurse manager observes a staff nurse administering the packed RBCs without wearing gloves. What does the nurse manager conclude?

The nurse should have worn gloves for self-protection.

A client has a blood pressure of 90/50 mm Hg during her first visit to the prenatal clinic. On a subsequent visit, at 34 weeks' gestation, her blood pressure is 120/76 mm Hg. The nurse concludes that could have occurred because of:

The possible development of preeclampsia

During a newborn assessment the nurse counts the infant's cord vessels. What does the nurse expect to observe in a healthy newborn?

Three vessels: one vein and two arteries

Immediately after the third stage of labor a nurse administers the prescribed oxytocin (Pitocin) infusion. Why is this medication administered?

To help the uterus contract

A client is receiving an epidural anesthetic during labor. For which side effect should the nurse monitor the client?

Urine retention -Anesthesia blocks the sensory pathways; therefore the mother does not sense bladder distention and may be unable to void.

What should the nurse do when an apnea monitor sounds an alarm 10 seconds after cessation of respirations?

Use tactile stimuli on the chest or extremities

A young client tells the nurse that her mother complains about having dysmenorrhea and asks the nurse what this means. How should the nurse describe dysmenorrhea?

Uterine pain during the menstrual period

A nurse instructs a client who is taking oral contraceptives to increase her intake of dietary supplements. Which supplement should be increased?

Vitamin C -Oral contraceptives can affect the metabolism of certain vitamins, particularly vitamin C, and supplementation may be required.

A thin 24-year-old woman who runs 10 miles each week asks the nurse for advice about preventing osteoporosis. Which vitamin and other dietary supplement should the nurse recommend?

Vitamin D and calcium citrate

A nurse is teaching a breastfeeding mother about cleansing her nipples. What technique should the nurse emphasize?

Wash the breasts and nipples with water when bathing.

A nurse is teaching a class about childbearing and contraceptive options. The nurse explains that fertilization of the ovum by the sperm occurs at a specific time. When does it occur?

When one sperm penetrates the wall of the ovum

What client behavior indicates to the nurse that a woman needs further teaching about breastfeeding her newborn?

When she leans forward to place her breast in the infant's mouth

A nurse elicits the Babinski reflex on a newborn. The nurse concludes that this finding indicates:

Immaturity of the central nervous system (CNS)

A nurse from the pediatric clinic who is strongly opposed to any chemical or mechanical method of birth control is asked to work in the family planning clinic. What is the most professional response that this nurse could give to the requesting supervisor?

"I would prefer another assignment that is not contrary to my beliefs."

The mother of a newborn son tells the nurse that she is concerned about circumcision because of the pain involved. What is the nurse's best response?

"The health care provider will tell you how your baby's pain will be controlled."

During a routine visit to the prenatal clinic a client listens to the fetal heartbeat for the first time. The client, commenting on how rapid it is, appears frightened and asks whether this is normal. The nurse should explain:

"The heart rate is usually rapid, and this one is in the expected range."

A postpartum client who was receiving an intravenous infusion of oxytocin (Pitocin) to stimulate labor asks the nurse why it is not being discontinued now that the baby is born. The nurse responds:

"The oxytocin causes contraction of the uterine musculature." -Oxytocin (Pitocin) intensifies contractions of the uterus and promotes return of the uterus to its prepregnant state.

When encouraging a client to cough and deep breathe after a bilateral mastectomy, the client says, "Leave me alone! Don't you know I'm in pain?" What is the nurse's most therapeutic response?

"What's your pain level right now?"

A client who is scheduled to have a hysterectomy starts to sob and says, "I told my husband that after this operation, I'll be only half a woman. He told me not to worry, but I know that he was just putting up a front." How should the nurse respond?

"You think this operation will affect how your husband feels about you as his wife."

A nurse is caring for a pregnant client with thrombophlebitis. Which anticoagulant medication may be prescribed? (Select all that apply.)

1 Heparin (Hep-Lock) 4 Enoxaparin (Lovenox)

A woman visits the prenatal clinic because an over-the-counter pregnancy test has rendered a positive result. After the initial examination verifies the pregnancy, the nurse explains some of the metabolic changes that occur during the first trimester of pregnancy. (Select all that apply.)

1 Sleep needs increase. 2 Fluid retention increases. 4 Calcium requirements increase. -Estrogen increases the secretion of corticosteroids, which decrease the basal metabolic rate, resulting in fatigue. Sodium is retained, and fluid retention increases to meet total needs. During the first trimester approximately 1.2 g of calcium is needed each day; this need continues throughout pregnancy as the fetal skeleton is being formed. Body temperature increases because of the increased metabolism related to the growth of the fetus. Carbohydrate needs increase because the secretion of insulin by the pancreas is increased; however, insulin is destroyed rapidly by the placenta. The stress of pregnancy may precipitate gestational diabetes.

Five minutes after being born, a newborn is pale; has irregular, slow respirations; has a heart rate of 120 beats/min; displays minimal flexion of the extremities; and has minimal reflex responses. What is this newborn's Apgar score?

5 -According to the Apgar scoring system, the newborn receives 2 points for heart rate, 0 for color, 1 for respiratory effort, 1 for muscle tone, and 1 for reflex irritability. An Apgar score of 3 is low. Scores of 5 and 6 are higher, but the newborn may still require stimulation and oxygen. Topics

A client who has type O Rh-positive blood gives birth. The neonate has type B Rh-negative blood. Eleven hours after birth, the infant's skin appears yellow. What is the most likely cause?

ABO incompatibility

A client in labor at 39 weeks' gestation is told by the health care provider that she will need a cesarean birth. The nurse reviews the client's prenatal history. What preexisting condition is the most likely reason for the cesarean birth?

Active genital herpes

What should be included in the plan of care for a client with class I cardiac disease during the last weeks of pregnancy?

Advising the client to limit stress, promoting rest after meals, and educating the client about the analgesia and anesthesia used during labor

The nurse is caring for a first-time mother at her first prenatal visit. The client confides, "I'm not sure about all this." Which research-based knowledge guides a nurse regarding the emotional factors of pregnancy?

Ambivalence and anxiety about mothering are common.

The fetus of a woman in labor is at +1 station. At what place in the pelvic area does the nurse conclude that the presenting part is located?

Below the ischial spines

After performing Leopold maneuvers on a laboring client, a nurse determines that the fetus is in the right occiput posterior (ROP) position. Where should the Doppler ultrasound transducer be placed to best auscultate fetal heart tones?

Below the umbilicus on the right side

A nurse is caring for a postpartum client who had abruptio placentae. Which finding indicates that disseminated intravascular coagulation (DIC) is occurring?

Bleeding at the venipuncture site

Which information is most important for a large-for-gestational-age (LGA) infant of a diabetic mother (IDM)?

Blood glucose level less than 40 mg/dL

During labor a client begins to experience dizziness and tingling of her hands. What should the nurse instruct the client to do?

Breathe into her cupped hands -Dizziness and tingling of the hands are signs of respiratory alkalosis , probably the result of hyperventilating; breathing into cupped hands promotes the rebreathing of carbon dioxide.

The parents of a newborn are told that their neonate may have Down syndrome and that additional diagnostic studies will be done to confirm this diagnosis. What procedure does the nurse expect to be performed?

Buccal smear

An infant has surgery for repair of a myelomeningocele. For which early sign of impending hydrocephalus should the nurse monitor the infant?

Bulging fontanels -After closure, spinal fluid may accumulate and reach the brain, increasing intracranial pressure (ICP) and causing the fontanels to bulge.

A nurse who is monitoring the blood glucose level of the term infant of a diabetic mother (IDM) identifies a blood glucose level of 48 mg/dL. What should the nurse do?

Continue to monitor the blood glucose level per policy. -A reading of 48 mg/dL is within the expected blood glucose range for a neonate (40-60 mg/dL) and requires no measures other than continued monitoring for the next 24 hours. Heel sticks are adequate for monitoring the blood glucose level of a neonate

The partner of a woman experiencing back pain in labor asks what he can do to help. The nurse demonstrates how to apply counterpressure to his partner's back. Where on the image should counterpressure be applied?

D Counterpressure on the lower sacrum relieves discomfort by lifting the fetal head off the spinal nerves when the fetus is in a posterior position.

A pregnant woman who was admitted to the high-risk maternity unit for severe hyperemesis gravidarum is receiving total parenteral nutrition (TPN). Intralipids are not being administered. For what potential complication should the nurse monitor the client?

Dehydration

The nurse explains to a woman in her 24th week of pregnancy that absorption of medications taken orally during pregnancy may be altered as a result of:

Delayed gastrointestinal emptying

A nurse is counseling a client with type 1 diabetes who has requested contraceptive information. On which method of contraception should the nurse place the most emphasis?

Diaphragm

Why is it important for a nurse in the prenatal clinic to provide nutritional counseling to all newly pregnant women?

Different cultural groups favor different essential nutrients.

What is a nurse's most important concern when caring for a client with a ruptured tubal pregnancy?

Diminished cardiac output

A couple in their late 30s who wish to have a child are referred for genetic counseling. They tell the nurse that they have a family history of an inheritable problem but have reservations about genetic counseling because they believe that genetic clinics favor abortion when the studies reveal a defective fetus. How should the nurse respond regarding genetic counseling?

Families are helped to understand the diagnosis, the probable cause of the disorder, and how the condition can be managed.

A health care provider prescribes carboprost (Hemabate) to be administered to a postpartum client with intractable vaginal bleeding. What client factor should alert the nurse to question the prescription?

History of asthma

Which adaptation does the nurse suspect is the result of early decompensation in a pregnant woman with cardiac problems?

Increasing fatigue -Increasing fatigue is one of the early signs of decompensating resulting from an increased cardiac workload. Hemoptysis is a later sign of cardiac decompensation that is associated with pulmonary edema. Tachycardia and generalized edema are later signs of cardiac decompensating and may be accompanied by other signs of heart failure.

A hysterectomy is scheduled for a client with endometrial cancer. Before the surgery, what should the nurse prepare the client to expect?

Indwelling urinary catheter

After a deep vein thrombosis developed in a postpartum client, an IV infusion of heparin therapy was instituted 2 days ago. The client's activated partial thromboplastin time (aPTT) is now 98 seconds. What should the nurse do?

Interrupt the infusion and notify the practitioner of the aPTT result -The heparin should be withheld, because 98 seconds is almost three times the normal time it takes a fibrin clot to form (25 to 36 seconds) and prolonged bleeding may result; the therapeutic range for heparin is one-and-a-half to two times the normal range. The primary health care provider should be notified

What is a common problem that affects the client in labor when an external fetal monitor has been applied to her abdomen?

Intrusion on movement

A woman is admitted for a hysterectomy and bilateral salpingo-oophorectomy. The nurse reviews the client's gynecological history. What condition does the client have that causes the nurse to anticipate an abdominal, rather than a vaginal, hysterectomy?

Large uterine fibroids -Attempting to remove a uterus with large uterine fibroids vaginally can cause trauma, resulting in hemorrhage. Vaginal hysterectomy is indicated for prolapsed uterus because the uterus is usually collapsed into the vagina. A hysterectomy is not the treatment of choice for mild cervical dysplasia; when a hysterectomy is necessary, the vaginal route is preferred. Urinary incontinence with coughing may be related to stress incontinence, which does not require a hysterectomy.

A 23-year-old woman comes to the clinic for a Pap smear. After the examination, the client confides that her mother died of endometrial cancer 1 year ago and says that she is afraid that she will die of the same cancer. Which risk factor stated by the client after an education session on risk factors indicates that further teaching is needed?

Late-onset menarche

After an uneventful pregnancy a client gives birth to an infant with a meningocele. The neonate has 1-minute and 5-minute Apgar scores of 9 and 10, respectively. What is the priority nursing care for this newborn?

Protecting the sac with moist sterile gauze

A client is admitted with a diagnosis of preeclampsia. What significant clinical finding does the nurse expect when reviewing the client's history?

Proteinuria

A breastfeeding mother asks the nurse what she can do to ease the discomfort caused by a cracked nipple. What should the nurse instruct the client to do?

Start feedings on the unaffected breast until the affected breast heals

After an emergency cesarean birth, the client tells the nurse that she was hoping for a "natural" childbirth but is glad that she and her baby are all right." Which postpartum phase of adjustment does this statement most closely typify?

Taking-in -By discussing the experience, the client is bringing it into reality; this is characteristic of the taking-in phase. The client is not ready to assume the tasks of the letting-go phase until the tasks of the taking-in and taking-hold phases have been completed. The taking-hold phase is marked by an increased desire to resume independence. The working-through phase is not a separate phase of adjustment to parenthood; this is not relevant.

A client has a cesarean birth. The nurse monitors the newborn's respiration because infants subjected to cesarean birth are more prone to atelectasis. Why does this occur?

The ribcage is not compressed, then released during birth.

At 38 weeks' gestation a client is admitted to the birthing unit in active labor, and an external fetal monitor is applied. Late fetal heart rate decelerations begin to appear when her cervix is dilated 6 cm and her contractions are occurring every 4 minutes and lasting 45 seconds. What does the nurse conclude is the cause of these late decelerations?

Uteroplacental insufficiency -Late decelerations are indicative of uteroplacental insufficiency and, left uncorrected, lead to fetal hypoxia, fetal myocardial depression, or both.

A nurse is caring for a client in the first stage of labor and an external fetal heart monitor is in place. What do the tracings indicate?

Variable decelerations

A client who has just begun breastfeeding complains that her nipples feel very sore. What should the nurse encourage the mother to do? (Select all that apply.)

1. Apply cool packs to her breasts to reduce the discomfort 2. Take the analgesic medication prescribed to limit the discomfort 5 Assume a different position when breastfeeding to adjust the infant's sucking

A nurse is teaching a class of expectant parents about changes that are to be expected during pregnancy. What changes does the nurse explain result from the melanocyte-stimulating hormone? (Select all that apply.)

1. Choasma 2. Linea nigra

A couple expresses a desire to use the rhythm method of birth control. The woman tells the nurse that she menstruates every 32 days. What should the nurse teach the couple about when the client's ovulation probably occurs?

14 days before the start of the next menses

A nurse determines that a newborn is in respiratory distress. Which signs confirm respiratory distress in the newborn? (Select all that apply.)

2 Cyanosis 4 Tachypnea 5 Retractions -Cyanosis occurs because of inadequate oxygenation. Tachypnea is a compensatory mechanism to increase oxygenation. Retractions occur in an effort to increase lung capacity. Crackles occur in the healthy newborn. Wheezing in the newborn is benign.

A nurse is discussing informed consent with a client who is scheduled for a hysterectomy. What should the informed consent include? (Select all that apply.)

2 Explanation of available alternative treatments 3 Answers to questions and concerns about the procedure 4 Complete description of the possible dangers and discomforts

A nurse is assessing a newborn with suspected retention of a fetal structure that will result in a congenital heart defect. Which fetal structures should undergo change after birth? (Select all that apply.)

2 Foramen ovale 4 Ductus arteriosus

A primigravida in her seventh week of gestation asks the nurse when she can expect to feel her baby move. The nurse replies that quickening usually occurs in the:

2. 20th week

A newborn of 30 weeks' gestation has a heart rate of 86 beats/min and slow, irregular respirations. The infant grimaces in response to suctioning, is cyanotic, and has flaccid muscle tone. What Apgar score should the nurse assign to this neonate?

2. 3 -A heart rate of less than 100 beats/min = 1; slow and irregular respirations = 1; grimaces in response to suctioning = 1; flaccid muscle tone = 0; and cyanosis = 0. This infant's Apgar score is 3. A score of 2 is too low. A score of 4 is too high, as is a score of 5.

A 16-year-old primigravida who appears to be at or close to term arrives at the emergency department stating that she is in labor and complaining of pain continuing between contractions. The nurse palpates the abdomen, which is firm and shows no sign of relaxation. What problem does the nurse conclude that the client is experiencing?

Abruptio placentae

What does an Apgar score recorded 5 minutes after birth help the nurse evaluate?

Adequacy of the transition to extrauterine life

A couple in their late 30s, expecting their first child, plans to have an amniocentesis. At what point in the pregnancy should the nurse tell the couple that the test it will be scheduled?

After the 14th week of pregnancy

When working with a client who has spontaneously aborted a pregnancy, it is important for the nurse to first deal with his or her own feelings about abortion, death, and loss so that he or she may :

Allow the clients to express their grief

A nurse provides a list of foods for a breastfeeding client with phenylketonuria (PKU) to avoid. Which nutrient is included on the list?

Amino acids -PKU is an inborn error of metabolism involving an inability to metabolize phenylalanine, an essential amino acid. Lactose, glucose, and fatty acids are all metabolized by people with PKU.

While monitoring the fetal heart rate (FHR) of a client in labor, the nurse identifies an increase of 15 beats more than the baseline rate of 135 beats/min that lasts 15 seconds. How should the nurse document this event?

An acceleration

A nurse who is caring for a client in labor uses nitrazine paper to test the pH of the client's leaking vaginal fluid. What color will the nitrazine paper turn if the leakage is amniotic fluid?

Blue

A new mother with class II heart disease tells a nurse that she is afraid that her heart condition will prevent her from caring for her baby and her home when she is discharged. How should the nurse respond?

By asking her to describe her concerns more fully

A pregnant client with severe preeclampsia is receiving IV magnesium sulfate. What should the nurse keep at the bedside to prepare for the possibility of magnesium sulfate toxicity?

Calcium gluconate

The nurse is teaching a prenatal breathing and relaxation class. What does the nurse suggest to best ease back discomfort during labor?

Having support persons use back massage techniques -The fetus exerts pressure against the spine during labor; back massage provides counterpressure, which eases the discomfort.

A nurse is caring for a client who is receiving IV magnesium sulfate for preeclampsia. At 37 weeks' gestation she gives birth to an infant weighing 4 lb. What clinical finding in the newborn may indicate magnesium sulfate toxicity?

Hypotonia

A nurse is caring for a client who is admitted to the birthing unit with a diagnosis of abruptio placentae. For what complication associated with this problem should the nurse monitor this client?

Hypovolemic shock

A client gives birth to a full-term male with an 8/9 Apgar score. What should the immediate nursing care of this newborn include?

Identifying the infant, assessing respirations, and keeping him warm

A woman who had a home birth brings the infant to the well-baby clinic on the third day after the birth, and the infant weighs 5% less than at birth. What does the nurse suspect as the cause of this weight loss?

Imbalance between nutrient intake and fluid loss

A nurse notes that a client is voiding frequently in small amounts 8 hours after giving birth. What should the nurse conclude about this small output of urine during the early postpartum period?

It may indicate retention of urine with overflow.

A woman with an active lifestyle is in her 30th week of pregnancy. Which activity will the nurse discourage?

Leg lifts and sit-ups

After a newborn has skin-to-skin contact with the mother, a nurse places the newborn under a radiant warmer. What complication is the nurse attempting to prevent?

Metabolic acidosis

What should be included in nursing care immediately after a sexual assault?

Obtaining the assault history from the client

At 32 weeks' gestation a client undergoes ultrasound, which reveals a low-lying placenta. What complication should the nurse anticipate as the client's pregnancy approaches term?

Painless vaginal bleeding

The nurse presents a program on breast self-examination. After a return demonstration the nurse concludes that she needs to review certain aspects of the teaching program. Which behavior by one of the students supports this conclusion?

Palpating each breast while in the sitting position

The nurse is caring for a client who is in the taking-in phase of the postpartum period. The area of health teaching that the client will be most responsive to is:

Perineal care

Which factor in a client's history increases the risk for osteoporosis?

Prolonged immobility

A nurse is planning to teach a new mother about breastfeeding. What should the nurse consider before preparing the client to breastfeed?

Suckling stimulates the release of oxytocin.

The parents of a newborn are concerned about red pinpoint dots on their infant's face and neck. How should the nurse explain the finding?

The cause is an increased intravascular pressure during birth.

During discharge teaching a client who just had a hysterectomy states, "After this surgery, I don't expect to be interested in sex anymore." What should the nurse consider before responding?

Many women incorrectly equate hysterectomy with loss of libido. -The uterus is often erroneously believed necessary for a satisfying sex life. Sexuality after hysterectomy should not be diminished, particularly because the fear of pregnancy no longer exists. Although the estrogen level is reduced, libido is influenced by psychological as well as hormonal factors. Although body image changes can interfere with sexuality, this is not an expectation for most women.

A health care provider determines that a fetus is in a breech presentation. For which complication should the nurse monitor the client?

Nonreassuring fetal signs, indicating prolapse of the cord

An expectant couple asks the nurse about the cause of low back pain during labor. The nurse replies that this pain occurs most often when the fetus is positioned:

Occiput posterior

A nurse is evaluating the rate of involution of a client's uterus on the second postpartum day. Where does the nurse expect the fundus to be located?

One or two fingerbreadths below the umbilicus -The fundus tends to stay at or slightly above the umbilicus for about 24 hours, then decrease in height by about one fingerbreadth per day. The location of the uterus during the first 24 hours postpartum is at or one fingerbreadth above the umbilicus

Which client should a nurse suspect is at increased risk for postpartum hemorrhage?

One who gives birth to an infant weighing 9 lb 8 oz -The risk for a postpartum hemorrhage is greater with large infants because the uterine musculature has been stretched excessively, thus impairing uterine contractions after the birth.

A postpartum client is being prepared for discharge. The laboratory report indicates that she has a white blood cell (WBC) count of 16,000/dL. What is the next nursing action?

Placing the report in the client's record because this is an expected postpartum finding

A strict vegetarian (vegan) becomes pregnant and asks the nurse whether there is anything special she should do in regard to her diet during pregnancy. What is most the important measure for the nurse to instruct the client to take?

Plan to eat from specific groups of vegetable proteins each day.

At 5 am, 2 hours after a long labor and vaginal birth, a client is transferred to the postpartum unit. What is the nurse's priority when planning morning care for this client?

Planning nursing care activities that provide time for the client to rest and sleep

A pregnant client asks the nurse for information about toxoplasmosis during pregnancy. What should the nurse teach the client?

Pork and beef should be cooked thoroughly.

A client at 40 weeks' gestation is admitted to the birthing unit in labor. During the initial examination the nurse uses Leopold maneuvers to palpate the abdomen. The purpose of this intervention is to assess the:

Position of the fetus -Duration of contractions is timed by lightly placing the examining hand on the fundus during a contraction or observing the electronic monitor. The station can be ascertained during a vaginal examination, which is not part of the Leopold maneuvers. The nurse palpates the abdomen to locate the head, back, and small parts of the fetus; the locations of these parts reveal the position of the fetus. Frequency of contractions is timed by lightly placing the examining hand on the fundus for several minutes or by observing the electronic monitor.

The nurse teaches a client who is to undergo amniocentesis that ultrasonography will be performed just before the procedure to determine the:

Position of the fetus and the placenta

A client with heart disease is admitted to the birthing suite. How can the nurse try to prevent the development of cardiac decompensation during her labor?

Positioning her on the side with her shoulders elevated.

Why is it important for the nurse to know the infant's gestational age and how it compares with the birthweight?

Potential problems may be identified.

A client is admitted in active labor at 39 weeks' gestation. During the initial examination the nurse identifies multiple red blister-like lesions on the edges of the client's vaginal orifice. Once the nurse has spoken to the practitioner and receive prescriptions, the priority nursing action is:

Preparing for a cesarean birth

A nurse weighs a neonate who is born at 29 weeks' gestation. The weight is 1619 g (3 lb 9 oz). In light of this weight and gestational age, how should this infant be classified?

Preterm

A client with severe preeclampsia in the high-risk unit is receiving an infusion of magnesium sulfate. If eclampsia were to occur, what action would the nurse take first?

Prevent injury

A primigravida in the first trimester tells a nurse that she has heard that hormones play an important role in pregnancy. Which hormone should the nurse tell the client maintains pregnancy?

Progesterone

A client is admitted to the birthing unit in active labor. Amniotomy is performed by the health care provider. What physiological change does the nurse expect to occur after the procedure?

Progressive dilation and effacement

A client in her 30th week of gestation is in preterm labor, and the practitioner prescribes betamethasone (Celestone). The client asks the nurse why she is being given this drug. As a basis for the response the nurse takes into consideration that it:

Promotes neonatal pulmonary maturity -Betamethasone (Celestone), a corticosteroid, accelerates lung maturity and reduces intravascular hemorrhage and necrotizing enterocolitis in the preterm neonate if given 24 hours before birth.

What antidote to the side effects of terbutaline (Brethine) should a nurse have available?

Propranolol (Inderal) -Propranolol (Inderal) is a beta-blocking agent that reverses the uterine inhibitory responses and cardiovascular effects of terbutaline (Brethine). Levodopa (L-dopa) is not an antidote for terbutaline; it is used in Parkinson's disease. Furosemide (Lasix) is a diuretic; it will not reverse the cardiovascular effects indicated. Ritodrine (Yutopar) may cause responses similar to those of terbutaline; it is sometimes used to halt premature labor because it inhibits beta2- receptors

A nurse is teaching a breastfeeding client about medications that are safe and unsafe for her to take. Which medication is contraindicated?

Propylthiouracil (PTU) -The concentration of propylthiouracil (PTU) excreted in breast milk is three to 12 times higher than its level in maternal serum; this may cause agranulocytosis or goiter in the infant. Heparin (Hep-Lock) is not excreted in breast milk. The amount of breast milk excretion of gentamicin (Garamycin) is unknown, but it can be given to infants directly without adverse effects. Diphenhydramine (Benadryl) is excreted in breast milk, but it does not adversely affect the infant when therapeutic doses are given to the mother.

A nurse is discussing immunizations needed to confer active immunity with a pregnant client during her first visit to the prenatal clinic. What information should the nurse consider including that the client will understand with regard to active immunity?

Protein substances are formed by the body to destroy or neutralize antigens.

A client at 36 weeks' gestation is admitted to the high-risk unit because she gained 5 lb in the previous week and there is a pronounced increase in blood pressure. What is the initial intervention in the client's plan of care?

Providing a dark, quiet room with minimal stimuli -Increasing cerebral edema may predispose the client to seizures; therefore stimuli of any kind should be minimized

What is the most important parameter for the nurse to monitor during the first 24 hours after the birth of an infant at 36 weeks' gestation?

Respiratory distress

The gravida 1 now para 1 woman delivered a 7-lb 6 oz female infant at 11 pm yesterday after a labor of 14 hours. After breakfast the nursery staff brings the baby to the new mother. The mother smiles at the baby, then asks that the nurse take the baby back to the nursery because she has not had a shower yet. One hour later the nurse returns with the infant. Again the mother smiles at the baby; then she holds her, kisses her, and feeds her a bottle. Immediately after feeding the baby, the mother calls the nursery and asks that the baby be picked up so she can take a nap. What behavior is the new mother demonstrating?

Taking-in

An adolescent who gave birth one day ago confides to the nurse that she hopes that her baby will be good and sleep through the night. What should the nurse include in the plan of care to facilitate a realistic expectation of a nighttime newborn schedule?

Talk softly and cuddle the baby when crying occurs

A couple who recently immigrated from Israel tell a nurse in the prenatal clinic that they are concerned about a genetic disease that is prevalent among Jewish people. Which genetic screening should the nurse expect the health care provider to recommend to determine the possibility of the couple's child's inheriting the disease?

Tay-Sachs disease -Tay-Sachs disease is a genetic disorder transmitted as an autosomal recessive trait that occurs primarily among Ashkenazi Jews. Cystic fibrosis, Phenylketonuria, and Turner syndrome do not have a higher prevalence in the Jewish population.

A 14-year-old emancipated minor at 22 weeks' gestation comes in for her second prenatal examination. As she enters the examination room with her mother, she tells the nurse that she does not want her mother present for the examination. What should the nurse say?

Telling the mother, "I'm sorry, but I need to ask you to stay in the waiting area."

A nurse is teaching a childbirth preparation class about the discomfort of labor. What is the greatest influence on the perception of pain for a woman in labor?

Tension of the client

When a client at 39 weeks' gestation arrives at the birthing suite she says, "I've been having contractions for 3 hours, and I think my water broke." What will the nurse do to confirm that the membranes have ruptured?

Test the leaking fluid with nitrazine paper

A young sexually active client at the family planning clinic is advised to have a Papanicolaou (Pap) smear. She has never had a Pap smear before. What should the nurse include in the explanation of this procedure?

The Pap smear can detect cancer of the cervix.

A nurse caring for a client who gave birth to a healthy neonate evaluates the client's uterine tone 8 hours later. How does the nurse determine that the uterus is demonstrating appropriate involution?

The amount of lochia rubra is moderate.

A female client with Hodgkin disease is to start total nodal irradiation. She and her partner, who are planning a family, become concerned when they learn that the radiation therapy includes the pelvic area. Before responding what must the nurse consider?

The eggs in the ovaries can be removed and frozen for future use.

A client underwent a mastectomy 24 hours ago. Which information will the nurse include in the plan of care?

The hand and elbow of the affected arm will be elevated above the shoulder.

After a client undergoes a biopsy for suspected cervical cancer, the laboratory report reveals a stage 0 lesion. What does a nurse conclude about this client's stage of cancer?

The lesion is carcinoma in situ.

An internal fetal monitor is applied while a client is in labor. What should the nurse explain about positioning while the monitor is in place?

The most comfortable position may be assumed.

A genetic counselor is working with a couple, each of whom is a carrier of an autosomal recessive disorder. Which statement indicates that the couple has understood the teaching about this disorder?

"There is a 1-in-4 chance of having a child with the disorder." - Mendelian genetic theory, when both parents are carriers of an autosomal recessive disorder there is a 25% probability that a child will have the disorder. There is a 25% probability that a child born to this couple will have the disorder. The statement that none will have the disorder, indicates that the couple does not understand Mendel's theory of probability. When both partners are carriers there is a 50% probability that a child will be a carrier and a 25% probability that a child will have the disorder. If one of the parents has the disorder there is a 50% probability that a child will have the disorder.

While a mother is inspecting her newborn she expresses concern that her baby's eyes are crossed. How should the nurse respond?

"This is expected. Your baby is trying to focus."

At 42 weeks' gestation a client gives birth to an 8-lb 5-oz newborn. On examining the infant, what does the nurse expect to observe? (Select all that apply.)

1 Long nails 2 Wrinkled skin

Which observations are suggestive of postmaturity in a newborn male? (Select all that apply.)

1 Profuse scalp hair 2 Parchmentlike skin 5 Creases covering the entire soles

A client at 12 weeks' gestation arrives in the prenatal clinic complaining of cramping and vaginal spotting. A pelvic examination reveals that the cervix is closed. Which probable diagnosis should the nurse expect?

Threatened abortion

What is the nurse's initial action immediately after assisting with a precipitous birth in the triage area of the emergency department?

Warming the newborn

One statement by a breastfeeding mother that indicates that the nurse's teaching about stimulating the let-down reflex has been successful is "I will:

Apply warm packs and massage my breasts before each feeding."

On the third postpartum day a client who is breastfeeding calls the clinic complaining of hot, hard, aching breasts. What recommendation should the nurse include in the response?

Apply warm, moist cloths to both breasts before nursing.

A client in labor is experiencing discomfort because her fetus is in the occiput posterior position. What nursing action will help relieve this discomfort?

Applying pressure against her sacrum

A nurse is caring for a newborn with a myelomeningocele. What should immediate nursing care for this infant include?

Applying sterile, moist nonadherent dressings to the sac

An infant born at 40 weeks' gestation weighs 6 lb 13 oz (3090 g). What category describes this neonate?

Appropriate for gestational age (AGA) and term

What is the best nursing action for a client in active labor whose cervix is dilated 4 cm and 100% effaced with the fetal head at 0 station?

Assist the client's coach in helping her with the use of breathing techniques

What characteristic does the nurse anticipate in an infant born at 32 weeks' gestation?

Barely visible areolae and nipples -Breast tissue is not palpable in a newborn of less than 33 weeks' gestation. The ear pinnae spring back in an infant at 36 weeks' gestation. Creases of the palms and soles are not clearly defined until after the 37th week of gestation. A zero-degree square window sign is present in an infant at 40 to 42 weeks' gestation.

A nurse is counseling a pregnant client with iron-deficiency anemia about when and how to take supplemental iron. What time of day and with what drink is iron absorption most efficient?

Before breakfast with orange juice -Iron should be taken before breakfast, on an empty stomach, to permit maximal absorption; ascorbic acid enhances the absorption of iron.

After a client gives birth she has the following vital signs: temperature 99.4° F (37.4° C); pulse rate 80 beats/min and regular; respiratory rate of 16 breaths/min, with even respirations; and blood pressure of 148/92 mm Hg. Which vital sign should the nurse continue to monitor?

Blood pressure

After the client gives birth her vital signs are temperature 99.3° F (37.4° C); pulse 80 beats/min, regular and strong; respirations 16/min, slow and even; and blood pressure 148/92 mm Hg. Which vital sign should the nurse check more frequently?

Blood pressure

A 16-year-old adolescent at 24 weeks' gestation visits the prenatal clinic for the first time. After the physical examination she tells the nurse, "I can't believe how big I am. Will I get much bigger?" What information about adolescent growth and development does the nurse need to know before responding?

Body image is very important to adolescents, so pregnant teenagers are concerned about body size.

A woman who was discharged recently from the hospital after undergoing a hysterectomy calls the clinic and states that she has tenderness, redness, and swelling in her right calf. What should the nurse instruct the client to do?

Call an ambulance to go to the emergency department.

Which behavior indicates to a nurse that a new mother is in the taking-hold phase?

Calling the baby by name -The mother has moved into the taking-hold phase when she takes control and becomes actively involved with her infant and calls the infant by name. She has completed the taking-in phase when her own needs no longer predominate. Talking about the labor and birth in the taking-in phase when she has the need to integrate the experience. Touching the baby with her fingertips is the initial early action of the taking-in phase. Being involved with the infants need to eat and sleep is part of the taking-in phase.

A client is being prepared for an emergency cesarean birth because of fetal compromise. What is the most important preoperative nursing action?

Confirming the signed consent

A client who has undergone a cesarean birth because of the presence of active genital herpes is transferred to the postpartum unit. What type of isolation precautions does the nurse plan to institute?

Contact

The nurse is caring for a newborn with caput succedaneum. The nurse is able to differentiate caput succedaneum from cephalhematoma because caput succedaneum features scalp edema that:

Crosses the suture line

A newborn whose mother has type 1 diabetes is receiving a continuous infusion of fluids with glucose. What should the nurse do when preparing to discontinue the IV?

Decrease the rate slowly

A preterm infant with respiratory distress syndrome (RDS) has blood drawn for an arterial blood gas analysis. What test result should the nurse anticipate for this infant?

Decreased blood pH

A client arrives at the clinic in preterm labor, and terbutaline (Brethine) is prescribed. For what therapeutic effect should the nurse monitor the client?

Decreased frequency and duration of contractions -Terbutaline sulfate (Brethine) is a β-mimetic that acts on the smooth muscles of the uterus to reduce contractility, which in turn inhibits dilation and the frequency and duration of contractions. Although terbutaline may increase blood pressure and pulse, this is a side, not a therapeutic, effect requiring frequent assessments. Terbutaline is not an analgesic. It should stop cervical dilation rather than increase it.

A nurse in the clinic, during a routine prenatal visit, notes bruises on the client's upper arms. When questioned, the client responds that her boyfriend was upset and hit her. What is the priority nursing action?

Developing a safety plan with the client

What should the nurse emphasize in a class about childbirth?

Education; exercise, and breathing techniques

In the second stage of labor the nurse should plan to discourage a client from holding her breath longer than 6 seconds while pushing with each contraction. What complication does this prevent?

Fetal hypoxia

A client at 32 weeks' gestation is admitted in active labor. Her cervix is effaced and dilated 4 cm. Intramuscular betamethasone (Celestone) 12 mg is prescribed. What should the nurse tell the client about why the medication is being given?

Fetal lung maturity is accelerated. -A steroid such as betamethasone (Celestone) or dexamethasone (Decadron) administered to the mother crosses the placenta and promotes lung maturity in the fetus.

Two days after delivery, a client has a temperature of 101° F (38.3), general malaise, anorexia, and chills. What does the nurse expect to identify on the client's laboratory report?

Increased white blood cell (WBC) count

An intravenous infusion of magnesium sulfate is prescribed for a client with severe preeclampsia. The dosage is twice the usual adult dosage. When a nurse questions the dosage, the health care provider insists that it is the desired dosage and directs the nurse to administer the medication. How should the nurse respond to this directive?

Withhold the dose and notify the nurse manager

What does the nurse expect concerning the alveoli in the lungs of a 28-week-gestation neonate?

They have a tendency to collapse with each breath

Respiratory acidosis is confirmed in a neonate with respiratory distress syndrome when the laboratory report reveals:

An increased Paco 2 of 55 mm Hg

What should the nurse teach a client about performing breast self-examination?

Compress the nipples to check for discharge.

A client at 36 weeks' gestation exhibits oligohydramnios. What newborn complication should the nurse anticipate?

Intrauterine growth restriction (IUGR)

What is the focus of the nurse's anticipatory guidance during the first trimester of pregnancy?

Physical changes of pregnancy

The clinic nurse is planning care for a client found to have chlamydia. Which treatment should the nurse plan to implement?

Administration of azithromycin (Zithromax) 1 g orally in a single dose

A nitrazine test strip that turns deep blue indicates that the fluid being tested has a pH of:

7.5

After 8 postpartum hours the nurse determines that a client's fundus is 3 cm above the umbilicus and displaced to the right. Which statement is most significant in confirming the reason for the location of the uterus?

"I went to the bathroom, but I can't seem to urinate." -Not being able to urinate, in conjunction with the other findings, suggests urine retention. Thirst is unrelated to the other findings; it is related to dehydration. Changing the pad once in 8 hours is an expected postpartum response. Contractions while breastfeeding are expected; oxytocin secretion, which is stimulated by infant suckling, causes the uterus to contract.

A premenopausal client is scheduled for a total abdominal hysterectomy. Which statement indicates to the nurse that the preoperative teaching has been understood?

"I won't have any more menstrual periods after this surgery." - Correct4 "I won't have any more menstrual periods after this surgery." Menstrual flow is the shedding of the endometrial lining of the uterus, and a hysterectomy is the removal of the uterus

The nurse determines that a young female client who is being treated for a sexually transmitted infection (STI) understands instructions regarding future sexual contacts. Which statement confirms the nurse's conclusion?

"I won't have unprotected sex again, and I'll tell my partners to be tested for STIs."

The nurse is conducting teaching for a client being discharged after an abdominal hysterectomy. Which statement by the client indicates a need for further teaching?

"I'm glad I'll be able to get back into my jogging routine next week."

A client in labor is being prepared for a cesarean birth. What is the most important nursing intervention before anesthesia is administered?

Obtaining informed consent

The nurse is caring for a client who has just received epidural anesthesia. Which finding would be of most concern?

Hypotension

On a routine prenatal visit the sign or symptom that a healthy primigravida at 20 weeks' gestation will most likely report for the first time is:

Quickening

A nurse determines that the husband of a client in the early phase of labor understands the teaching from childbirth classes when he helps his wife use the breathing pattern of:

Slow-chest

A client in active labor starts screaming, "The baby is coming! Do something!" What is the first nursing action?

Checking the perineal area for the presenting part

What should the nurse do to enhance a neonate's behavioral development?

Help the parents stimulate their awake baby through touch, sound, and sight

A client is admitted in preterm labor. After intravenous tocolytic medications are administered, contractions cease and she is discharged. She is to receive oral terbutaline (Brethine) 5 mg every 6 hours at home. When should the nurse advise the client to take the medication?

With food `

A primigravida complains of morning sickness. What should the nurse plan to teach her?

Avoid long periods without food

A multigravida in active phase of labor says, "I feel all wet. I think I urinated." What should the nurse do first?

Inspect her perineal area

The fetus of a client in labor is found to be at +1 station. What location does +1 station describe?

Just below the ischial spines

A client who has had a mastectomy asks what the term ERP-positive means. How should the nurse explain this finding?

The tumor cells generally exhibit a positive response to hormone therapy that reduces estrogen.

During a childbirth class the nurse determines that the women understand how to use effleurage correctly when they are observed:

Massaging their abdomens gently with their fingertips -Effleurage is a gentle massage of the abdomen that is effective during the first stage of labor because it distracts the client from the discomfort of the contractions. Rocking gently on the knees, known as the pelvic rock, is used during pregnancy to relieve backache. Practicing panting to avoid pushing during labor is a technique of breathing. Taking deep breaths before imagined contractions is also a technique of breathing.

A breastfeeding mother experiences redness and pain in the left breast, a temperature of 100.8° F (38.2° C), chills, and malaise. What condition does the nurse suspect?

Mastitis

A nurse who is caring for a mother and her newborn infant reviews their record. In light of the data the record contains, what nursing intervention is required?

Maternal rubella vaccination

A pregnant client is experiencing nausea and vomiting. The nurse determines that this discomfort:

May be related to an increased human chorionic gonadotropin level

A 36-year-old multigravida who is at 14 weeks' gestation is scheduled for an α-fetoprotein test. She asks the nurse, "What does this test do?" The nurse bases the response on the knowledge that this test can reveal:

Neural tube defects

The nurse teaches a pregnant client why she needs a folic acid supplement. Which neonatal disorder does folic acid prevent?

Neural tube defects -A folic acid supplement (0.4 mg/day) greatly reduces the incidence of fetal neural tube defects.

A client at 42 weeks' gestation has a reactive nonstress test. The nurse determines that the client understands what she was taught about the results when she is overheard telling her husband that the test was:

Normal because of an increase in fetal heart rate (FHR) with fetal movement

A nurse in the family planning clinic reviews the health history of a sexually active 16-year-old girl whose chief concern is a thick, burning discharge accompanied by a burning sensation and lower abdominal pain. After an examination the girl is informed that she may have a sexually transmitted infection (STI) that requires treatment. The adolescent is concerned that her parents will discover that she has been sexually active and asks the nurse whether her parents will be contacted. The nurse explains that her parents will:

Not be contacted, because treatment at the clinic is confidential

A newborn's hands and feet are cyanotic and there is circumoral pallor when the infant cries or feeds. What should the nurse do?

Notify the practitioner, because circumoral pallor may indicate cardiac problems

A postpartum client is scheduled to have a tubal ligation. She has asked that her husband not be told about the procedure because she has told him that she is having exploratory surgery. The client's husband asks the nurse why his wife needs to have exploratory surgery. How should the nurse respond?

"Have you talked to your wife about your concerns?"

A nurse teaches a client with asthma about her illness during pregnancy. Which statement by the client indicates that the nurse's teaching has been effective?

"I can use my albuterol inhaler if it's absolutely necessary."

A woman who has just delivered an infant asks to take the placenta home with her and her new baby on discharge. What is the most appropriate response?

"I need to check the hospital protocol for our policy on that practice."

A pregnant client whose first child has Down syndrome is about to undergo amniocentesis. The client tells the nurse that she does not know what she will do if this fetus has the same diagnosis. The client asks the nurse, "Do you think abortion is the same as killing?" How should the nurse respond?

"I really can't answer that question. Are you ambivalent about abortion?"

After a client has a spontaneous abortion at 12 weeks' gestation, the nurse notes that she and her partner are visibly upset. The partner has tears in his eyes, and the client is sobbing quietly with her face turned to the wall. At this time, what is the nurse's most therapeutic statement?

"I'll be here if you want to talk."

A nurse is teaching a postpartum client the characteristics of lochia and any deviations that should be reported immediately. What client statement indicates that the teaching was effective?

"I'll notify the clinic if my lochia starts to smell bad." -Lochia has a characteristic menstrual musky or fleshy smell. A foul-smelling discharge, along with fever and uterine tenderness, suggests an infection. Passing clots is a common occurrence. Lochia changing from red to pink is expected as lochia rubra progresses to lochia serosa. Although many women have a minimal discharge after 2 weeks, it is not uncommon for lochia alba to last 6 weeks.

A client in active labor is rushed from the emergency department to the labor and birth suite screaming, "Knock me out!" Examination reveals that her cervix is dilated 9 cm. What should the nurse say while trying to calm the client?

"Medication may interfere with the baby's first breaths; keep breathing." -Analgesia crosses the placental barrier ; when birth is imminent, it can cause respiratory depression in the newborn.

A nurse is teaching a prenatal class about infant safety. After the class several of the students are heard discussing what they have learned. The nurse determines that the teaching has been effective when one of the future parents states:

"My mother can't believe that babies are supposed to sleep on their backs, not their stomachs."

A client at 16 weeks' gestation is being treated for Trichomonas vaginalis. Which statement best indicates to the nurse that the client has learned measures to prevent a recurrence?

"My partner has to get treated before we have sex again."

A female client who has been sexually active for 5 years is found to have gonorrhea. The client is upset and asks the nurse, "What can I do to keep from getting another infection in the future?" Which statement by the client indicates that the teaching by the nurse was effective?

"My partner has to use a condom all the time."

A 17-year-old client tells the nurse that her sister had an ectopic pregnancy about 3 months ago and had to have her fallopian tube removed. The nurse determines that this young woman needs additional information when she states:

"My sister is lucky, because she won't have a period again."

After the birth of her daughter, a mother tells the nurse, "I was told that my baby has to have an injection of vitamin K. She's so small to be getting a shot. Why does she have to have it?" How should the nurse respond?

"Newborns are deficient in vitamin K. This treatment will protect your baby from bleeding."

A client who is breastfeeding tells a nurse that her breasts are swollen and painful. What can the nurse teach her to do to limit engorgement?

"Nurse at least every 3 hours for at least 10 minutes on each breast."

A client asks the nurse what she should do if she forgets to take the pill one day. How should the nurse respond?

"On the next day take one pill in the morning and one before bedtime."

A couple arrives at the newborn nursery asking to take their newborn grandson to his mother's room. What is the best response by the nurse?

"Please go on to see your daughter. I'll bring the baby to her room."

A client who is scheduled for an amniocentesis says, "I'm glad that this test will show if my baby is well." How should the nurse respond?

"Potential defects caused by chromosomal errors can be detected."

The nurse concludes that a couple with a newborn with Erb's palsy has an accurate understanding of the infant's prognosis. Which statement confirms this conclusion?

"Recovery usually occurs in about 3 months."

A client has just given birth to an infant with Down syndrome. The mother is crying and asks the nurse what she is supposed to do now. What is the nurse's best response?

"Tell me what you know about Down syndrome."

Which statement made by a pregnant client to a nurse indicates that the client does not understand the teaching about fetal growth and development?

"The baby gets food from the amniotic fluid." -The amniotic fluid serves as a protective environment; the fetus depends on the placenta, along with the umbilical blood vessels, for nutrients and oxygen. "The baby is smaller if the mother smokes," "The baby's oxygen is provided by the mother," and "The baby's umbilical cord has two arteries and one vein" are all true statements, and further teaching would not be required.

A pregnant client in the third trimester tells the nurse in the prenatal clinic that she has heartburn after every meal. What explanation should the nurse give about the cause of the heartburn?

"The cardiac sphincter relaxes and allows acid to be regurgitated."

A pregnant client tells the nurse in the prenatal clinic that although she and her husband do not have the disease, she has a 1-year-old daughter with sickle cell anemia. She asks the nurse, "Will this baby also have sickle cell anemia?" How should the nurse respond?

"The chance that another child will have sickle cell anemia is 25%."

After receiving a diagnosis of placenta previa, the client asks the nurse what this means. What is the nurse's best response?

"The placenta is implanted in the lower uterine segment, and it's covering part or all of the cervical opening."

The family of a pregnant client with myasthenia gravis asks the nurse whether the client will be an invalid. What is the best response by the nurse?

"The progression is slow, so people with myasthenia will spend their younger life with few problems."

A mother is inspecting her newborn girl for the first time. The infant's breasts are edematous, and she has a pink vaginal discharge. How should the nurse respond when the mother asks what is wrong?

"The swelling and discharge are expected. They're a response to your hormones."

On her first postpartum day, a client asks the nurse whether her baby has had a test for phenylketonuria (PKU) yet. How should the nurse reply?

"The test won't be done until your baby has had enough milk for the results to be accurate." -The PKU test cannot be done until the newborn has ingested a high-phenylalanine (formula or breast milk) diet for at least 24 hours.

During a childbirth class, several participants have questions about the elective induction of labor. One participant states that it is more convenient for a woman with a busy schedule. What evidenced-based information should the nurse provide to the participant?

"The widespread use of elective induction increases the risk of unfavorable outcomes."

On the first day after a mastectomy, a nurse encourages the client to perform exercises such as flexion and extension of the fingers and pronation and supination of the hand. The client asks why she has to do these exercises. The best response by the nurse is:

"They will help stimulate peripheral circulation."

A client has just been informed by the health care provider that she has cervical polyps. The client asks the nurse whether she should worry about them. How should the nurse respond?

"They're usually benign, and a biopsy rules out a malignancy."

A new mother asks the nurse administering erythromycin ophthalmic ointment to her newborn why her baby must be subjected to this procedure. What is the best response by the nurse?

"This antibiotic helps keep babies from contracting eye infections."

A 63-year-old woman with the diagnosis of estrogen-receptor positive cancer of the breast undergoes lumpectomy and radiation therapy, and tamoxifen (Nolvadex) is prescribed. The client asks the nurse how long she will have to take the medication. The nurse responds:

"You'll need to take it for 5 years, after which it will be discontinued." -Tamoxifen is an estrogen antagonist antineoplastic medication that has been found to be effective in 50% to 60% of women with estrogen receptor-positive cancer of the breast. After 5 years of administration there is an increased risk of complications and the drug is discontinued. Tamoxifen usually is prescribed for 5 years after initiation of therapy, not for the rest of the client's life.

Four weeks after giving birth, a client is agitated and tells the clinic nurse, "The baby cries all the time, and I don't know what to do." What question should the nurse ask before planning nursing care?

"What is the baby's daily schedule?"

A client visiting the prenatal clinic for the first time tells the nurse that she has heard conflicting stories about sex during pregnancy and asks about continuing sexual activity. How should the nurse respond?

"With an uncomplicated pregnancy, there are no limitations on sexual activity."

An older female client tells the nurse in the clinic that she has a cystocele that was diagnosed a year ago. She has urinary frequency and burning on urination. The client asks, "The doctor wanted me to have surgery for the cystocele last year, but I can manage with peripads. It won't hurt not to have surgery, will it?" How should the nurse respond?

"Yes, you're risking kidney damage." -A cystocele is a herniation of the bladder through the vaginal wall resulting from weakened pelvic structures. In this condition the herniated bladder does not empty effectively and urinary stasis, chronic Infection; and renal failure may result. The surgery improves bladder function and prevents renal failure; it is needed. Bowel obstruction is a complication of a rectocele, not cystocele. Although corrective surgery will reduce perineal pressure, its primary purpose is to improve bladder function and prevent complications

A woman visits the clinic for an annual physical examination, and herpes genitalis is diagnosed. The client asks how the disease can be diagnosed without any tests. How should the nurse reply?

"You have blisters on the skin around your vagina." -Herpes genitalis is characterized by a cluster of vesicles, not one lesion or a rash or vaginal discharge. The characteristic sign of herpes genitalis is a cluster of vesicles (blisters) on the vulva, perineum, vagina, cervix, and/or perianal area. These rupture spontaneously, leaving painful erosions.

A multipara who is admitted to the hospital for repair of a rectocele and cystocele asks a nurse why these problems happened to her. How should the nurse respond?

"You have relaxation of the muscles in your lower pelvis."

A married couple has been using oral contraceptives to delay pregnancy. When the wife misses her regular menstrual period, she decides to find out whether she is pregnant. She tells the nurse that pregnancy may have occurred because she missed her contraceptive pills for 1 week when she had the flu. How should the nurse respond?

"You may be correct. The effect of contraceptive pills depends on their being taken on a regular schedule."

A young woman has been using oral contraceptives. When she misses her regular menstrual period, she visits the women's health clinic and tells the nurse that she may be pregnant because she missed taking her contraceptive pills for 1 week when she had the flu. How should the nurse respond?

"You may be right. One of the reasons that an exact schedule is prescribed for birth control pills is that they have to be taken regularly to be effective."

A postpartum client tells the nurse that she wishes to breastfeed. When the nurse brings her newborn to be breastfed, the client asks whether she may drink a small glass of wine to help her relax. How should the nurse respond?

"You seem a little tense. Tell me how you feel about breastfeeding." -Stating that the client seems tense and initiating a discussion honors the client's feelings and encourages expression of them; there is no reference to alcohol consumption and its relaxing effects. Alcohol ingestion should not be encouraged because it enters the breast milk.

A client who just gave birth has three young children at home. She comments to the nursery nurse that she must prop the baby during feedings when she returns home because she has too much to do and, anyway, holding babies during feedings spoils them. What is the nurse's best response?

"You seem concerned about time. Let's talk about it."

A client asks the nurse at the family planning clinic whether contraception is needed while she is breastfeeding. How should the nurse reply?

"You should use contraceptives, because ovulation may occur without a period." -Anovulation occurs in nursing mothers for varying periods; breastfeeding is not a reliable method of birth control. Periods may not occur for several months; sexual relations need not be delayed this long. Ovulation can occur without menstruation. Lactation may delay menses but does not reliably suppress ovulation.

During the postpartum period a client with heart disease and type 2 diabetes asks a nurse, "Which contraceptives will I be able to use to prevent pregnancy in the near future?" How should the nurse respond?

"You should use foam with a condom to prevent pregnancy — this is the safest method for women with your illnesses."

A client is scheduled for a laparoscopic bilateral tubal ligation. What should the nurse include in preoperative teaching?

"You will be admitted as an outpatient for same-day surgery." -A laparoscopic tubal ligation takes about 20 minutes to perform. The client is admitted as an outpatient and goes home the same day after she recovers from the anesthesia.

A client who is scheduled to have an abdominal panhysterectomy asks how the surgery will affect her periods. How should the nurse respond?

"You won't have any more periods."

The parents of a newborn tell the nurse that they do not want their infant's eyes treated with a prophylactic agent. How should the nurse respond?

"You'll have to sign an informed consent to refuse the treatment."

A new mother who wishes to breastfeed her infant asks a nurse whether she needs to alter her diet. How should the nurse respond?

"You'll need extra amounts of the same foods you've been eating, plus more fluids."

A new mother wishes to breastfeed her infant and asks the nurse whether she needs to alter her diet. How should the nurse respond?

"You'll need greater amounts of the same foods you've been eating and more fluids."

A client at 35 weeks' gestation asks a nurse why her breathing has become more difficult. How should the nurse respond?

"Your diaphragm has been displaced upward."

During a counseling discussion of nutrition, a nurse explains to a pregnant client that she will need additional calcium during pregnancy and that the best source is milk. The client states, "I never drink milk or eat milk products. They turn my stomach." What is the nurse's best reply?

"Your practitioner can prescribe calcium supplements."

A client at 39 weeks' gestation is admitted for induction of labor. Knowing that several medications are used to induce labor, a nurse identifies those that may be prescribed. (Select all that apply.)

1 Oxytocin (Pitocin) 2 Misoprostol (Cytotec) 5 Dinoprostone (Prepidil)

What actions are part of nursing care during the fourth stage of labor for the client with a fourth-degree laceration? (Select all that apply.)

1 Pain management with oral analgesics 3 Assessment of the site every 15 minutes 5 Application of an ice pack for 20-minute intervals

A nurse is assessing a client who is being admitted for surgical repair of a rectocele. What signs or symptoms does the nurse expect the client to report? (Select all that apply.)

1 Painful intercourse 3 Bearing-down sensations -The posterior vaginal wall is pushed forward by the herniation of the rectum; this protrusion causes painful intercourse. The posterior vaginal wall is pushed forward by the herniation of the rectum; this protrusion increases rectal pressure and causes the bearing-down sensation. A rectocele is not accompanied by abdominal pain. Urinary stress incontinence is the primary sign of a cystocele. A cystocele, not a rectocele, is associated with urinary tract infections.

A nurse is caring for an adolescent in labor an hour after she was admitted to the birthing unit. The adolescent is anxious and tense. She cries during contractions and asks the nurse for epidural anesthesia. The nurse obtains the adolescent's current vital signs and reviews her history and admission information. What nursing interventions are essential before epidural anesthesia is administered? (Select all that apply.)

1 Performing a baseline vaginal examination 2 Telling the adolescent what to expect with each procedure 3 Identifying risk factors that contraindicate epidural anesthesia

A nurse is testing a newborn's heel blood for the level of glucose. Which newborn does the nurse anticipate will experience hypoglycemia? (Select all that apply.)

1 Preterm infant 3 Small-for-gestational-age infant 4 Large-for-gestational-age infant

An infant born in the 36th week of gestation weighs 4 lb 3 oz (2062 g) and has Apgar scores of 7 and 9. What nursing actions will be performed on the infant's admission to the nursery? (Select all that apply.)

1 Recording of vital signs 4 Evaluation of the neonate's health status 5 Supportive measures to keep the neonate's body temperature stable

After a difficult birth, a neonate has an Apgar score of 8 after 5 minutes. Which signs met the criteria of 2 points? (Select all that apply.)

1 Reflex irritability: cry 2 Respiratory rate: good cry 3 Heart rate: 110 beats/min

What are the primary nursing interventions when a client is receiving an infusion of magnesium sulfate for severe preeclampsia? (Select all that apply.)

1 Restricting visitors 4 Maintaining a quiet environment

A nurse is being oriented to a prenatal clinic after graduation. The new nurse takes a course on several tests during pregnancy. Place the tests in the order in which they should be performed during pregnancy.

1 Sickle cell screening 2 α-Fetoprotein (AFP) testing for neural tube defects 3 Serum glucose for gestational diabetes 4 Fetal movement test 5 Group B Streptococcus culture

Which characteristics should alert the nurse to conclude that a male newborn is a preterm infant? (Select all that apply.)

1 Small breast buds 2 Wrinkled thin skin 5 Pinnae that remain flat when folded

What are the indicators of nutritional risk in pregnancy in a client who is of normal weight? (Select all that apply.)

1 Smoker 2 Twin gestation

A nurse is observing the newborn of a known opioid user for signs of withdrawal. What clinical manifestations does the nurse expect to identify? (Select all that apply.)

1 Sneezing 2 Hyperactivity 3 High-pitched cry

A pregnant woman at 6 week's gestation tells the nurse at her first prenatal visit that she uses an over-the-counter herbal product as a health supplement that has been approved by the Food and Drug Administration. What should the nurse recommend to the client? (Select all that apply.)

1 Stop taking the supplement immediately. 2 Discuss the use of the supplement with the practitioner. 5 Discuss the use of any over-the-counter products with the practitioner.

The nurse is teaching a sex education course to high school students. What should the nurse teach them about why gonorrhea is difficult to control? (Select all that apply.)

1 Symptoms of the disease are vague. 3 The incubation period is relatively short. 4 Causative organisms have become resistant to treatment. -Many clients with gonorrhea are asymptomatic. The incubation period is 3 to 5 days. There is no effective readily available blood test for gonorrhea. Gonorrhea responds well to treatment, but the Centers for Disease Control and Prevention has received several reports of resistant strains; at times backup secondary medications must be used. Urethral/vaginal smears or cultures are specific for the identification of the gonococcal organism.

A pregnant client is concerned that she may have been infected with HIV. What information should a nurse include when counseling this client about HIV testing? (Select all that apply.)

1 The risks of passing the virus to the fetus 2 What positive or negative test results indicate 5 The emotional, legal, and medical implications of test results

A new mother's laboratory results indicate the presence of cocaine and alcohol. Which craniofacial characteristic indicates to the nurse that the newborn has fetal alcohol syndrome (FAS)? (Select all that apply.)

1 Thin upper lip 3 Small upturned nose 5 Smooth vertical ridge in the upper lip

A primigravida at 34 weeks' gestation tells the nurse that she is beginning to experience some lower back pain. What should the nurse recommend that the client do? (Select all that apply.)

1 Wear low-heeled shoes 4 Perform pelvic tilt exercises several times a day

A client in labor is admitted to the birthing room. The exam reveals that the fetus is at -1 station. Where is the presenting part?

1 cm above the ischial spines

A nurse is caring for a postpartum client. Where does the nurse expect the fundus to be located if involution is progressing as expected 12 hours after birth?

1 cm above the umbilicus

A woman has made the decision to have breast augmentation surgery, and the procedure is to be performed on an outpatient basis. As part of the preoperative protocol, the nurse provides teaching regarding the discharge instructions. Which instructions apply to this type of surgery? (Select all that apply.)

1. Avoid taking aspirin or NSAIDs (e.g., ibuprofen [Advil]) for pain relief. 2 Sleep with your head and torso elevated for at least 1 week. 5 Take your temperature daily and notify the clinic if it goes above 99.6° F.

In childbirth classes the nurse is teaching paced breathing techniques for use during labor. In which order should the breathing techniques be used as labor progresses?

1. Cleansing breaths 2. Slow, deep breaths 3. Modified paced breathing 4. Pant-blow breathing 5. Slow, exhalation pushing

Contraceptives that contain estrogen-like and/or progesterone-like compounds are prepared in a variety of forms. Which contraceptives should the nurse tell clients have a hormonal component? (Select all that apply.)

1. Oral drugs 6, Transdermal agents

By how much should the nurse instruct a pregnant client to increase her daily protein intake?

10g -Ten grams is the amount of increase in the daily protein intake recommended for pregnant women by the Food and Nutrition Board of the National Academy of Sciences. Twenty, 30, and 40 g are all more than the recommended amount, although most women in developed countries exceed the requirement.

A 16-year-old primigravida at 36 weeks' gestation visits the prenatal clinic for a routine examination. Her blood pressure is significantly increased, and there is 1+ proteinuria. The client's blood pressure had been averaging 92/70 mm Hg during her previous prenatal visits. What is the lowest blood pressure that should cause the nurse to become concerned?

122/86 mm Hg -An increase of 30 mm Hg systolic and/or 15 mm Hg diastolic has been removed from the official definition of preeclampsia. The new definition encourages practitioners to consider the total situation in determining a diagnosis of preeclampsia.

A client who had a child with Tay-Sachs disease is pregnant and is to have an amniocentesis to determine whether the fetus has the disease. The nurse counsels her to plan for the procedure at the optimal time for the procedure at:

14 to 16 weeks' gestation -An amniocentesis is done at this time because a therapeutic abortion may be legally and safely performed if desired by the parents. Six to 8 weeks' gestation is too early to perform an amniocentesis because the uterus has not ascended into the abdomen and there is little amniotic fluid present. Although an amniocentesis and therapeutic abortion may be performed at this time, it is preferred that they be done as early as possible. Twenty-two to 24 weeks' gestation is too late; the parents should not delay amniocentesis if they are considering a therapeutic abortion.

A pregnant woman tells a nurse in the prenatal clinic that she knows that folic acid is very important during pregnancy and that she is taking a prescribed supplement. She asks the nurse what foods contain folic acid (folate) so she may add them to her diet in its natural form. Which foods should the nurse recommend? (Select all that apply.)

4 Black and pinto beans 5 Enriched bread and pasta

A client's nipples become sore and tender as a result of her newborn's vigorous suckling. What should the nurse recommend that the mother do to alleviate the soreness? (Select all that apply.)

4 Expose the nipples to air several times a day. 5 Apply hydrogel pads to the nipples after each feeding.

A nurse is trying to determine whether a pregnant woman's membranes have ruptured. What findings support the conclusion that they have ruptured? (Select all that apply.)

4 Nitrazine paper turns blue on contact with the fluid. 5 Microscopic examination of the fluid reveals ferning/

A nurse in a family planning clinic determines that a client understands the discussion about using a cervical cap with a spermicide when the client states that after intercourse, a cervical cap must be left in place for at least:

6 hours

The nurse instructs a pregnant client in the sources of protein that can be used to meet the increased daily requirement during pregnancy. How many grams of protein should the client eat each day?

60 g

A primigravida is admitted to the emergency department with a sharp, shooting pain in the lower abdomen and vaginal spotting. A ruptured tubal pregnancy is diagnosed. During what week of gestation does this condition most commonly occur?

6th

At 1 minute after birth the nurse determines that an infant is crying, has a heart rate of 140 breaths/min, has blue hands and feet, resists the suction catheter, and keeps the legs flexed and the arms extended. What Apgar score should the nurse assign?

8

One minute after birth a nurse notes that a newborn is crying, has a heart rate of 140 beats/min, is acrocyanotic, resists the suction catheter, and keeps the arms extended. What Apgar score should the nurse assign to the newborn? Record your answer using a whole number. ___

8

A practitioner prescribes penicillin G benzathine suspension (Bicillin L-A) 2.45 million units for a client with a sexually transmitted infection (STI). The medication is available in a multidose vial of 10 mL in which 1 mL = 300,000 units. How many milliliters should the nurse administer? Record your answer using one decimal place. ____ mL.

8.2

Which client is most at risk for osteoporosis?

A 66-year-old white woman, 5 foot 1 inch and 100 lb, who is a paralegal -A postmenopausal woman who is small-boned, thin, and relatively sedentary is at risk for osteoporosis; other risk factors are family history, and white or Asian ethnicity. The postmenopausal years are considered to be 65 years and older; however, each individual is unique. A perimenopausal woman who is relatively heavy and does not smoke is at less risk for osteoporosis than is a thin postmenopausal woman. The perimenopausal years are considered to be 45 to 64 years of age; however, each individual is unique. Postmenopausal women who are black are at lower risk for osteoporosis than are white and Asian women. A perimenopausal woman who takes a daily calcium supplement is at less risk for osteoporosis than a woman who does not take a calcium supplement.

A registered nurse (RN) on the postpartum unit is providing care to four maternal/infant couplets and is running behind. Which nursing action is best delegated to a licensed practical nurse/licensed vocational nurse (LPN/LVN) who also works on the unit?

Administering 2 tablets of acetaminophen and oxycodone (Percocet) to a client who rates her pain as 7 of 10

At 12 weeks' gestation, a client who is Rh negative expels the total products of conception. What is the nursing action after it has been determined that she has not been previously sensitized?

Administering RhoGAM within 72 hours

A woman in active labor arrives at the birthing unit. She tells the nurse that she was found to have a chlamydial infection the last time she visited the clinic but that she stopped taking the antibiotic after 3 days because she "felt better." What would the nurse anticipate as part of the plan of care, in light of this history?

Administration of antibiotics before delivery

A nurse is observing the electronic fetal monitor as a client in labor enters the second stage. The nurse identifies early decelerations of the fetal heart rate with a return to the baseline at the end of each contraction. What does this usually indicate?

Fetal head compression -Early decelerations are expected occurrences as the fetal head passes through the birth canal; the fetal heart rate returns to baseline quickly, indicating fetal well-being. The data do not indicate that the mother has diabetes. Variable decelerations occur with umbilical cord compression, not prolapse. Maternal hypotension will cause late decelerations because of fetal hypoxia.

A primigravida at term is admitted to the birthing room in active labor. Later, when the client is dilated 8 cm, she tells the nurse that she has the urge to push. The nurse instructs her to pant-blow at this time because pushing can cause which of the following?

Cervical edema -The head cannot emerge when the cervix is not fully dilated. Pushing in this situation may cause cervical edema, predisposing the client to cervical laceration.

During prenatal classes the nurse teaches the difference between true labor and false labor. How does the nurse explain the difference?

Cervix effaces and dilates during true labor.

A client in the birthing suite has spontaneous rupture of the membranes, after which a prolapsed cord is identified. The nurse calls for help and with a sterile gloved hand moves the fetal head off the cord. What should the nurse anticipate?

Cesarean birth

During the postpartum period a client tells a nurse that she has been having leg cramps. Which foods should the nurse encourage the client to eat?

Cheese and broccoli

A nurse is teaching a client about the oral contraceptive prescribed by the primary health care provider. Which condition identified by the client indicates understanding of when the drug should be stopped immediately and the health care provider notified?

Chest pain

Shortly after birth the nurse instills erythromycin ophthalmic ointment in the newborn's eyes. The father asks why an antibiotic is needed because the mother does not have an infection. The nurse explains that it protects the newborn from:

Chlamydia and gonorrhea

A nurse in the clinic determines that a 4-day-old neonate who was born at home has a purulent discharge from the eyes. What condition does the nurse suspect?

Chlamydia trachomatis infection

A client who is in labor is admitted 30 hours after her membranes ruptured. For what condition does the nurse anticipate that the client is most at risk?

Chorioamnionitis -The risk of developing chorioamnionitis (intra-amniotic infection) is increased with prolonged rupture of the membranes; foul-smelling fluid is a sign of infection. A prolapsed cord usually occurs shortly after the membranes rupture, not 1½ days later. Placenta previa is an abnormally implanted placenta; it is unrelated to ruptured membranes. Premature separation of the placenta is unrelated to ruptured membranes.

A couple interested in family planning ask the nurse about the cervical mucus method of family planning. The nurse explains that with this method the couple must avoid intercourse when and a few days after the cervical mucus is:

Clear and stretchable

The day after a client has a cesarean birth, the indwelling catheter is removed. The nurse concludes that urinary function has returned when the:

Client voids 300 mL of urine within 4 hours of catheter removal

A client in active labor is admitted to the birthing room. A vaginal examination reveals that the cervix is dilated 6 to 7 cm. In light of this finding, the nurse expects that the:

Client's contractions will become longer and more frequent

A nurse assesses a healthy 8-lb 8-oz (3860-gm) newborn who was given Apgar scores of 9 at 1 minute and 10 at 5 minutes. Which category of the Apgar score received a 1 rating at one minute?

Color

After a modified radical mastectomy a client has two portable wound drainage systems in place. What is an important intervention as the nurse cares for these drainage systems?

Compressing the drainage receptacles after emptying them to maintain suction -Portable wound drainage systems are self-contained and may be emptied and compressed to reestablish negative pressure, which promotes drainage. Portable wound drainage systems are not irrigated; they drain by way of negative pressure. Portable wound drainage systems have collection chambers, so another drainage system is not needed. Portable wound drainage systems are self-contained closed systems.

A client at 35 weeks' gestation who has had no prenatal care arrives in labor and delivery and is found to be 20 percent effaced and 2 cm dilated, with her membranes intact and contractions 3 minutes apart. The nurse notices some ruptured blisterlike vesicles in the genital area. What should the nurse's next action be?

Contacting the health care provider about the need for a cesarean birth

One hour after a birth a nurse palpates a client's fundus to determine whether involution is taking place. The fundus is firm, in the midline, and two fingerbreadths below the umbilicus. What should the nurse do next?

Continue periodic evaluations and record the findings

How does the nurse know whether a client is in true labor?

Contractions occur every 5 to 10 minutes; the cervix is dilated 2 cm and 75% effaced, and dilation has increased to 3 cm in 2 hours.

What potential complication should the nurse anticipate when a pregnant client has premature rupture of the membranes?

Cord prolapse

What is the best method for the nurse to use when evaluating blood loss in a client with placenta previa?

Count or weigh perineal pads

A 20-year-old developmentally disabled woman is a resident in a group home. She has had four abortions in the past 2 years, and the agency supervisor recommends that she be sterilized. It is obvious that the client is unable to exercise informed consent for sterilization. The nurse understands that the procedure cannot be performed without legal consent from the:

Court-appointed individual or group

A client is admitted to the birthing room in active labor. The nurse determines that the fetus is in the left occiput posterior (LOP) position. At what point can the fetal heart be heard?

D

A client who has just had a cesarean birth is receiving IV fluids and has an indwelling catheter. The client's fluid intake will need to be increased when the nurse identifies:

Dark amber urine -A dark amber or tea color indicates highly concentrated urine and requires additional hydration of the client.

A client with type 1 diabetes is scheduled for an amniocentesis at 36 weeks' gestation. She asks the nurse why this is being done so late in her pregnancy. What should the nurse consider before responding?

Fetal lung maturity may be evaluated.

The nurse explains to a pregnant client undergoing a nonstress test that the test is a way of evaluating the condition of the fetus by comparing the fetal heart rate with:

Fetal movement

A pregnant client who is scheduled for a nonstress test (NST) asks a nurse how the test can show that "my baby is all right." The nurse explains that it is a way of evaluating the condition of the fetus by comparing the fetal heart rate (FHR) with:

Fetal physical activity -The FHR should increase with physical activity; a reactive NST reveals accelerations of 15 beats/min, lasting 15 seconds with fetal movement. This response indicates fetal well-being.

A nonstress test is scheduled for a client with preeclampsia. During the nonstress test the nurse concludes that if nonperiodic accelerations of the fetal heart rate occur with fetal movement, this probably indicates:

Fetal well-being

What is the primary outcome for client care in the third stage of labor?

Firmly contracted uterine fundus -The third stage of labor spans the time from the birth of the baby to the delivery of the placenta; a firmly contracted uterus is desired because it minimizes blood loss. Providing comfort is a desirable goal but is secondary to the life-threatening possibility of hemorrhage associated with a boggy uterus. Efficient fetal heart beat-to-beat variability is a concern in the first and second stages of labor; it is no longer applicable after the fetus is born. The maternal respiratory rate may vary above or below this range

During the examination of a client in labor, the cervix is determined to be dilated 4 cm. What stage of labor does the nurse record?

First -The first stage of labor is from zero cervical dilation to full cervical dilation (10 cm). The second stage is from full cervical dilation to delivery. The prodromal stage is before cervical dilation begins. The transitional phase is first stage of labor, from 8 cm of dilation to 10 cm of dilation

A client is admitted with a diagnosis of stage 0 cervical cancer (carcinoma in situ). What does the nurse emphasize while helping the client understand her diagnosis and prognosis?

Five-year survival rates for this cancer are nearly 100% with early treatment.

An infant is born in the breech position and diagnosed with Erb palsy (Erb-Duchenne paralysis). What clinical manifestation supports this conclusion?

Flaccid arm with the elbow extended on the affected side

A nurse evaluates that a client who is taking oral contraceptives understands the related dietary teaching when the client states, "While I'm taking birth control pills I should increase my intake of foods containing:

Folic acid."

A client who is pregnant for the first time and carrying twins is scheduled for a cesarean birth. What should preoperative teaching include?

Frequent ambulation is begun within 24 hours.

A client is admitted to the birthing unit with uterine tenderness and minimal dark-red vaginal bleeding. She has a marginal abruptio placentae. The priority evaluation includes fetal status, vital signs, skin color, and urine output. What additional information is essential?

Fundal height

A pregnant client is admitted to the high-risk unit with uterine tenderness and some dark-red vaginal bleeding. Abruptio placentae is diagnosed. What priority evaluation should be included with vital signs, skin color, urine output, and fetal heart rate?

Fundal height

After 2 weeks of radiation therapy for cancer of the breast a client experiences some erythema over the area being radiated. The area is sensitive but not painful. She states that she has been using tepid water and a soft washcloth when cleansing the area and applying an ice pack three times a day. What does the nurse conclude from this information?

Further teaching on skin care is necessary.

While a client is being interviewed on her first prenatal visit she states that she has a 4-year-old son who was born at 41 weeks' gestation and a 3-year-old daughter who was born at 35 weeks' gestation and lost one pregnancy at 9 weeks and another at 18 weeks. Using the GTPAL system, how would you record this information?

G5 T1 P1 A2 L2 -The client is gravida (G) 5: the current pregnancy, the 41-week pregnancy, the 35-week pregnancy, the 9-week pregnancy, and the 18-week pregnancy. She has had one term (T) pregnancy (one that lasts 40 weeks plus or minus 2 weeks): the 41-week pregnancy. The 35-week pregnancy is considered preterm (P). Pregnancies that end before 20 weeks are considered abortions, so the losses at 9 and 18 weeks would be scored as A2. The other options do not consider the present pregnancy or the correct definitions of term and preterm or do not include the abortions.

The husband of a woman who had her fourth child 3 weeks ago states she has been irritable and crying frequently since bringing her newborn home. He asks the nurse whether this is normal. The nurse tries to help him understand the situation by stating that:

Having four children is tiring and assistance may be needed.

In the second hour after the client gives birth her uterus is firm, above the level of the umbilicus, and to the right of midline. What is the most appropriate nursing action?

Having the client empty her bladder -A full bladder elevates the uterus and displaces it to the right. Even though the uterus feels firm, it may relax enough to foster bleeding. Therefore the bladder should be emptied to improve uterine tone. Watching for signs of retained secundines may be done if emptying the bladder does not rectify the situation. If parts of the placenta, umbilical cord, or fetal membranes are not fully expelled during the third stage of labor, their retention limits uterine contraction and involution; a boggy uterus and bleeding may be evident. Vigorous massage tires the uterus, and even with massage the uterus is unable to contract over a full bladder. Explaining to the client that this is a sign of uterine stabilization is not correct; the uterus will not remain contracted over a full bladder.

When a nurse who is carrying a newborn to the mother enters the room, a visitor asks to hold the infant. The visitor is sneezing and coughing. What is the most important measure for the nurse to take?

Having the visitor step outside the room

A nurse determines that a client who, although ambivalent, is considering an abortion because of financial difficulties and is in crisis. How should the nurse intervene to alleviate the crisis?

Helping the client express her feelings

How should a nurse direct care for a client in the transition phase of the first stage of labor?

Helping the client maintain control

During a client's labor the fetal monitor reveals a fetal heart pattern that signifies uteroplacental insufficiency. What is the nurse's first intervention?

Helping the client turn to the side-lying position

During the fourth stage of labor, the assessment of a primipara who has had a vaginal birth reveals a moderate to large amount of lochia rubra, a firm fundus that is at the umbilicus and deviated to the right, and pain that she rates as a 3 on a scale of 1 to 10. What is the priority nursing action?

Helping the client void -A fundus that is deviated to the right during the fourth stage of labor commonly is caused by a distended bladder ; if the bladder remains distended, involution will be inhibited, resulting in a boggy uterus that is prone to hemorrhage. The fundus is firm and does not need to be massaged. The fundus is firm; there is no need to increase the rate of the oxytocin (Pitocin) infusion. Because the client's pain is minimal, the priority is emptying the bladder to prevent hemorrhage.

A primipara delivered 12 hours ago. Although an ice bag has been applied to her perineal area, the client continues to complain of rectal pressure resulting in excruciating pain in the area of the episiotomy that is not relieved with analgesics. What does the nurse conclude is the cause of the client's pain?

Hematoma in the perineal area -Pain becomes excruciating with hematoma development at the episiotomy site because of pressure on surrounding nerve endings. This pain is not relieved by the application of ice because ice only reduces edema formation around the incision.

A woman's pregnancy has been uneventful, and she has gained 25 lb. At term her hemoglobin level is 10.6 g/dL and her hematocrit is 31%. What does the nurse identify as the reason for these hemoglobin and hematocrit levels?

Hemodilution

A nurse is caring for a client who has had a spontaneous abortion. For what complication should the nurse monitor this client?

Hemorrhage

A client presents to the clinic with complaints of nausea and amenorrhea and reports that she obtained a positive result on a home pregnancy test. Which component of the history is most indicative of pregnancy?

Her urine immunoassay test is positive,

A client who is at 12 weeks' gestation tells a nurse at the prenatal clinic that she is experiencing severe nausea and frequent vomiting. The nurse suspects that the client has hyperemesis gravidarum. What factor is frequently associated with this disorder?

High level of chorionic gonadotropin -A high level of chorionic gonadotropin is frequently associated with severe vomiting during pregnancy and may result in hyperemesis gravidarum. A high level may also occur in the presence of a hydatidiform mole or multiple pregnancy.

A 31-year-old client is seeking contraceptive information. Before responding to the client's questions about contraceptives, the nurse obtains a health history. What factor in the client's history indicates to the nurse that oral contraceptives are contraindicated?

History of borderline hypertension -Oral contraceptives may cause or exacerbate hypertension; borderline hypertension places the client at risk for a brain attack

A primigravida at 12 weeks' gestation complains of nausea and vomiting during a visit to the prenatal clinic. Which pregnancy hormone should the nurse explain is thought to be responsible for nausea and vomiting during the first trimester?

Human chorionic gonadotropin (hCG)

A nurse is caring for four clients, each with a different medical condition. Which condition should the nurse anticipate will result in the client's being instructed by the health care provider not to breastfeed?

Human immunodeficiency virus -Breastfeeding by a mother infected with HIV is contraindicated because breast milk can transmit the virus to the infant

A 24-year-old client complains to the nurse in the women's health clinic that her breasts become tender before her menstrual period. What should the nurse recommend that the client do 1 week before an expected menses?

Decrease caffeine intake -The client is exhibiting one symptom of premenstrual syndrome (PMS); eliminating food and beverages containing caffeine can limit breast swelling. Salt intake should be reduced premenstrually to limit the development of edema. Increased protein intake is unnecessary if the client is eating a nutritious diet. Exercise should be increased before the menstrual period to help ease the symptoms of PMS.

A local anesthetic (pudendal block) is administered to a client as second-stage labor begins. For what side effect does the nurse monitor for the client?

Decreased blood pressure

A client with worsening preeclampsia is admitted to the high-risk unit, and the nurse manager places her in a private room. A nonstimulating environment is important for a client with increased cerebral irritability because it:

Decreases the probability of generalized seizures

A client's temperature is 100.4° F 12 hours after a spontaneous vaginal birth. What does the nurse suspect is the cause of the increased temperature?

Dehydration

A client who recently was told by her practitioner that she has extensive terminal metastatic carcinoma of the breast tells the nurse that she believes an error has been made. She states that she does not have breast cancer, and she is not going to die. The nurse determines that the client is experiencing the stage of death and dying known as:

Denial

A pregnant client at 30 weeks' gestation begins to experience contractions every 5 to 7 minutes. She is admitted with a diagnosis of preterm labor. Although the client is being given tocolytic therapy her cervix continues to dilate, and it is determined that a preterm birth is inevitable. Which medication does the nurse expect the health care provider to prescribe?

Dexamethasone -Dexamethasone is a glucocorticoid that stimulates the production of fetal lung surfactants, which are needed for fetal lung maturity; administration is started 48 hours before the expected birth. Norgestrel is a contraceptive hormone; it is not used for preterm labor. Aminophylline is a bronchodilator; it is not used for preterm labor. Magnesium sulfate is used for tocolytic therapy and has been somewhat effective in delaying preterm labor. However, this client's labor is progressing and the birth is inevitable.

What is the safest and most reliable birth control method for the nurse to recommend to a client with type 1 diabetes?

Diaphragm with a spermicidal gel

A client in labor is receiving an oxytocin (Pitocin) infusion. What should the nurse do first when repetitive late decelerations of the fetal heart rate are observed?

Discontinue the oxytocin infusion

A primigravida who is at 38 weeks' gestation is undergoing a nonstress test. The nurse determines that the baseline fetal heart rate is 130 to 140 beats/min. It rises to 160 on two occasions and 157 once during a 20-minute period. Each of the episodes in which the heart rate is increased lasts 20 seconds. What action should the nurse take?

Discontinuing the test because the pattern is reassuring

A client is found to have preeclampsia, and bedrest at home is prescribed. It is doubtful that this client will be able to comply because she has two preschool children. What should be included in the plan of care that may help the client follow the prescribed regimen?

Discuss why bedrest is necessary

In a noisy room a sleeping newborn initially startles and exhibits rapid movements but soon goes back to sleep. What is the most appropriate nursing action in response to this behavior?

Documenting an intact reflex

A primigravida asks when she will be able to hear the fetal heartbeat for the first time. The nurse should explain that the heartbeat can be heard with:

Doppler ultrasound at 10 to 12 weeks

Sonography of a primigravida who is at 15 weeks' gestation reveals a twin pregnancy. The nurse reviews with the client the risks of a multiple pregnancy that were explained by the health care provider. Which condition does the client identify that indicates the need for further instruction about complications associated with a multiple gestation?

Down syndrome

While reviewing laboratory results of clients seen at a maternity clinic, the nurse notes that one client's maternal serum α-fetoprotein level is lower than is typical. The nurse recognizes that this may be associated with:

Down syndrome

A client at 35 weeks' gestation calls the prenatal clinic, concerned that she has "not felt the baby move as much as usual." The most appropriate recommendation by the nurse is to have the client call the clinic with the results after she has:

Drunk a glass of orange juice and timed 10 fetal movements -Drinking orange juice can increase fetal movement. Fetal kick count, either the number counted in 30 minutes or the time it takes for 10 kicks to occur, is the accepted method of assessing the fetus for the appropriate amount of movement.

The health care provider hands a neonate to a nurse immediately after birth. What should the nurse do next for the newborn?

Dry and provide skin-to-skin contact with the mother -The priority is preventing heat loss; drying the newborn prevents heat loss through evaporation, and skin-to-skin contact with the mother provides a warm environment while promoting attachment.

A nurse is caring for four clients, each with a different medical condition. Which condition should the nurse anticipate will result in the client's being instructed by the health care provider not to breastfeed?

Human immunodeficiency virus -Breastfeeding by a mother infected with HIV is contraindicated because breast milk can transmit the virus to the infant. Breastfeeding by a mother with mastitis is not always contraindicated; during antibiotic treatment the mother can maintain lactation by pumping the breasts and discarding the milk. Once the infection has resolved, breastfeeding may be resumed.

A 28-year-old woman is recovering from her third consecutive spontaneous abortion in 2 years. What is the most therapeutic nursing intervention for this client at her follow-up appointment?

Encouraging the client to verbalize her feelings about the loss

While caring for a client during labor, the nurse remembers that the second stage of labor:

Ends at the time of birth

Two days after birth a neonate's head circumference is 16 inches (40 cm) and the chest circumference is 13 inches (32.5 cm). What does the nurse infer from these measurements?

Enlarged head

A nurse in the fertility clinic is instructing a client who will be using progesterone gel vaginally in the treatment of luteal phase infertility. When discussing the side effects of progesterone, what should the nurse tell the client to expect?

Enlarged, tender breasts

The cervix of a client in labor is fully dilated and 100% effaced. The fetal head is at +3 station, the fetal heart rate ranges from 140 to 150 beats/min, and the contractions, lasting 60 seconds, are 2 minutes apart. What does the nurse expect to see when inspecting the perineum?

Enlarging area of caput with each contraction -The client should be pushing with each contraction; with the head at +3 station, each push will bring more of the caput into view at the vaginal opening. It is too early for the perineum to be stretched to the point of tearing; if this should occur later, an episiotomy may be performed. Meconium is discoloring the amniotic fluid; it is an unexpected finding that may indicate that the fetus is at risk. There is a decreased, not an increased, amount of amniotic fluid at the end of labor.

During a prenatal interview at 20 weeks' gestation, the nurse determines that the client has a history of pica. What is the most appropriate nursing action?

Ensuring that the client's diet is nutritionally adequate

A client is taking a progesterone oral contraceptive (minipill). The nurse instructs the client to take one pill daily during the:

Entire menstrual cycle

A nurse discusses the type of anesthesia that will be used for a vaginal birth with a client who has class I cardiac disease. Which type of block is most appropriate for this client?

Epidural

At 30 weeks' gestation a client with class II cardiac disease expresses concern about her labor and asks the nurse what to expect. What does the nurse tell the client to expect if cardiac decompensation occurs?

Epidural anesthesia with a vacuum extraction birth

A nurse is caring for a client with placenta previa who is in labor. What action is most important for the nurse to take?

Evaluating external blood loss by counting pads

What is the priority nursing intervention during the 2 hours after a cesarean birth?

Evaluating the lochia to identify the complication of hemorrhage

Immediately after birth, a newborn is dried before being placed in skin-to-skin contact with the mother. What type of heat loss does this intervention prevent?

Evaporation

A pregnant client with sickle cell anemia visits the clinic each month for a routine examination. What additional observation should be made during every visit?

Evidence of pyelonephritis

When discussing future health management with a client who has had a total hysterectomy, the nurse advises regular physical examinations. The client agrees and adds, "It won't be so hard to go now that I won't need the pelvic examination and Pap smear." How should the nurse respond?

Explain why regular pelvic examinations and Pap smears of vaginal secretions will be necessary in the future.

What is the best nursing intervention to achieve the cooperation of an extremely anxious pregnant client during her first pelvic examination?

Explaining the procedure and maintaining eye contact while touching the client gently

A resident practitioner in the birthing unit asks the nurse to prepare for a vaginal examination on a client with a low-lying placenta who is in early labor. What is the priority nursing action?

Explaining why a vaginal examination should not be performed -A vaginal examination may cause separation of the placenta, resulting in hemorrhage. The nurse should discuss the situation with the resident, away from the client, because it is imperative that a vaginal examination not be performed without preparation for a cesarean birth.

A nurse in the fertility clinic works with couples who have been trying to become pregnant for more than 1 year. How can the nurse help ease the feeling of isolation that infertile couples often experience?

Explore ways to promote communication with family and friends

A nurse teaching a prenatal class is asked why infants of diabetic mothers are larger than those born to women who do not have diabetes. On what information about pregnant women with diabetes should the nurse base the response?

Extra circulating glucose causes the fetus to acquire fatty deposits.

A pregnant client with cardiac disease asks a nurse to clarify what she was told about making the birth easier for her. What should the nurse remind her is an option to facilitate birth?

Facilitating the birth with vacuum extraction -Vacuum extraction will decrease the workload of the heart during expulsion and permit a vaginal birth.

During their first visit to the prenatal clinic a couple asks the nurse whether the woman should have an amniocentesis for genetic studies. Which factor indicates that an amniocentesis should be performed?

Family history of genetic abnormalities

During a class for prepared childbirth, the nurse teacher discusses the importance of the spurt of energy that occurs before labor. Why is it important to conserve this energy?

Fatigue may influence pain medication requirements. -Fatigue will interfere with the successful use of other coping strategies such as distraction; this may lead to the client's need for pain medication.

A nurse is planning care for a client who gave birth to a preterm male infant. What most common response does the nurse anticipate that the mother may experience?

Feelings of failure and loss of control

A client is concerned about gaining weight during pregnancy. What should the nurse tell the client is the cause of the greatest weight gain during pregnancy?

Fetal growth

A client measuring at 18 weeks' gestation visits the prenatal clinic stating that she is still very nauseated and vomits frequently. Physical examination reveals a brown vaginal discharge and a blood pressure of 148/90 mm Hg. What condition does the nurse suspect the client is experiencing?

Hydatidiform mole -A hydatidiform mole, in which chorionic villi degenerate into grapelike vesicles, is the cause of these signs and symptoms. Although vomiting may cause dehydration, this conclusion ignores the vaginal discharge and hypertension.

A nurse is caring for a client in labor who is receiving epidural anesthesia. For which common side effect of this route of anesthesia should the client be monitored?

Hypotensive episodes

Three days after birth, a breastfeeding newborn becomes jaundiced. The parents bring the infant to the clinic and blood is drawn for an indirect serum bilirubin determination, which reveals a concentration of 12 mg/dL. The nurse explains that what the infant has is physiological jaundice, a benign condition, caused by:

Immature liver function

A client making her first visit to the prenatal clinic asks which immunization can be administered safely to a pregnant woman. What should the nurse tell her?

Inactive influenza -The inactive influenza and diphtheria, tetanus, pertussis (dTAP) can be safely administered during the first trimester of pregnancy, although dTAP is recommended at 27 to 36 weeks' gestation to provide immunity to the mother and infant. Rubella (measles) and rubeola (German measles) are both live viruses that should never be administered during pregnancy because they can have teratogenic effects. The inactivated influenza vaccine may be given because it is a killed virus vaccine and will not have a teratogenic effect.

A client who suspects that she is 6 weeks pregnant appears mildly anxious as she is waiting for her first obstetric appointment. What symptom of mild anxiety does the nurse expect this client to experience?

Increased alertness -Increased alertness is an expected common behavior that occurs in new or different situations when a person is mildly anxious. The other options are a common sign of moderate to severe anxiety.

A nurse caring for a pregnant woman determines that she is engaging in the practice of pica. Why should the nurse prepare a teaching plan for this client?

Inedible items are being ingested. -Pica is the eating of inedibles such as starch or dirt. There is a cultural influence on this practice, but it may also be related to malnutrition or anemia. Food cravings frequently occur in pregnant women. If many foods are causing nausea and vomiting, the client has morning sickness. If it continues past the first trimester, it may be hyperemesis gravidarum. The dislike for essential food groups does not describe the practice of pica.

A client in preterm labor is receiving subcutaneous terbutaline (Brethine) tocolytic therapy. The nursing action that is most important during the initial administration of this medication is assessing the client's:

Pulse rate continuously -Tachycardia is an expected side effect of terbutaline , a betamimetic agent. The pulse rate should be monitored continuously, and the rate should be no more than 120 beats/min. The reflexes are not affected by terbutaline, which relaxes uterine musculature. This medication does not affect the client's level of consciousness. Although intake and output are assessed, an indwelling catheter is not indicated.

Identify the position of the fetus whose buttocks are in the fundus, whose fetal back is on the maternal right side between the midline, and lateral surface of the abdomen, and whose attitude is general flexion.

ROA

A nurse is assessing a primigravida who was admitted in early labor after her membranes ruptured. She is at 41 weeks' gestation. Her contractions are irregular and her cervix is dilated 3 cm. The fetal head is at station 0 and the fetal heart rate tracing is reactive. How can the nurse help the client facilitate labor?

Take a walk around the unit with her

A husband who is coaching his wife during labor demonstrates an understanding of the transition phase of labor when, as his wife starts to push with each contraction, he instructs her to:

Take quick, shallow breaths, and then blow

After being shown to the parents, a preterm male newborn weighing 3 lb 15 oz (1500 g) is moved to the neonatal intensive care unit. What should the nurse's plan for parental visits include?

Taking them to visit their son as soon as possible

A primipara about to be discharged with her newborn asks the nurse many questions regarding infant care. What phase of maternal adjustment does this behavior illustrate?

Taking-hold -The taking-hold phase, which begins about the second or third postpartum day, involves concern about being a "good" mother; the new mother is most receptive to teaching at this time. Let-down is not related to bonding. The let-down reflex refers to the flow of milk in response to suckling and is caused by the release of oxytocin from the posterior pituitary. The taking-in phase is the first period of adjustment to parenthood. It includes the first 2 postpartum days; the mother is passive and dependent and preoccupied with her own needs. The behavior described refers to the taking-hold phase of bonding. Early parenting involves many behaviors, of which taking hold is only one.

A client is admitted to the birthing unit because fluid is leaking from her vagina. She is unsure whether her "bag of water" has broken. What should the nurse do to help determine whether the fluid is amniotic fluid?

Test the fluid with nitrazine paper -Amniotic fluid is slightly alkaline, and urine is acidic; when moistened with amniotic fluid, nitrazine will turn dark blue, indicating an alkaline substance. Inspecting the fluid is a subjective assessment and may be inaccurate.

What should the care of a newborn infant whose mother has had untreated syphilis since the second trimester of pregnancy include?

Testing for congenital syphilis

A primipara gives birth to an infant weighing 9 lb 15 oz (4508 g). During labor a midline episiotomy is performed and the client sustains a third-degree laceration. The client tells the nurse that her perineal area is very painful. What should the nurse consider before explaining the reason for the pain?

The anal sphincter muscle has been injured. -A third-degree laceration extends through the perineal muscles and continues through the anal sphincter muscle. Cutting of the perineal muscles constitutes a second-degree laceration. Trauma to the rectum constitutes a fourth-degree laceration. Damage to superficial muscles is a first-degree laceration.

The nurse is to collect a blood sample for glucose testing from the infant of a diabetic mother. The previous sample revealed a level of 38 mg/dL. Mark the area that is safe for the heelstick.

The center of the heel must be avoided because of the presence of the plantar artery. The heelstick must be done because the previous blood glucose level, which was under 40, is indicative of hypoglycemia.

A nurse is caring for a client during the transition phase of labor. The nurse determines that the client has entered the second stage of labor when:

The client reports that she feels the urge to move her bowels

A client's membranes rupture spontaneously during the latent phase of the first stage of labor, and the fluid is greenish brown. What does the nurse conclude?

The fetus may be compromised in utero.

A nurse is caring for a client at 42 weeks' gestation who is having a contraction stress test (CST). What does a positive result indicate?

The function of the placenta has diminished.

The nurse is helping a mother breastfeed her newborn. What is the best indication that the newborn has achieved an effective attachment to the breast?

The mouth covers most of the areolar surface. -Parents need support and reassurance that their newborn is not permanently damaged. Cephalohematomas do not cause impaired neurological function

A pregnant client with diabetes is referred to the dietitian in the prenatal clinic for nutritional assessment and counseling. What should the nurse emphasize when reinforcing the client's dietary program?

The need to eat a pregnancy diet that meets increased dietary needs and to adjust the insulin dosage as necessary

In her 37th week of gestation, a client with type 1 diabetes has amniocentesis to determine fetal lung maturity. The lecithin/sphingomyelin ratio is 2:1, phosphatidylglycerol is present, and creatinine is 2 mg/dL. What conclusion should the nurse draw from this information?

The newborn should be free from respiratory problems. -These test results confirm fetal lung maturity, and the neonate should be free of major respiratory problems. They do not indicate the need for a cesarean birth.

A mother is seeing her newborn, who has visible birth defects, for the first time. When she sees her baby, she becomes disturbed, pushes away, and tells a nurse, "Oh, take the baby away; I never want to see it again." What does the nurse conclude from this behavior?

The response is a common one in a new mother who is finding it difficult to accept that her newborn is less than perfect.

A couple at the prenatal clinic for a first visit tells the nurse that their 2-year-old child has just been found to have cystic fibrosis. They state there is no family history of this disorder. They ask the nurse about the chances of their having another child with cystic fibrosis. Knowing that this disorder has an autosomal recessive mode of inheritance, how should the nurse respond?

There is a 25% chance the baby will be affected and a 50% chance that the baby will be a carrier.

A contraction stress test (CST) is performed on a client at 40 weeks' gestation. The findings are interpreted as negative. What does the nurse conclude from this interpretation?

There will be weekly retesting because, at this time, the fetus has oxygen reserves.

A nonstress test (NST) is scheduled for a client with mild preeclampsia. During the test, the client asks the nurse what it means when the fetal heart rate goes up every time the fetus moves. What should the nurse consider before responding?

These accelerations are a sign of fetal well-being.

The parents of a newborn ask the nurse about several areas of deep-blue coloring on their baby's lower back and buttocks. The nurse's response is based on the information that:

These areas usually are normal and will fade within the first year.

Continuous positive-pressure ventilation therapy by way of an endotracheal tube is started in a newborn with respiratory distress syndrome (RDS). The nurse determines that the infant's breath sounds on the right side are diminished and that the point of maximum impulse (PMI) of the heartbeat is in the left axillary line. How should the nurse interpret these data?

These findings indicate that the infant may have a pneumothorax, and the health care provider should be contacted immediately.

A client at 7 weeks' gestation tells a nurse in the prenatal clinic that she is sick every morning with nausea and vomiting and adds that she does not think she can tolerate it throughout her pregnancy. The nurse assures her that this is a common occurrence in early pregnancy and will probably disappear by the end of the:

Third month -Because of a decrease in chorionic gonadotropin, morning sickness seldom persists beyond the first trimester. Morning sickness usually ends at the end of the third month, when the chorionic gonadotropin level falls. It is still present in the second month because of the high level of chorionic gonadotropin.

A woman who is 28 weeks pregnant calls the clinic to report that she is frightened because she is leaking breast milk. The best response is to tell her that:

This can be a normal occurrence during pregnancy

A client gives birth vaginally, with a midline episiotomy, to an infant who weighs 8 lb 13 oz (4000 g). An ice pack is applied to the perineum to ease the swelling and pain. The client complains, "This pain in my vaginal and rectum is excruciating, and my vagina feels so full and heavy." What does the nurse suspect as the cause of the pain?

Vaginal hematoma

What nursing intervention should be implemented routinely after a client has a vacuum aspiration abortion?

Encouraging the client to take the prescribed antibiotic medication

A mother whose newborn infant son has a cleft lip and palate asks how to feed her baby because he has difficulty suckling. What information should the nurse provide concerning safe feeding technique for this infant?

"Give him brief rest periods and frequent burpings during feedings so he can get rid of swallowed air."

On her first prenatal visit a client says to the nurse, "I guess I'll be having an internal examination today." What is the nurse's best response?

"Have you ever had an internal examination done before?"

What signs and symptoms of withdrawal does the nurse identify in a postpartum client with a history of opioid abuse?

Irritability and muscle tremors

What should the nurse explain to a newly pregnant client with cardiac disease?

Maintenance dosages of cardiac medications will probably be increased.

Thirty minutes after a client gives birth, the nurse palpates the client's uterus. It is relaxed and the lochia is excessive. What is the nurse's initial action?Massage the uterus

Massage the uterus

A pregnant client's blood test reveals an increased alpha-fetoprotein (AFP) level. What condition does the nurse suspect that this result indicates?

Neural tube defect

A 35-year-old client is scheduled for a vaginal hysterectomy. She asks the nurse about the changes she should expect after surgery. How should the nurse respond?

"A hysterectomy doesn't affect the chronological age when menopause usually occurs." -As the term hysterectomy implies, only the uterus is removed. The ovaries remain; therefore, the client will experience menopause around the same time as women who have functioning ovaries. The client will ovulate and will not experience surgical menopause because the ovaries are not removed with a hysterectomy. There should be no discomfort if there is an appropriate period of healing before the resumption of sexual intercourse.

A client in preterm labor does not respond to therapy, and birth seems imminent. The client begins to cry and says, "I'm so worried about my baby." What is the nurse's best response?

"All of this must leave you very confused and frightened."

A nurse on the postpartum unit discusses breast care with a client who is formula feeding her newborn. Which statement indicates to the nurse that more teaching is needed?

"Applying heat to my breasts will help ease the discomfort."

A new mother asks the nurse whether she may wash her baby in a tub after they go home. What is the nurse's best response?

"Babies can be bathed in a tub after the cord has fallen off."

During a client's first visit to the prenatal clinic, a nurse discusses a pregnancy diet. The client states that her mother told her that she should restrict her salt intake. What is the nurse's best response?

"Because you need salt to maintain body water Balance; it is not restricted. Just eat a well-balanced diet."

While teaching a prenatal class about infant feeding, the nurse is asked about the relationship between breast size and ease of breastfeeding. How should the nurse respond?

"Breast size is not related to milk production."

The nurse has completed a prenatal class for women who are expecting their first babies. Which statement by a pregnant woman indicates the need for additional teaching?

"During pregnancy it's safe for me to use my regular herbal remedies."

A nurse is teaching a childbirth class to a group of pregnant women. One of the women asks the nurse at what point during the pregnancy the embryo becomes a fetus. How should the nurse respond?

"During the eighth week of the pregnancy."

One hour postpartum a nurse evaluates the amount of vaginal bleeding and determines that a client's uterus has become relaxed and boggy. What should the nurse do next?

Massage the uterus until firm

After 18 months of unsuccessful attempts at conception by a client, primary infertility related to anovulatory cycles is diagnosed. Clomiphene citrate (Clomid) is prescribed. The nurse concludes that the client understands the teaching about the correct time to take the clomiphene when the she states:

"I'll start the pills on the fifth day of my cycle." -The objective is to stimulate ovulation near the 14th day of the menstrual cycle, and this is achieved by taking the medication on the fifth through the ninth days; there is an increase in two pituitary gonadotropins luteinizing hormone and follicle-stimulating hormone, with subsequent ovarian stimulation.

The nurse is preparing a client for epidural anesthesia. Which client statement would cause the nurse to stop the placement of the epidural catheter?

"I'm not exactly sure how an epidural works."

A nurse is instructing a client to cough and deep-breathe after an emergency cesarean birth. The client says, "Get out of here. Can't you see that I'm in pain?" Which response will be the most effective?

"If you can't cough, try taking six very deep breaths."

While inspecting her newborn a mother asks the nurse whether her baby has flat feet. How should the nurse respond?

"Infants' feet appear flat because the arch is covered with a fat pad."

A new mother who has begun breastfeeding asks for assistance removing the baby from her breast. What should the nurse teach her?

"Insert your finger in the corner of the baby's mouth to break the suction."

A 24-year-old client who has had type 1 diabetes for 6 years is concerned about how her pregnancy will affect her diet and insulin needs. How should the nurse respond?

"Insulin dosage and dietary needs will be adjusted in accordance with the results of blood glucose monitoring."

A client is admitted with a diagnosis of torsion of the testes. How should the nurse respond when the client asks, "Why do I have to have surgery right now?"?

"Irreversible damage occurs after a few hours."

A woman at 22 weeks' gestation is admitted with heavy bleeding and severe abdominal cramping. When told that no fetal heart sounds can be detected, the client says to the nurse, "We wanted this baby so badly." How should the nurse respond?

"It must be difficult to lose this baby that was important to you both."

While waiting for his 39-year-old wife to change clothes after an amniocentesis, the husband says to the nurse, "I sure hope that they don't find anything wrong because of my wife's age. I don't know how we'd deal with a child with Down syndrome. We already have two small children at home." What is the nurse's best response?

"It must be difficult, worrying about whether your baby will be disabled."

A client seeking advice about contraception asks a nurse about how an intrauterine device (IUD) prevents pregnancy. How should the nurse respond?

"It produces a spermicidal intrauterine environment." -Intrauterine devices produce a spermicidal intrauterine environment. A copper IUD (ParaGard T380A) inflames the endometrium, damaging or killing sperm and preventing fertilization and/or implantation; a Mirena IUD (LNG-IUS) releases levonorgestrel, damaging sperm and causing the endometrium to atrophy, thus preventing fertilization and/implantation. A diaphragm blocks the cervical os.

On the first postpartum day, a client whose infant is rooming in asks the nurse to return her baby to the nursery and bring the baby to her only at feeding times. How should the nurse respond?

"It seems that you've changed your mind about rooming in."

A primigravida is admitted to the birthing suite at term with contractions occurring every 5 to 8 minutes and a bloody show. She and her partner attended childbirth preparation classes. Vaginal examination reveals that the cervix is dilated 3 cm and 75% effaced. The fetus is at +1 station in occiput anterior position, and the membranes are intact. The client is cheerful and relaxed and asks the nurse whether it is all right for her to walk around. In light of the nurse's observations regarding the contractions and the client's knowledge of the physiology and mechanism of labor, how should the nurse respond?

"It's all right for you to walk as long as you feel comfortable and your membranes are intact."

At 12 weeks' gestation a client with a history of frequent spontaneous abortions says to the nurse, "Every day I wonder whether I'll be able to have this baby." How should the nurse respond?

"It's understandable for you to be worried that you won't be able to carry this pregnancy to term. You've had a difficult time."

After an incomplete abortion, a client tells a nurse that although her health care provider explained what an incomplete abortion was, she did not understand. What is the best response by the nurse?

"It's when the fetus is expelled but other parts of the pregnancy remain in the uterus."

A client with mild preeclampsia is told that she must remain on bedrest at home. The client starts to cry and tells the nurse that she has two small children at home who need her. How should the nurse respond?

"Let's explore your available current support and opportunities for child care."

New parents are asked to sign the consent for their son to be circumcised. They ask for the nurse's opinion of the procedure. How should the nurse respond?

"Let's talk about it, because there are advantages and disadvantages."

During labor a client states that she does not want eyedrops or ointment placed in her baby's eyes immediately after birth. How should the nurse respond?

"Let's talk about why you don't want the medicine to be put into your baby's eyes."

After an uneventful 8-hour labor a client gives birth. Once the airway has been ensured and the neonate has been dried and wrapped in a blanket, the nurse places the newborn in the mother's arms. The mother asks, "Is my baby normal?" What is the best response by the nurse?

"Let's unwrap your baby so you can see for yourself."

A client who underwent mastectomy because of breast cancer is now undergoing chemotherapy, which has caused hair loss. The client states, "I feel like I've lost my sense of power." What is the nurse's best response?

"Losing power seems important to you."

A client on her first prenatal clinic visit is at 6 weeks' gestation. She asks how long she may continue to work and when she should plan to quit. How should the nurse respond?

"What activities does your job entail?"

While being admitted for a lumpectomy the client begins to cry and says, "I found the lump a few months ago, but I didn't go to the doctor because of what it could be." How should the nurse reply?

"This has been frightening for you."

A pregnant client has a positive group B Streptoccus (GBS) test at 36 weeks' gestation. What is the priority instruction that the nurse will include in the client's teaching plan?

"This information will be in your prenatal record, but please remind your labor and delivery nurse of this finding."

A client at 37 weeks' gestation gives birth to a healthy boy. While inspecting her newborn in the birthing room, the client becomes concerned and asks, "What's this sticky white stuff all over him?" How should the nurse respond?

"This is vernix. It helps protect the baby while he's in the uterus." -A factual response will allay the mother's concern. Vernix caseosa is a cheesy white substance that covers the fetus and confers protection from the amniotic fluid while the fetus is in utero. Most of it disappears by 40 weeks' gestation. Milia are white pinpoint dots (sebaceous glands) on the newborn's nose, chin, and forehead that disappear within a few weeks. The nurse should explain only what vernix is; referring to the infant as preterm may unnecessarily alarm the mother.

A client undergoes dilation and curettage (D&C) after an early miscarriage (spontaneous abortion). The nurse finds her crying later in the day. What is the most appropriate statement by the nurse?

"This must be a very hard experience for you to deal with."

During a follow-up appointment, a client at 21 weeks' gestation is found to have hyperemesis gravidarum. The client says, "Why is this happening to me? I don't know whether I can continue like this." What is the best response by the nurse?

"This must be physically and emotionally challenging for you."

A 37-year-old woman is admitted to the unit with severe menorrhagia. During a conversation, the nurse learns that the client has a history of fibroids, menorrhagia, pelvic pain, and depression. The client has been undergoing hormone therapy in hopes of easing the symptoms and reducing fibroids' size, without success. The lab reports hemoglobin and hematocrit readings of 6.8 and 20.2, respectively. The client begins to sob and cries, "I don't know what to do — the doctor is recommending a hysterectomy, but I haven't had children yet!" What is the best response by the nurse?

"This must be so difficult for you. Children are really important to you?"

After 5 years of unprotected intercourse, a childless couple comes to the fertility clinic. The husband tells the nurse that his parents have promised to make a down payment on a house for them if his wife gets pregnant this year. What is the nurse's best response to this comment?

"This must be very difficult for you with this added pressure."

A newborn with a severe bilateral cleft lip and palate is shown to the father first. The father says, "How could this happen to us? What's my wife going to do? It would've been better if she'd never gotten pregnant." How should the nurse respond?

"This must be very hard on you. I can go with you when your wife sees the baby."

A new mother exclaims to the nurse, "My baby looks like a Conehead!" How should the nurse respond?

"This often happens as the baby's head moves down the birth canal—the bones move for easier passage."

As the nurse helps a postpartum client change her perineal pad, the client comments, "I wish you didn't have to look at the pad it's embarrassing for me." What is the best nursing response?

"This seems to be uncomfortable for you, but I have to estimate the amount of blood loss to prevent any problems."

During a routine 32-week prenatal visit, a client tells the nurse that she has had difficulty sleeping on her back at night. What should the nurse advise the client about her position when she sleeps?

"Turn from side to side."

A client arrives in the birthing room with the fetal caput emerging. What should the nurse say to the client during a contraction?

"Use the panting-breathing pattern."

During a prenatal visit a client who is at 36 weeks' gestation states that she is having uncomfortable irregular contractions. How should the nurse respond?

"Walk around until they subside."

A nurse is giving discharge instructions to a new mother. What is the most important instruction to help prevent postpartum infection?

"Wash your hands before and after changing your sanitary napkins."

A client who is pregnant for the first time and is carrying twins is scheduled for a cesarean birth. What should the nurse tell the client to expect?

"We'll be encouraging you to walk early after surgery."

A nurse reviews the prescribed treatment with the parents of an infant born with bilateral clubfeet. Which parental statement indicates to the nurse that further education is required?

"We'll have to have the baby fitted with prosthetic devices before he'll be able to walk."

The clinic nurse is reviewing the dietary intake of a 16-year-old who is 12 weeks pregnant. What is the nurse's next action?

-Asking the client, "How many servings of dairy do you generally consume each day?" -Once the nurse knows the number of servings of calcium-rich foods the client consumes each day, the nutrition teaching plan can be personalized to her needs. The daily required intake of calcium for clients under 19 is 1300 mg. This amount can be obtained from 4 cups of milk or yogurt per day. Because there are sources of calcium other than milk, asking whether the client likes milk is not an appropriate first question. *Alternative sources of calcium include cheese, yogurt, figs, kale, sardines, orange juice with added calcium, creamy pesto, and cheese sauce.*

A nurse is to administer 5000 units of heparin to a prenatal client on prolonged bedrest. The label indicates that there are 20,000 units of heparin in each milliliter of solution. How much solution should the nurse give the client? Include a leading zero if applicable. Record your answer using two decimal places. ___ mL

0.25

After a client's membranes rupture spontaneously, the nurse sees the umbilical cord protruding from the vagina. Place the nursing interventions in order of priority.

1 Call for assistance and don sterile gloves 2 Insert two fingers into the vagina and exert upward pressure against the fetal presenting part 3 Put a rolled towel under one hip and place in the modified Sims position 4 Administer oxygen to the mother and monitor fetal heart tones

Four days after a vaginal hysterectomy a client calls the follow-up service and tells the nurse that she has a yellowish-green vaginal discharge. The nurse advises the client to return to the clinic for an evaluation. Which symptoms are suggestive of a vaginal infection? (Select all that apply.)

1 Abdominal pain 3 Rising temperature

A nurse is caring for a client with severe preeclampsia who is receiving magnesium sulfate. What side effects indicate that the serum magnesium level may be excessive? (Select all that apply.)

1 Absence of the knee-jerk reflex 5 Respiratory rate of 11 breaths/min

A nurse is teaching a prenatal class about the changes that occur during the second trimester of pregnancy. What cardiovascular changes should the nurse include? (Select all that apply.)

1 Cardiac output increases. 2 Blood pressure decreases. 3 The heart is displaced upward.

The nurse teaches a client about the increased need for vitamin A to meet the demands imposed by rapid fetal tissue growth during pregnancy. Which foods should the nurse encourage the client to ingest to meet this increased need? (Select all that apply.)

1 Carrots 4 Sweet potatoes

Which risk factors are associated with the future development of osteoporosis in women? (Select all that apply.)

1 Cigarette smoking 4 Familial predisposition 5 Inadequate intake of dietary calcium

A client at 43 weeks' gestation has just given birth to an infant with typical postmaturity characteristics. Which postmature signs does the nurse identify? (Select all that apply.)

1 Cracked and peeling skin 2 Long scalp hair and fingernails 5 Creases covering the neonate's full soles and palms

A woman is being seen in the prenatal clinic at 36 weeks' gestation. The nurse is reviewing signs and symptoms that should be reported to health care provider with the mother. Which signs and symptoms require further evaluation by the health care provider? (Select all that apply.)

1 Decreased urine output 5 Contractions that are regular and 5 minutes apart

A client undergoes anterior and posterior surgical repair of a cystocele and rectocele and returns from the postanesthesia care unit with an indwelling catheter in place. What should the nurse tell the client about the primary reasons for the catheter? (Select all that apply.)

1 Discomfort is minimized. 3 Retention of urine is prevented. 4 Pressure on the suture line is relieved.

List the mechanisms of labor in the correct sequence:

1 Engagement 2 Descent 3 Flexion 4 Internal rotation 5 Extension 6 Restitution 7 External rotation 8 Expulsion

A client asks the nurse about the use of an intrauterine device (IUD) for contraception. What information should the nurse include in the response? (Select all that apply.)

1 Expulsion of the device 2 Occasional dyspareunia 4 Risk for perforation of the uterus

A client at 7 weeks' gestation tells the nurse in the prenatal clinic that she has been bothered by episodes of nausea, but no vomiting, throughout the day. What should the nurse recommend? (Select all that apply.)

1 Focus on and repeat a rhythmic chant. 5 Eat small, frequent meals and eat dry crackers in between.

A client with preeclampsia is admitted to the labor and birthing suite. Her blood pressure is 130/90 and she has 2+ protein in her urine and edema of the hands and face. Which signs or symptoms are suggestive that HELLP syndrome is developing? (Select all that apply.)

1 Headache 3 Abdominal pain 5 Flulike symptoms

A client is admitted to the birthing suite with a blood pressure of 150/90 mm Hg, 3+ proteinuria, and edema of the hands and face. A diagnosis of severe preeclampsia is made. What other clinical findings support this diagnosis? (Select all that apply.)

1 Headache 3 Abdominal pain 5 Visual disturbances

On her first visit to the prenatal clinic a client with rheumatic heart disease asks the nurse whether she has special nutritional needs. What supplements in addition to the regular pregnancy diet and prenatal vitamin and minerals will she need? (Select all that apply.)

1 Iron 3 Folic acid

An almost term client reports that her fetus is moving less this week than last week. Which responses are appropriate? (Select all that apply.)

2 "It would be good for you to come to labor and delivery to be evaluated today." 3 "Always call the health care provider if you're worried that your baby isn't moving enough." 4 "Let me teach you how to conduct a kick count, and then you can call me when you've done one."

What findings occur with supine hypotensive syndrome? (Select all that apply.)

2 Feeling of faintness 4 Increased venous pressure 6 Decreased systolic pressure

A nurse is teaching a female client about using oral contraceptives and when to report complications. What clinical findings should alert the client to stop taking the contraceptive and notify the health care provider immediately? (Select all that apply.)

2 Headaches 4 Visual distrubances -Headaches, either sudden or persistent, may indicate hypertension or a cardiovascular event. Visual disorders, such as partial or complete loss of vision or double vision, may indicate neuro-ocular lesions, which are associated with the use of some oral contraceptives.

A nurse is caring for a client who is receiving internal radiation for cancer of the cervix. For which adverse reactions to the radiotherapy should the client be monitored? (Select all that apply.)

2 Hemorrhage 5 Increased temperature -Excessive sloughing of tissue may cause hemorrhage and is considered an adverse reaction. Infection, marked by an increase in temperature, may also develop from excessive sloughing of tissue. Nausea is an expected side effect of internal radiotherapy.

The nurse is obtaining a health history from a client with endometriosis. What consequences can occur as a result of this disorder? (Select all that apply.)

2 Metrorrhagia 4 Bowel strictures 5 Voiding difficulties -Metrorrhagia is a possible complication; bleeding between periods is due to the bleeding of endometrial tissue outside the uterus. The excessive tissue in endometriosis may impinge on the colon and cause ribbonlike stools. The endometrial tissue may impinge on the bladder and ureters and cause voiding difficulties. Menopause is a developmental adaptation. It does not complicate endometriosis, which is the presence of endometrial tissue outside the uterus that is prone to bleeding. Endometrial tissue does not enter the brain, so there are no cerebral complications.

A nurse teaches a pregnant woman about the need to increase her intake of complete proteins. Which foods identified by the client indicate that the teaching is effective? (Select all that apply.)

2 Milk 3 Eggs 6 Cheese -Milk contains animal proteins, which are complete proteins that contain all of the essential amino acids. Eggs contain animal proteins, which are complete proteins that contain all the essential amino acids. Cheese contains milk, which is a complete protein that contains of all the essential amino acids. Nuts are incomplete proteins. Bread is not a complete protein. Beans are not complete proteins unless eaten in a specific combination with soy products.

Methods of relieving back pain are explained during a childbirth class. What activities identified by the client permit the nurse to conclude that the teaching has been understood? (Select all that apply.)

2 Pelvic rocking 3 Forward tilting 4 Sacral pressure -Pelvic rocking eases tension in the muscles of the lumbar region. Lumbar pain during pregnancy results from the changes in posture as the uterus grows. Forward tilting eases tension in the muscles of the lumbar region. Lumbar pain during pregnancy results from the changes in posture as the uterus grows. Applying the heel of the hand to the laboring client's sacral area (counterpressure) helps relieve the back discomfort that may result when a fetus is in the occiput posterior position.

A client with a large fetus is to have a pudendal block during the second stage of labor. What does the nurse plan to instruct the client about the effectiveness of the block? (Select all that apply.)

2 Perineal pain will not be felt. 5 The bearing-down reflex will be diminished.

Women who become pregnant for the first time at a later reproductive age (35 years or older) are at risk for what complications? (Select all that apply.)

2 Preterm labor 3 Multiple gestation 4 Chromosomal anomalies 5 Bleeding in the first trimester

A nurse is caring for a client in active labor. What positions should the nurse encourage the client to assume to help promote comfort during back labor? (Select all that apply.)

2 Sitting 4 Lateral 5 Knee-chest

Which statements by a client with hyperemesis gravidarum would confirm that the client needs further teaching? (Select all that apply.)

3 "I'll start limiting my carbohydrates." 4 "I'll lie down for at least 2 hours after I eat."

A nurse is estimating a newborn's gestational age. What parameters should the nurse evaluate? (Select all that apply.)

3 Breast size 5 Genital development

A 2-day-old infant who weighs 6 lb (2722 g) is fed formula every 4 hours. Newborns need about 73 mL of fluid per pound of body weight each day. In light of this information, approximately how much formula should the infant receive at each feeding?

2 to 3 oz -Infants require about 73 mL of fluid per pound and 60 calories a day per pound for growth. The infant's weight of 6 lb × 73 mL of fluid = 438 mL. If fed every 4 hours the infant will have six feedings: 438 ÷ 6 = 73 mL; 73 ÷ 30 (30 mL/oz) = 2.4 oz. Therefore the infant should be offered 2 to 3 oz per feeding. One or 2 oz is inadequate for this newborn. Three to 5 oz is excessive for this newborn.

Although a client in labor is prepared and plans to participate in the labor and birth process, she states that she is in severe discomfort. The nurse administers the prescribed butorphanol (Stadol). Which phase of labor is the safest time for the nurse to administer this medication?

2. Active phase -Although a client in labor is prepared and plans to participate in the labor and birth process, she states that she is in severe discomfort. The nurse administers the prescribed butorphanol (Stadol). Which phase of labor is the safest time for the nurse to administer this medication?

A nurse is planning a childbirth education class about maternal psychological and physiological changes as pregnancy nears term. Which problems and concerns should the nurse include in the presentation? (Select all that apply.)

2. Nesting needs increase. 4. Anxiety and childbirth increases. 5. Gastrointestinal motility decreases.

A client arrives at the prenatal clinic and tells the nurse that she thinks that she is pregnant. The first day of the client's last menstrual period (LMP) was September 14, 2013. Using Naegele's Rule, what date in June 2014 is the client's estimated date of birth (EDB)? Record your answer as a whole number. ________

21

A client is found to have pelvic inflammatory disease, and the health care provider prescribes intravenous cefotetan 2 g twice a day. The instructions on the vial of cefotetan say to reconstitute 20 mL of diluent to yield 1 g/10.5 mL. How much solution should the nurse add to the 100-mL bag of 0.9% sodium chloride? Record your answer using a whole number. ___ mL

21 mL

A 30-year-old client with a 35-day menstrual cycle is trying to become pregnant. The nurse counsels the client and her partner about the optimal timing of intercourse during the cycle. The nurse determines that the counseling has been effective when the couple state that they should have intercourse on the:

21st day of the cycle -Ovulation usually occurs 14 days before menses; in a 35-day cycle, ovulation may occur as late as the 21st day. Day 12 day of the cycle is the proliferative phase of the cycle; ovulation has not yet occurred. If the woman had a 28-day cycle, ovulation is expected 14th day of the cycle. By the 25th day of the cycle, the ovum in this woman has passed out of the fallopian tube and can no longer be fertilized.

The school nurse is teaching a group of 16-year-old girls about the female reproductive system. One student asks how long after ovulation it is possible for conception to occur. The most accurate response by the nurse is based on the knowledge that an ovum is no longer viable after:

24 hours

A primigravida with type 1 diabetes is having her first prenatal visit. While discussing changes in insulin needs during pregnancy and after birth, the nurse explains that in light of the client's blood glucose readings she should expect to increase her insulin dosage. Between which weeks of gestation is this expected to occur?

24th and 28th weeks of gestation

A 20-year-old woman is known to be heterozygous for the cystic fibrosis (CF) gene. Her husband's genotype is unknown at present and the couple is expecting their first child. What should the nurse tell the couple about the probability of their baby's having CF?

25% or less -Males with cystic fibrosis are usually sterile; therefore the father does not have cystic fibrosis, but he could be a carrier. If both parents are heterozygous carriers, the chance of having a child with CF is 25%. When one parent is a heterozygous carrier and the other has two unaffected genes, the chance of having a child who has CF is 0% but the chance of having a child who is a carrier is 50%. If both parents are heterozygous carriers or if one parent is a heterozygous carrier and one parent has two unaffected genes, the chance of having a child that is a carrier, not a child that is affected, is 50%.

A client is to undergo amniocentesis at 38 weeks' gestation to determine fetal lung maturity. What lecithin/sphingomyelin ratio (L/S ratio) is adequate for the nurse to conclude that the fetus' lungs are mature enough to sustain extrauterine life?

2:1 -The lecithin concentration increases abruptly at 35 weeks, reaching a level that is twice the amount of sphingomyelin, which decreases concurrently. At 30 to 32 weeks' gestation, the amounts of lecithin and sphingomyelin are equal, indicating lung immaturity. A ratio of 1:4 does not reflect fetal lung maturity; nor does a ratio of 3:4.

The health care provider has prescribed 500 mg of cephalexin (Keflex) by mouth every 6 hours for 10 days for a client with mastitis. The health care provider has given the client 24 sample tablets of 250 mg apiece. How many days should this supply last? Record your answer using a whole number. _____ days

3

An estrogen-progestin oral contraceptive is prescribed for a client. Which adverse effects should the nurse teach the client to report to the health care provider? (Select all that apply.)

3 Chest Pain 5 Breast soreness 6 Calf tenderness -Estrogen-progestin contraceptives have been associated with breast malignancy; clinical manifestations include breast soreness, thickening, and lack of symmetry. These contraceptives have also been associated with thrombophlebitis; clinical manifestations of thrombophlebitis include calf tenderness and redness and heat over the affected area. If the clot travels, it could present as a pulmonary embolism, so chest pain should be reported. Lethargy, dizziness, and constipation are not expected side effects of this medication

A nurse suspects that a newborn's mother had rubella during the first trimester of pregnancy. Which newborn problems support this assumption? (Select all that apply.)

3 Deafness 5 Cardiac anomalies

What clinical findings does the nurse expect to observe in a newborn with trisomy 21 (Down syndrome)? (Select all that apply.)

3 Protruding tongue 4 Hypotonic muscle tone 6 Broad nose with a depressed bridge

A client who is at risk for seizures as a result of severe preeclampsia is receiving an IV infusion of magnesium sulfate. What findings cause the nurse to determine that the client is showing signs of magnesium sulfate toxicity? (Select all that apply.)

3 Respirations of 10/min 4 Loss of patellar reflexes

A client appears at the clinic after getting a positive result on a home pregnancy test. She states that her last menstrual period began 10 weeks ago. The client expresses fear because she has been recently found to have syphilis. What prescriptions will the nurse expect to receive from the health care provider because of this information? (Select all that apply.)

3 Screening and testing of sexual partners 5 Intramuscular benzathine penicillin G, 2.4 million units, one dose

A client has a diagnosis of an unruptured tubal pregnancy. Which findings correlate with this diagnosis? (Select all that apply.)

3 Unilateral abdominal pain 4 History of a sexually transmitted infection

A client who is taking an oral contraceptive calls the nurse with concerns about side effects of the medication. Which adverse effect of this medication should alert the nurse to inform the client to immediately stop the contraceptive and contact the health care provider? (Select all that apply.)

3. Visual disturbances 4. Persistent headaches

A neonate weighing 5 lb 6 oz (2438 g) is born in a cesarean birth and admitted to the newborn nursery. What range of resting respiratory rate should the nurse anticipate?

30 to 60 breaths/min

A nurse is caring for several pregnant clients in the prenatal clinic. Which client causes the most concern because of her predisposition to placenta previa?

30 years old, gravida 6, para 5

Which pregnant client does the nurse suspect is most likely to have placenta previa?

30 years old, gravida 6, para 5

A 24-year-old client who has been told that she is pregnant is at her first prenatal visit. She is 5 feet 6 inches tall and weighs 130 lb. What should the nutrition plan regarding her daily caloric intake include?

340 more calories during the second trimester

A client with stage 4 ovarian cancer is admitted for dehydration. The client is to receive an intravenous bolus of 500 mL D5W for 1 hour, after which the rate is to be changed to 150 mL/hr. The drop factor is 15 gtt/mL. At what rate, in drops per minute, should the nurse regulate the IV after delivery of the 500-mL bolus?

38

A neonate is born with exstrophy of the bladder, and the parents are upset. They are told that corrective surgery will be performed as soon as possible. How can the nurse best help the parents at this time?

Caring for the newborn in the same manner as any other newborn

What should a nurse include in the teaching plan for a couple seeking information about family planning?

A condom must be held in place by the rim when the penis is withdrawn from the vagina.

A nurse decides on a teaching plan for a new mother and her infant. What should the plan include?

A demonstration and explanation of infant care

The nurse is caring for a group of postpartum clients. Which one should the nurse monitor most closely?

A grand multipara who just had her sixth child -A grand multipara is a woman who has had at least 6 births . Multiparity contributes to an increased incidence of uterine atony because the uterine muscle may not contract effectively, leading to postpartum hemorrhage

A nurse prepares to administer vitamin K to a newborn. Why is vitamin K given specifically to newborns?

A newborn's intestinal tract does not synthesize it for several days after birth.

At 6 weeks' gestation a client is found to have gonorrhea. What medication does a nurse expect the health care provider to prescribe?

Ceftriaxone (Rocephin)

A client who is pregnant for the first time expels the products of conception at 12 weeks' gestation. The client's blood type is Rh negative. What should the nurse anticipate concerning the administration of Rho(D) immune globulin (RhoGAM)?

Administer RhoGAM within 72 hours of the miscarriage. -Rho(D) immune globulin (RhoGAM) should be given within 72 hours of a miscarriage or birth to have an effect on future pregnancies. RhoGAM is always indicated at the termination of a pregnancy, whether it is at term or before term and whether the fetus is alive or dead. It is not necessary to administer RhoGAM this early.

When entering the room of a client in active labor to answer the call light, the nurse sees that she ashen gray, dyspneic, and clutching her chest. What should the nurse do after pressing the emergency light in the client's room?

Administer oxygen by face mask

A newborn's total body response to noise or movement is often distressing to the parents. What should the nurse tell the parents this response represents?

A reflex that is expected in the healthy newborn

What should a nurse anticipate about the insulin requirements of a client with diabetes on her first postpartum day?

A sharp, sudden decrease -Insulin requirements may fall suddenly during the first 24 to 48 postpartum hours because the endocrine changes of pregnancy are reversed. Insulin requirements do not suddenly increase, remain unchanged, or decrease slowly and steadily at this time

A pregnant woman who is in the third trimester arrives in the emergency department with vaginal bleeding. She states that she snorted cocaine approximately 2 hours ago. Which complication does the nurse suspect as the cause of the bleeding?

Abruptio placentae -Abruptio placentae is associated with cocaine use; it occurs in the third trimester. Placenta previa is seen in the third trimester but is not associated with cocaine use. A tubal pregnancy is identified in the first trimester. Spontaneous abortion occurs in the first two trimesters.

A woman has just received the news that she is pregnant. She is ambivalent about the pregnancy because she had planned to go back to work when her youngest child started school next year. What developmental task of pregnancy must the woman accomplish in the first trimester of pregnancy?

Accept that she is pregnant

A nurse takes into consideration that the effect PKU has on the infant's development will depend on:

Adherence to a corrective diet instituted early

A client is scheduled for a modified radical mastectomy. What nursing intervention is most important in the client's preoperative plan of care?

Allowing her to express her feelings about surgery

The nurse teaches a high school sex education class that herpes genitalis infection cannot be cured but that the disease is marked by remissions and exacerbations. What else should the students be taught about this infection?

Although exacerbations occur they are not as severe as the initial episode.

A client who expected to use the Lamaze technique throughout labor has an emergency cesarean birth. Three days later the client is found crying and tells the nurse that she is extremely disappointed because a cesarean birth was necessary. She asks the nurse why this happened to her. On what factor should the nurse base a response?

An emergency cesarean birth is traumatic psychologically because of the loss of the expected birth experience.

Twenty-four hours after an uncomplicated labor and birth a client's complete blood count reveals a white blood cell (WBC) count of 17,000/mm3. How should the nurse interpret this WBC count?

An expected response to the process of labor and birth

After an abdominal hysterectomy the client returns to the unit with an indwelling catheter. The nurse notes that the urine in the client's collection bag has become increasingly sanguineous. What complication does the nurse suspect?

An incisional nick in the bladder

During a pelvic examination of a 24-year-old woman, the nurse suspects a vaginal infection because of the presence of a white curdlike vaginal discharge. What other assessment supports a fungal vaginal infection?

An itchy perineum -An itchy perineum usually occurs with candidiasis, a fungal infection; pruritus is the most common symptom

A client elects to have her pregnancy terminated after finding out at 16 weeks' gestation that she is carrying a fetus with Down syndrome. What should the nurse conclude about an abortion at this stage of the pregnancy?

An opportunity for the client to express feelings about her decision should be provided. -The client must feel comfortable enough to verbalize her feelings; this will help her complete the grieving process.

A husband sits in the waiting room while his wife is getting her infertility prescription refilled by the clinic pharmacist. As the nurse sits down beside him, he blurts, "It's like there are three of us in bed—my wife, me, and the doctor." What feeling is reflected by this statement?

Anger

A nurse is admitting a pregnant client who has mitral valve stenosis to the high-risk unit. What prophylactic medication does the nurse anticipate administering during the intrapartum period?

Antibiotic

After a client gives birth, what physiological occurrence indicates to the nurse that the placenta is beginning to separate from the uterus and is ready to be expelled?

Appearance of a sudden gush of blood

A client with a third-degree uterine prolapse is scheduled for a vaginoplasty. What should the nurse anticipate the surgeon will prescribe?

Apply moist compresses to the uterine prolapse. -Moist compresses may be prescribed to prevent ulcerations. Ambulation is contraindicated; it will predispose the client to the development of ulcerations. Elevating the foot of the bed is ineffective because gravity alone does not correct the prolapse. Supporting the prolapsed uterus with a sanitary pad may cause irritation and should be avoided.

At her first prenatal clinic visit a primigravida has blood drawn for a rubella antibody screening test, and the results are positive. What intervention is important when the nurse discusses this finding with the client?

Asking her whether she has had German measles and when she had the disease -The positive result indicates that the client has had rubella or was vaccinated. The nurse should determine whether she has had the disease, because it is important to know whether it was before or after she became pregnant; if she had rubella at the start of her pregnancy, the fetus is at risk. A rubella booster, either at the next visit or after the birth, is not necessary because the client has active immunity. More information is needed before the client can be told that no treatment will be needed.

On her second visit to the fertility clinic, a client whose temperature charts demonstrate an ovulatory pattern and a regular menstrual cycle requests fertility drugs. What is the nurse's best intervention?

Asking the client to have her partner's semen examined -Because the client is ovulating, the inability to conceive may be related to her partner's fertility.

After reading that nutrition during pregnancy is important for optimal growth and development of a baby, a pregnant woman asks the nurse what foods she should be eating. The nurse begins the teaching/learning process by:

Asking the client what she usually eats at each meal

While caring for a client who gave birth 1day ago, the nurse determines that the client's uterine fundus is firm at one fingerbreath below the umbilicus, blood pressure is 110/70 mm Hg, pulse is 72 beats/min, and respirations are 16 breaths/min. The client's perineal pad is saturated with lochia rubra. What is the priority nursing action?

Asking the client when she last changed the perineal pad -The amount of lochia would be excessive if the pad were saturated in 15 minutes; saturating the pad in 2 hours is considered heavy bleeding. If the pad has not been changed for a longer period, this could account for the large quantity of lochia. These findings cannot be supported without additional information.

A client who is to undergo dilation and curettage, and conization of the cervix for cancer appears tense and anxious. What is the best approach for the nurse to support the client emotionally?

Asking whether something is troubling the client and whether she'd like to talk about it

Which client care activity may a nurse safely delegate to a nursing assistant?

Assisting a client who is recovering from an abdominal hysterectomy to the bathroom

In the second hour after a client gives birth, her uterus is found to be firm, above the level of the umbilicus, and to the right of midline. What is the appropriate nursing intervention at this time?

Assisting the client to the bathroom to empty her bladder

The nurse is caring for a client who has a newborn with a neurological impairment. What is the most important nursing action?

Assisting the client with the grieving process

The nurse is teaching a client who is scheduling a vasectomy. What information is essential that the nurse explain to the client?

At least 15 ejaculations to clear the tract of sperm must occur before the semen is checked.

A nurse is obtaining the health history of a woman who is visiting the prenatal clinic for the first time. She states that she is 5 months pregnant. For what positive sign of pregnancy should the nurse look in this patient?

Audible fetal heartbeat

During the discharge conference with a client who has had a hysterectomy the nurse includes instructions for avoiding the thromboembolic phenomena that may occur as a complication. What should these instructions include?

Avoid sitting for long periods of time.

A client at 35 weeks' gestation is experiencing contractions. Her cervix is dilated 2 cm. The nurse teaches the client that sexual activity, particularly intercourse, should be:

Avoided to limit the onset of labor

A primigravida tells the nurse that she has morning sickness. What suggestion should the nurse make to help relieve the nausea?

Avoiding long periods without food

A client with severe preeclampsia is receiving an IV infusion of magnesium sulfate. The nurse remembers that magnesium sulfate is a:

Central nervous system depressant that blocks neuromuscular transmissions

During the discharge examination of a 2-day-old newborn, the nurse observes an edematous area confined to the right side of the scalp. How should the nurse document this condition?

Cephalhematoma

While observing a newborn with a diaphragmatic hernia, what does the nurse expect to identify?

Barrel-shaped chest

A nurse withholds methylergonovine maleate (Methergine) from a postpartum client. What clinical finding supports the withholding of the medication?

Blood pressure of 160/90 mm Hg -Methylergonovine maleate can cause hypertension and should not be given to a client with an increased blood pressure.

On the third postpartum day a mother visits the clinic and asks why her newborn's skin has begun to appear yellow. The nurse explains that the change in her infant's skin tone is the result of:

Breakdown of fetal red blood cells

A nurse is caring for a preterm neonate with physiological jaundice who requires phototherapy. What is the action of this therapy?

Breaks down the bilirubin into a conjugated form

A client who is visiting the family planning clinic is prescribed an oral contraceptive. As part of teaching, the nurse plans to inform the client of the possibility of:

Breakthrough bleeding -Breakthrough bleeding, or midcycle bleeding, commonly occurs when women start using oral contraceptives. If it persists, the dosage should be changed. There is no evidence that cervicitis, ovarian cysts, or fibrocystic disease is related to the use of oral contraceptives.

A nurse is reviewing a postmenopausal client's history, which reveals that the client previously received hormonal replacement therapy (HRT) as treatment for osteoporosis. For which problem does HRT increase the client's risk?

Breast cancer

A few weeks after discharge, a postpartum client experiences mastitis and telephones for advice concerning breastfeeding. The nurse notifies the practitioner to have antibiotics prescribed. What should the nurse recommend that the client do?

Breastfeed often to keep the breasts empty.

A client in active labor becomes very uncomfortable and asks a nurse for pain medication. Nalbuphine (Nubain) is prescribed. How does this medication relieve pain?

By acting on opioid receptors to reduce pain -Nalbuphine (Nubain) is classified as an opioid analgesic and is effective in relieving pain; it induces little or no newborn respiratory depression. Nalbuphine does not induce amnesia, act as an anesthetic, or induce sleep.

How should a nurse prepare to administer Rho(D) immune globulin (RhoGAM) to a postpartum client?

By ensuring that the client is Rh negative and the neonate is Rh positive

A client who had a cesarean birth is unable to void 3 hours after the removal of an indwelling catheter. How can the nurse evaluate whether the client's bladder is distended?

By palpating the client's suprapubic area gently

How should the nurse assess a newborn's grasp reflex?

By pressing the examining fingers against the palms of the newborn's hands

A 7-lb, 4-oz (3290-g) boy is admitted to the nursery and placed in a warm crib. The neonate begins to choke on mucus. How should the nurse suction him with a bulb syringe?

By suctioning the mouth before the nostrils

How does the nurse know that a client at 40 weeks' gestation is experiencing true labor?

Cervical dilation

The nurse visualizes and palpates a generalized, soft, edematous area of the scalp on the occiput of a newborn. What does the nurse suspect?

Caput succedaneum

A nurse is caring for a client with tertiary syphilis. Which body system should the nurse monitor most closely?

Cardiovascular

On reporting to the labor and delivery area a primipara indicates to the nurse that her contractions are occurring every 5 minutes. Upon further inquiry the nurse learns that the client has not attended any childbirth classes, and a cervical assessment reveals that she is in labor. When is the best time for the nurse to include education on simple breathing and relaxation techniques?

During the latent phase of the first stage of labor

The nurse discusses fetal weight gain with a pregnant client. When does it usually show a marked increase?

During the third trimester --During the third trimester the fetus is laying down fat deposits and gaining the most weight. Fetal weight gain occurs throughout pregnancy, but it is most marked in the third trimester. There is little fetal weight gain during the first trimester, when organ development is occurring.

A 26-year-old woman whose sister recently had a mastectomy calls the local women's health center for an appointment for a mammogram. What should the nurse tell the client when preparing her for the mammogram?

Each breast will be firmly compressed between two plates.

A nurse is preparing to counsel a client whose two previous pregnancies were uneventful, ending in term vaginal births of healthy children. What should the nurse consider about multiparas with previous uneventful pregnancies before beginning prenatal counseling?

Each pregnancy is a unique experience that is stressful despite multiparity.

During a male newborn's first encounter with his mother the nurse encourages her to undress him. The mother strokes him with her whole hand and while looking at him intently says, "He feels so velvety, and he is going to be just as good looking as his daddy." The baby is alert and responsive while gazing at his mother. What is the nurse's assessment of this first mother-infant encounter?

Early parenting behavior

A woman visits the clinic because she has dysmenorrhea. What goal should the nurse identify for this client?

Easing the pain of the client's menstruation.

A client had a fourth-degree perineal laceration during the birth of her neonate. What should the nurse recommend to protect the area from additional trauma?

Eat a high-fiber diet with increased fluid intake.

A client is receiving antibiotics and antifungal medications for the treatment of a recurring vaginal infection. What should the nurse encourage the client to do to compensate for the effect of these medications?

Eat yogurt daily

A client who has missed two menstrual periods arrives at the prenatal clinic with vaginal bleeding and one-sided lower quadrant pain. What condition does the nurse suspect?

Ectopic pregnancy -A tubal ectopic pregnancy causes first-trimester bleeding; unless an embryo and placenta happen to be located in the abdominal cavity, they cannot grow outside the uterus for longer than 10 to 12 weeks without causing the classic signs of pressure and bleeding.

The nurse should explain to the newly pregnant primigravida that the fetal heartbeat will first be heard with:

Electronic Doppler ultrasonography at 10 to 12 weeks

What should a nurse include in the discharge instructions for a woman who has undergone breast-conserving surgery (lumpectomy) for breast cancer?

Emphasizing the importance of breast self-examination -A client who has cancer of one breast is at risk for the development of cancer in the remaining breast; therefore breast self-examination is important.

A nurse plans to evaluate a postpartum client's uterine fundus for involution. What should the nurse ask the client to do before this assessment?

Empty her bladder.

A multigravida of Asian descent weighs 104 lb, having gained 14 pounds during the pregnancy. On her second postpartum day, the client's temperature is 99.2° F (37.3° C). She has had poor dietary intake since admission. What should the nurse do?

Encourage the family to bring in special foods preferred in their culture

A client has a cesarean birth. What is the most important nursing intervention to prevent thromboembolism on the client's first postpartum day?

Encouraging frequent ambulation

A nurse is preparing a pregnant client for an amniocentesis. What should nursing care include?

Encouraging her to void before the test

A client with a history of endometriosis has abdominal surgery to remove adhesions. What should this client's postoperative plan of care include?

Encouraging the client to ambulate in the hallway -Muscle contraction during ambulation improves venous return, which prevents venous stasis and thrombus formation.

A client who is 21 weeks pregnant loses the baby because of an incompetent cervix. Once the client's physical needs have been assessed and met, what is the best way for the nurse to meet the client's psychological needs?

Encouraging the client to see and hold the baby while still possible

A pregnant client arrives at the prenatal clinic, and the nurse obtains her obstetrical history. The client has two children at home, one born at 38 weeks' gestation and the second born at 34 weeks' gestation. She has also had one miscarriage, at 18 weeks, and an elective abortion. Using the GTPAL system, record the client's obstetrical record.

G5 T1 P1 A2 L2 -G (gravida ) stands for the total number of pregnancies a client has had. Gravida 5 indicates that this is the client's fifth pregnancy. T (term) stands for the number of neonates born at the expected date of birth. The neonate born at 38 weeks' gestation was born at term. P (preterm) stands for the number of neonates born before the expected date of birth. The neonate born at 34 weeks' gestation was born preterm. A (abortion or miscarriage) stands for the birth of a fetus before 20 weeks' gestation. Both the miscarriage and elective abortion are considered abortions. L (living) stands for the number of living children at the time of assessment. The client has two living children.

A pregnant client is making her first antepartum visit. She has a 2-year-old son born at 40 weeks, a 5-year-old daughter born at 38 weeks, and 7-year-old twin daughters born at 35 weeks. She had a spontaneous abortion 3 years ago at 10 weeks. How does the nurse, using the GTPAL format, document the client's obstetric history?

G5 T2 P1 A1 L4

Absence or weakness of which of the following reflexes during the newborn assessment should the nurse report to the health care provider?

Gag

A parent of a newborn asks, "Why do I have to scrub my baby's formula bottles?" What information about a newborn should the nurse consider before replying in language that the parent will understand?

Gastric acidity is low and does not provide bacteriostatic protection.

After being transported to the hospital by the ambulance, a pregnant woman is brought into the emergency department on a stretcher. The nurse notes that the fetus' head has emerged. How should the nurse assist the mother in the birth of the fetus' anterior shoulder?

Gently guiding the head downward -After the newborn's head has rotated externally, the nurse gently guides the head downward for the birth of the anterior shoulder. Gradually flexing the head toward the mother's thigh, gently putting pressure on the head by pulling upward, and gradually extending the head above the mother's symphysis pubis are all contraindicated.

A nurse is obtaining a health history from a primigravida on her first visit to the prenatal clinic. Before discussing the client's health habits with her, what does the nurse consider the most important factor in the survival of the client's newborn?

Gestational age and birthweight

A 15-year-old emancipated minor gave birth to a boy 36 hours ago and has requested a circumcision. What is the nurse's priority?

Getting an informed consent signed by the mother of the baby

The nurse is caring for a group of postpartum clients. Which factor puts a client at increased risk for postpartum hemorrhage?

Giving birth to a baby weighing 9 lb 8 oz

A nurse is caring for a group of postpartum clients. Which client is at the highest risk for disseminated intravascular coagulation (DIC)?

Gravida I who has had an intrauterine fetal death -Intrauterine fetal death is one of the risk factors for DIC; other risk factors include abruptio placentae, amniotic fluid embolism, sepsis, and liver disease. Multiple pregnancy, endometriosis, and high birthweight are not risk factors for DIC.

A client in labor, who is at term, is admitted to the birthing room. The fetus is in the left occiput posterior position. The client's membranes rupture spontaneously. What observation requires the nurse to notify the practitioner?

Greenish amniotic fluid

A primigravida is admitted to the birthing unit in active labor. The fetus is in a breech presentation. What physiological response does the nurse expect during this client's labor?

Greenish-tinged amniotic fluid -Greenish amniotic fluid is common in a breech presentation because the contracting uterus exerts pressure on the fetus' lower colon, forcing the expulsion of meconium.

What prenatal teaching is applicable for a client who is between 13 and 24 weeks' gestation?

Growth of the fetus, personal hygiene, and nutritional guidance

The four essential components of labor are powers, passageway, passenger, and psyche. Passageway refers to the bony pelvis. What type of pelvis is considered the most favorable for a vaginal delivery?

Gynecoid -A gynecoid pelvis is considered most favorable for a vaginal birth because the inlet allows the fetus room to pass. The gynecoid pelvis is considered the typical female pelvis. An android pelvis, which has a heart shape, is considered a male pelvis. The fetus often gets stuck. The anthropoid pelvis is elongated, with a roomy anterior posterior dimension and a narrower transverse diameter than the gynecoid pelvis. Although delivery is possible with this type of pelvis, it is less likely to be successful. The platypelloid pelvis is flat, with a compressed oval shape as the middle opening, instead of an open circle like the gynecoid pelvis. This is a rare type of pelvis.

The nurse is teaching a prenatal class to expectant mothers in their first trimester of pregnancy. In addition to discussing the need for 0.6 mg/day of folic acid replacement, which dietary choice that is high in folic acid should the nurse recommend?

Half cup of cooked spinach -A half cup of cooked spinach provides 100 mcg of folic acid per serving. One egg, a slice of bread, and half a cup of corn each provides only 20 mcg per serving.

A married 35-year-old client is to undergo a tubal ligation. The factor in the history that contributes most to the healthy resolution of any emotional problem associated with sterilization is that the client:

Has a son and daughter and feels that her family is complete

A nurse expects signs of respiratory distress syndrome (RDS) in a neonate whose mother:

Has type 1 diabetes -Infants of diabetic mothers are at risk for respiratory distress syndrome as a result of delayed synthesis of surfactant caused by a high serum level of insulin.

A neighbor who is a nurse is called on to assist with an emergency home birth. What should the nurse do to help expel the placenta?

Have the mother breastfeed the newborn

After a cesarean birth a nurse performs fundal checks every 15 minutes. The nurse determines that the fundus is soft and boggy. What is the priority nursing action at this time?

Massaging the client's fundus

A client at 10 weeks' gestation phones the prenatal clinic to report that she is experiencing some vaginal bleeding and abdominal cramping. The nurse arranges for her to go to the local hospital. The vaginal examination reveals that her cervix is dilated 2 cm. What diagnosis should the nurse expect?

Inevitable abortion -Once cervical dilation has begun, the abortion is classified as inevitable. In septic abortion the cervix is dilated and there is bleeding; also, the discharge is malodorous. Bleeding and cramping may be present, but the cervix is still closed in a threatened abortion. The products of conception have been partially expelled in an incomplete abortion.

A client's membranes ruptured 20 hours before admission. The client was in labor for 24 hours before giving birth. For which postpartum complication is she at risk?

Infection

A nurse who is admitting a newborn to the nursery observes a fetal scalp monitor site on the scalp. For what complication should the nurse monitor this newborn?

Infection

A client who is at 10 weeks' gestation returns for her second prenatal visit. She asks why she has to urinate so often. The nurse tells her that urinary frequency in the first trimester is:

Influenced by the enlarging uterus, which is still within the pelvis

A pregnant client with an infection tells the nurse that she has taken tetracycline (Tetracyn) for infections on other occasions and prefers to take it now. The nurse tells the client that tetracycline is avoided in the treatment of infections in pregnant women because it:

Influences the fetus's teeth buds -Tetracycline (Tetracyn) has an affinity for calcium; if used during tooth bud development it may cause discoloration of teeth. Tetracycline does not adversely affect breastfeeding, cause fetal allergies to the medication, or increase the fetus's tolerance of the medication.

A nurse is interviewing a female client with a tentative diagnosis of cystitis pending laboratory results. The nurse anticipates that the causative agent of the cystitis is Escherichia coli. The nurse anticipates this microorganism because it:

Inhabits the intestinal tract

Shortly after birth a newborn is found to have Erb's palsy. What condition does the nurse suspect caused this problem?

Injury to brachial plexus during birth

A 26-year-old primigravida experiencing severe abdominal pain is brought to the emergency department by ambulance with a suspected ruptured tubal pregnancy. What should the nurse do first?

Insert an intravenous catheter

A 16-year-old client has a steady boyfriend with whom she is having sexual relations. She asks the nurse how she can protect herself from contracting HIV. What should the nurse advise her to do?

Insist that her partner use a condom when having sex

After a mastectomy, a client returns from surgery with a closed suction drainage system in place and a dry sterile dressing covering the incision. What should the nurse do when observing this client for signs of bleeding?

Inspect the bedclothes under the client's axillary area for signs of drainage

The nurse is caring for a client in her third trimester who is to undergo amniocentesis. What should the nurse do to prepare the client for this test?

Instruct her to void immediately before the test

A client at term is admitted in active labor. She has tested positive for HIV. Which intervention in the standard orders should the nurse question as a risk to the fetus?

Internal fetal scalp electrode

A nurse is observing a newborn's respiratory rate. What clinical findings indicate that the rate is within the expected range?

Irregular, abdominal, 30 to 60/min -The expected breathing pattern is abdominal and irregular in rhythm and depth (alternating between shallow and deep); the expected rate ranges from 30 to 60 breaths/min.Newborns' respirations are irregular and abdominal.

At a client's first prenatal visit, the healthcare provider performs a pelvic examination, stating that the client's cervix is bluish purple, which is known as the Chadwick sign. The client becomes concerned and asks whether something is wrong. The best response is "This is expected; it:

Is caused by increased blood flow to the uterus during pregnancy" -Stating that the Chadwick sign is caused by increased blood flow to the uterus during pregnancy underscores the normalcy of Chadwick's sign and provides a simple explanation of the cause; women often need reassurance that the physical changes associated with pregnancy are expected. Stating that the Chadwick sign helps confirm pregnancy answers part of the question but fails to explain why it occurs. The Chadwick sign is a probable sign of pregnancy; it is not seen in nonpregnant women. There is no free blood circulating in the uterus during pregnancy.

A pregnant client with severe preeclampsia is receiving an infusion of magnesium sulfate. What does the nurse identify as the main reason that this medication is administered?

It acts as an anticonvulsant.

While showing a new mother how to care for her infant's umbilical cord stump, the nurse explains that the stump is a potential source of infection because:

It contains exposed tissue and blood

A client in the 38th week of gestation exhibits a slight increase in blood pressure. The health care provider advises her to remain in bed at home in a side-lying position. The client asks why this is important. What is the nurse's response regarding the advantage of this position?

It increases blood flow to the fetus.

A pregnant woman asks the nurse when she may expect her baby to be born. She tells the nurse that her last menstrual period began on April 14. According to Nägele's rule, what is the client's expected date of birth (EDB)?

January 21

A 23-year-old woman arrives at the prenatal clinic because she thinks that she is pregnant. Her last menstrual period began on March 31, and her pregnancy test reveals a positive result. According to Nägele's rule, what is this client's expected date of birth (EDB)?

January 7

What does the nurse do to elicit the Moro reflex during a newborn assessment?

Jars the infant's bassinet suddenly but gently -Sudden movement causes the startle response (Moro reflex), which begins with extension and abduction of the extremities with a C shape formed by the index finger and thumb, followed by flexion and adduction of extremities and ending with return of the arms to a relaxed position

A nurse is assessing a newborn for signs of hyperbilirubinemia (pathological jaundice). What clinical finding confirms this complication?

Jaundice that develops in the first 12 to 24 hours

How does the nurse provide kangaroo care to a preterm infant?

Keep the newborn in skin-to-skin contact with the parent.

What nursing care is most important for a newborn with respiratory distress syndrome (RDS)?

Keeping the infant in a warm environment

What nursing action best promotes parent-infant attachment behaviors?

Keeping the new family together immediately after the birth -Research strongly supports the theory that there is a sensitive period during the first few hours of life that is important for the promotion of parent-infant attachment. Parent-infant bonding can take place with or without visitors. Encouraging rooming-in is helpful because it increases the amount of contact between the parents and the newborn, but it is not as significant as those critical first few hours after the birth.

A newborn who is born at 36 weeks' gestation weighs 8 lb 13 oz (3997 g). How should the nurse document this finding?

LGA and preterm

A nurse performs Leopold's maneuvers on a newly admitted client in labor. Palpation reveals a soft, firm mass in the fundus; a firm, smooth mass on the mother's left side; several knobs and protrusions on the mother's right side; and a hard, round, movable mass in the pubic area with the brow on the right. On the basis of these findings, the nurse determines that the fetal position is:

LOA

A client is scheduled to have a contraction stress test (CST) to determine fetal well-being. Which type of fetal heart rate (FHR) decelerations constitutes a nonreassuring outcome?

Late -The fetus with a borderline cardiac reserve will show hypoxia, evidenced by a decreased FHR with minimal stress, making the test positive. Early decelerations, a response to head compression, are benign. "Baseline" is not used to describe the baseline measurement; the baseline rate is determined before the test or early in the test to provide a basis for comparison, not to indicate fetal compromise. Variable decelerations are nonuniform drops in FHR before, during, or after a contraction; these are related to partial, brief cord compression that can be eliminated with a change in the mother's position.

What factor identified by the nurse in a client's history places the client at increased risk for breast cancer?

Late beginning of childbearing

A nurse concludes that a positive contraction stress test (CST) result may be indicative of potential fetal compromise. A CST result is considered positive when during contractions the fetal heart rate shows:

Late decelerations

A client in preterm labor is to receive a tocolytic medication, and bedrest is prescribed. Which position should the nurse suggest that the client maintain while on bedrest?

Lateral -The lateral position relieves pressure on the vena cava, thereby promoting venous return and increasing placental perfusion. The supine position promotes hypotension because the pressure of the gravid uterus on the vena cava interferes with the return of blood from the lower extremities. The Fowler position promotes hypotension because the pressure of the gravid uterus on the vena cava interferes with the return of blood from the lower extremities. The semi-Fowler position promotes hypotension because the pressure of the gravid uterus on the vena cava interferes with the return of blood from the lower extremities.

A pregnant client at 37 weeks' gestation is taught about signs and symptoms that should be reported immediately to the primary care provider. The nurse determines that the client understands the information presented when she states that she will immediately report:

Leakage of fluid from the vagina

What should the nurse recommend to a new mother when teaching her about the care of the umbilical cord area?

Leave the area untouched or clean with soap and water, then pat it dry.

A client with cervical cancer is to undergo a course of internal radiation. The client returns to her lead-lined room on the oncology unit with an indwelling urinary catheter and a vaginal applicator in place. Once the practitioner has loaded the applicator with the radiation source, the nurse's plan of care should include:

Leaving the urinary catheter undisturbed

A client is admitted in active labor. The nurse, performing Leopold maneuvers, determines that the fetus is in the left occiput anterior (LOA) position. Where should the nurse place the transducer of the electronic fetal monitor?

Left lower quadrant

A client is admitted to the high-risk prenatal unit with the diagnosis of placenta previa. What should the nurse instruct the client to do?

Lie on her side to avoid putting pressure on the vena cava -The side-lying position decreases pressure on the vena cava from the gravid uterus, ensuring adequate oxygenation of the fetus. Without proper positioning, breathing techniques will be less effective.

A pregnant client with a history of preterm labor is at home on bedrest. What instructions should a teaching plan for this client include?

Lie on the side with the head raised on a small pillow.

A nurse is caring for a client in labor whose cervix is dilated 6 cm. The client is receiving epidural analgesia. What common response to regional anesthesia does the nurse anticipate?

Lightheadedness

A client who had tocolytic therapy for preterm labor is being discharged. What instructions should the nurse include in the teaching plan?

Limit daily activities.

What nursing care is required for a client with a radium implant for cancer of the cervix?

Limiting the client's activity to avoid dislodging the radium insert

During the first hour after a cesarean birth, a nurse notes that the client's lochia has saturated one perineal pad. In light of the knowledge of expected lochial flow, what should the nurse conclude that this indicates?

Lochial flow within expected limits

A client starting her second trimester asks a nurse in the prenatal clinic whether she can safely take an over-the-counter (OTC) medicine now that she is past the first 3 months of pregnancy. The nurse explains why she should consult with her health care provider before taking any oral medications. What physiological alteration associated with pregnancy may change the client's response to medication?

Longer gastrointestinal emptying time -Gastrointestinal motility is reduced during pregnancy because of the high level of placental progesterone and displacement of the stomach superiorly and the intestines laterally and posteriorly; absorption of some drugs, vitamins, and minerals may be increased because of their slow passage through the gastrointestinal tract. The glomerular filtration rate increases during pregnancy. The amount of gastric secretion is somewhat lower in the first and second trimesters; it increases in the third trimester. The development of fetal-placental circulation is unrelated to the absorption of drugs.

A 28-year-old woman is scheduled to undergo a laparoscopic bilateral salpingo-oophorectomy. What does a nurse expect to be the client's priority concern?

Loss of childbearing potential

A 30-year-old woman is to undergo total abdominal hysterectomy for noninvasive endometrial cancer. The nurse anticipates the client may have difficulty adjusting emotionally to this type of surgery. What is the most common reason for this difficulty?

Loss of femininity

A nurse is teaching a prenatal class about smoking during pregnancy. What neonatal consequence of maternal smoking should the nurse include in the teaching?

Low birth weight

A client at 22 weeks' gestation asks the nurse how to prevent back pain as her pregnancy progresses. What does the nurse suggest that she wear?

Low-heeled shoes -Low-heeled supportive shoes help maintain the body's center of gravity over the hips, limiting arching of the back that compensates for the increased weight in the abdominal area.

The practitioner diagnoses placenta previa. What does this indicate to the nurse about the condition of the placenta?

Low-lying

A nurse who is caring for a 32-week appropriate-for-gestational-age (AGA) neonate establishes a list of objectives for the infant. What objective should receive the highest priority?

Maintain respirations

What is an important nursing intervention when a client is receiving intravenous (IV) magnesium sulfate for preeclampsia?

Maintaining a quiet, darkened environment

A client at 39 weeks' gestation arrives in the birthing suite reporting that she is having regular contractions. A vaginal examination reveals that the presentation is a double-footling breech. The practitioner decides to proceed to a cesarean birth under regional anesthesia. What is an important intervention to help prevent postoperative maternal complications?

Maintaining adequate hydration

A nurse is caring for a client with vaginal bleeding caused by placenta previa. What is the best nursing intervention to delay the birth of the fetus?

Maintaining bed rest

A client comes to the fertility clinic for hysterosalpingography using radiopaque contrast material to determine whether her fallopian tubes are patent. When preparing for the test, the nurse explains to the client that she:

May have some persistent shoulder pain for 14 hours after the test -The nurse is describing referred pain from passage of the contrast medium through the tubes; it is usually indicative of tubal patency. An anesthetic is not given; the client's complaint of pain can be managed with a position change and mild analgesics. The client may resume usual activities as soon as the test is over. The client usually does not experience nausea or vomiting.

A nurse helps a client to the bathroom to void several times during the first stage of labor. This is done because a full bladder:

May inhibit the progress of labor

What does the nursing care for an infant with necrotizing enterocolitis (NEC) include?

Measuring abdominal girth every 2 hours

The day after a hysterectomy, the client asks for sanitary pads because she feels that she is going to menstruate. What information should influence the nurse's response?

Menstruation will not occur because the uterus has been removed. -Menstruation is shedding of the endometrial lining of the uterus. A woman who has undergone a hysterectomy has had her uterus removed and will no longer menstruate. Abdominal pain is common after a hysterectomy, but menstruation is impossible after the surgery; additional symptoms are necessary before any conclusion can be made. Frank bleeding is not expected after a hysterectomy.

A multipara is admitted to the birthing room in active labor. Her temperature is 98° F (36.7° C), pulse 70 beats/min, respirations 18 breaths/min, and blood pressure 126/76 mm Hg. A vaginal examination reveals a cervix that is 90% effaced and 7 cm dilated with the vertex presenting at 2+ station. The client is complaining of pain and asks for medication. Which medication should be avoided because it may cause respiratory depression in the newborn?

Meperidine (Demerol) -Meperidine (Demerol) is an opioid that can cause respiratory depression in the neonate if administered less than 4 hours before birth.

A nursing instructor provides education for the students on thermoregulation in the nursery. The students determine that in the healthy full-term neonate, heat production is accomplished by:

Metabolism of brown fat

A nurse is caring for a client who has contracted a trichomonal infection. Which oral drug should the nurse anticipate that the health care provider will most likely prescribe?

Metronidazole (Flagyl) -Metronidazole (Flagyl) is a potent amebicide. It is effective in eradicating the protozoan Trichomonas vaginalis. Penicillin is administered for its effect on bacterial, not protozoal, infections. Gentian violet is a local anti-infective that is applied topically; it may cause discoloration of the skin. It is effective against Candida albicans. Nystatin (Mycostatin) is an antifungal for infections caused by C. albicans.

A nurse is assigned an adolescent who gave birth 12 hours ago. She continually talks on the phone to her friends and does not respond when her new baby cries. What is the best immediate intervention?

Modeling appropriate behaviors that encourage infant bonding

What type of lochia should the nurse expect to observe on a client's pad on the fourth day after a vaginal delivery?

Moderate serosa

The nurse observes several dark round areas on a newborn's buttocks on a dark-skinned neonate. How should this observation be documented?

Mongolian spots -Mongolian spots are bluish-black areas of pigmentation commonly found on the back and buttocks of dark-skinned newborns; they are benign and fade gradually over time.

A client who is in the first trimester is being discharged after a week of hospitalization for hyperemesis gravidarum. She is to be maintained at home with rehydration infusion therapy. What is the priority nursing activity for the home health nurse?

Monitoring the client for signs of electrolyte imbalances

A client at 6 weeks' gestation who has type 1 diabetes is attending the prenatal clinic for the first time. The nurse explains that during the first trimester insulin requirements may decrease because:

Morning sickness may lead to decreased food intake.

The parents of a newborn with phenylketonuria (PKU) ask a nurse how to prevent future problems. What must the nurse consider before responding?

Most important is the institution of a corrective formula soon after birth.

A nurse understands the stages of parental adjustment that follow the birth of an at-risk infant who is in the neonatal intensive care unit. To better plan nursing care, the nurse bases observations and assessments on the recognition that the:

Mother should be reunited with her infant as soon as possible to enhance adjustment

What must the LPN observe first when planning to promote mother-infant attachment?

Mother-infant interaction

A nurse who is caring for a postpartum client is concerned because the woman is at risk for hemorrhage. Which factor in the client's history alerted the nurse to this concern?

Multifetal pregnancy

The nurse is caring for four clients on the postpartum unit. Which client will most likely state that she is having difficulty sleeping because of afterbirth pains?

Multipara who has vaginally delivered three children -A multipara's uterus tends to contract and relax spasmodically, even if uterine tone is effective, resulting in pain that may require an analgesic for relief.

When reviewing the history of a client admitted in preterm labor during her 30th week of gestation, the nurse suspects a risk factor associated with this client's preterm labor. What is this risk factor?

Multiple urinary tract infections

When calculating an Apgar score for a newborn, what is given a score in addition to the heart rate?

Muscle tone

A client in labor begins to experience contractions 2 to 3 minutes apart and lasting about 45 seconds. Between contractions the nurse identifies a fetal heart rate (FHR) of 100 beats/min on the internal fetal monitor. What is the next nursing action?

Notifying the health care provider -Bradycardia (baseline FHR slower than 110 beats/min) indicates that the fetus may be compromised, requiring medical intervention. Resuming continuous fetal heart monitoring may be dangerous; the fetus may be compromised, and time should not be spent on monitoring. Continuing to monitor the maternal vital signs is not the priority at this time. The expected FHR is 110 to 160 beats/min between contractions.

A 36-year-old primagravida is receiving treatment for preeclampsia at 29 weeks' gestation. In light of the latest information on the client's record, what does the nurse identify as the priority of care?

Notifying the primary health care provider about the epigastric pain, headache, and blurred vision

A female client is scheduled for a hysterectomy. While discussing the preoperative preparations, the nurse determines that the client's understanding of the surgery is inadequate. What is the next nursing intervention?

Notifying the surgeon that the client needs more information -Legally the person performing the surgery is responsible for informing the client adequately; the nurse may clarify information, witness the client's signature, and co-sign the consent form. Describing the proposed surgery to the client is beyond the scope of nursing practice. The nurse could face criminal charges of assault and battery for proceeding when there is a lack of informed consent . Explaining gently that she should have asked more questions places blame on the client; it is the responsibility of the surgeon to impart the vital information required for consent.

A nurse in the newborn nursery is monitoring an infant for jaundice related to ABO incompatibility. What blood type does the mother usually have to cause this incompatibility?

O

A client who has breast cancer had postlumpectomy chemotherapy and is now scheduled for radiation on an outpatient basis. What is an important nursing intervention while the client is receiving radiation?

Observing the irradiated site daily for redness or irritation

A client at 24 weeks' gestation arrives at the clinic for a routine examination. She tells the nurse, "I feel puffy all over." In light of this statement, what is most important?

Obtaining her blood pressure

A client at 40 weeks' gestation is admitted to the birthing unit in early active labor. She tells the nurse that her membranes ruptured 26 hours ago. Assessments of the fetal heart rate range between 168 and 174 beats/min. What is the priority nursing action?

Obtaining maternal vital signs

During a routine prenatal office visit at 26 weeks' gestation, a client states that she is getting fat all over and that she even needed to buy bigger shoes. What is the next nursing action?

Obtaining the client's weight and blood pressure

A nurse assesses the process of involution by measuring the location of the client's fundus during the postpartum period. Mark the location the fundus is expected to be 1 day after birth in a client whose bladder is not distended.

One day after birth, the fundus is expected to be at the level of the umbilicus. In the first 12 hours after birth the uterus is expected to be one fingerbreath above the umbilicus. It is then expected to descend by approximately one fingerbreath per day until it descends under the pubic bone, usually around day 10.

A 38-year-old client attends the prenatal clinic for the first time. A nurse explains that several tests will be performed, one of which is the serum alpha-fetoprotein test. The client asks what the test will reveal. What should the nurse include in the reply?

Open neural tube defects

A nurse instills an antibiotic ophthalmic ointment into a newborn's eyes. What condition does this medication prevent?

Ophthalmia neonatorum

A woman in labor arrives at the birthing unit. She tells the nurse, "They told me the last time I was at the clinic that I had chlamydia, but I stopped taking the antibiotic after 3 days because I felt better." What potential neonatal disorder transmitted during birth most concerns the nurse because of the inadequate treatment?

Ophthalmia neonatorum

A nurse is planning to use a newborn's foot to obtain blood for the required newborn metabolic testing. What part of the foot is the best site to use for the puncture?

Outer heel

A sonogram performed on a client in the third trimester demonstrates a low-lying placenta. The nurse should teach the client that she is at risk for:

Painless vaginal bleeding

A nurse concludes that a laboring couple has benefited from the Lamaze method of childbirth preparation when during the transition phase of labor they use the breathing pattern known as:

Pant-blow -Panting and blowing keeps the glottis open so the mother cannot hold her breath and bear down.

A nurse is preparing a client with a ruptured tubal pregnancy for immediate surgery. What type of surgery should the informed consent include?

Salpingectomy

While mopping the kitchen floor, a client at 37 weeks' gestation experiences a sudden sharp pain in her abdomen with a period of fetal hyperactivity. When the client arrives at the prenatal clinic, the nurse examines her and detects fundal tenderness and a small amount of dark-red bleeding. What does the nurse conclude is the probable cause of these clinical manifestations?

Partial abruptio placentae -Typical manifestations of abruptio placentae are sudden sharp localized pain and small amounts of dark-red bleeding caused by some degree of placental separation. True labor begins with regular contractions, not sharp localized pain. There is no pain with placenta previa, just the presence of bright-red bleeding. There are no data to indicate that the client sustained an injury.

A mother is concerned that her newborn will be exposed to communicable diseases when she goes home. While teaching the mother ways to decrease the risk of Infection; what type of immunity should the nurse explain was transferred to her baby through the placenta?

Passive natural -Passive natural immunity is developed from an antigen-antibody response in the mother that is transmitted to the fetus. Active natural immunity is acquired by an individual in response to a disease or an infection. Active artificial immunity is acquired by an individual in response to small amounts of antigenic material (e.g., vaccination). Passive artificial immunity is conferred by the injection of antibodies prepared in another host.

A nurse is caring for several new mothers in the birthing unit, all in the taking-in phase of the postpartum period. What information is most appropriate for these clients at this time?

Perineal care -During the taking-in phase a woman is primarily concerned with being cared for and being cared about. Infant feeding is best taught during the taking-hold phase of postpartum adjustment. Infant hygiene is best taught during the taking-hold phase of postpartum adjustment. Family planning is not a primary concern during the immediate postpartum period.

A client who is having a difficult labor is found to have cephalopelvic disproportion. Which medical order should the nurse question?

Piggyback another 10-unit bag of oxytocin (Pitocin).

A client with frank vaginal bleeding is admitted to the birthing unit at 30 weeks' gestation. The admission data include blood pressure of 110/70 mm Hg, pulse of 90 beats/min, respiratory rate of 22 breaths/min, fetal heart rate of 132 beats/min. The uterus is nontender, the client is reporting no contractions, and the membranes are intact. In light of this information, what problem does the nurse suspect?

Placenta previa -A nontender uterus and bright-red bleeding are classic signs of placenta previa ; as the cervix dilates, the overlying placenta separates from the uterus and begins to bleed.

A pregnant client asks a nurse for information about toxoplasmosis during pregnancy. What should the nurse teach the client about how to prevent the transmission of toxoplasmosis?

Pork and beef should be cooked well before being eaten.

During labor the nurse encourages the client to void periodically. The nurse knows that an over distended urinary bladder during labor can:

Predispose the client to uterine hemorrhage after birth

A pregnant client with type 1 diabetes is visiting the prenatal clinic for the first time. What is the primary long-term goal for this client?

Pregnancy will end with the birth of a healthy infant.

A multipara whose membranes have ruptured is admitted in early labor. Assessment reveals a breech presentation, cervical dilation of 3 cm, and fetal station at -2. For what complication should the nurse assess when caring for this client?

Prolapse of the umbilical cord

The nurse administers the prescribed vitamin K intramuscularly to a newborn immediately after birth to:

Promote the synthesis of prothrombin

A client who is in preterm labor at 34 weeks' gestation is receiving intravenous tocolytic therapy. The frequency of her contractions increases to every 10 minutes, and her cervix dilates to 4 cm. The infusion is discontinued. Toward what outcome should the priority nursing care be directed at this time?

Promotion of maternal and fetal well-being during labor

A nurse is teaching a class of expectant parents about nutritional needs during pregnancy. What information should the nurse include?

Protein needs increase to at least 70 g/day during pregnancy.

A client at 36 weeks' gestation arrives at the prenatal clinic for a routine examination. The nurse determines that the client's blood pressure has increased from 102/60 to 134/88 mm Hg and becomes concerned she may be experiencing mild preeclampsia. What other sign of mild preeclampsia does the nurse anticipate?

Proteinuria of 1+

While auscultating the lungs of a client admitted with severe preeclampsia, the nurse identifies crackles. What inference does the nurse make when considering the presence of crackles in the lungs?

Pulmonary edema has developed.

A nurse is evaluating a client's understanding regarding postoperative concerns after mastectomy. Which development near and around the incision noted by the client should be reported to her practitioner?

Swelling with erythema

When performing Leopold maneuvers on a client who has been admitted to the birthing room, the nurse identifies a firm, round prominence over the symphysis pubis; a smooth, convex structure along her right side; irregular lumps along her left side; and a soft roundness in the fundus. What is the fetal position?

ROA -The fetus is in a right occiput anterior (ROA) position ; the prominence over the symphysis suggests a vertex and the fetal occiput and back are in the right anterior quadrant. Left occiput posterior (LOP) is ruled out because the occiput is not located in the left posterior quadrant; the occiput and back are on the mother's right side. Right sacral anterior (RAS) is ruled out because the fetus is in a vertex, not a breech, presentation. Left occipital anterior (LOA) is ruled out by the presence of irregular lumps on the left side, suggesting that the fetus's back is in the mother's right quadrant.

Examination of a client in active labor reveals fetal heart sounds in the right lower quadrant. The head is in the anterior position, is well flexed, and is at the level of the ischial spines. What fetal position should the nurse document?

ROA, 0 station

While caring for a client in labor, a nurse notes that during a contraction there is a 15-beat/min acceleration of the fetal heart rate above the baseline. What is the nurse's next action?

Record the fetal response to contractions and continue to monitor the heart rate

A nurse is counseling a pregnant client who is a vegetarian. What should the nurse plan to do to ensure optimal nutrition during the pregnancy?

Refer the client to a dietitian to help plan her daily menu

A client is scheduled for amniocentesis. What should the nurse do before the procedure?

Remind the client to empty her bladder

During the second reactive period a newborn becomes more alert and responsive and there is an increase in mucus production and gagging. What should the nurse do first?

Remove secretions from the pharynx

A client who recently gave birth is transferred to the postpartum unit by the nurse. What must the nurse do first to avoid a charge of abandonment?

Report the client's condition to the responsible staff member

A nurse is counseling a pregnant woman with type 1 diabetes. What is the most important nursing consideration in the planning of care for this client?

Requirement of intensive prenatal care

A nurse administers the prescribed intravenous dose of magnesium sulfate to a client with severe preeclampsia. What adverse effect should the nurse address when evaluating the client's response to the medication?

Respiratory depression

In her 36th week of gestation, a client with type 1 diabetes has a 9-lb 10-oz infant in a cesarean birth. For which condition should the nurse monitor this infant of a diabetic mother?

Respiratory distress syndrome

While a client at 30 weeks' gestation is being examined in the prenatal clinic, the nurse identifies a respiratory rate of 26/min, blood pressure of 100/60, and diaphragmatic tenderness, and the client reports increased urinary output. Which finding indicates that the client may be experiencing a complication?

Respiratory rate

A nurse discusses herpes genitalis as part of a high school sex education program. The nurse explains to the students that herpes genitalis is:

Responsible for local as well as systemic reactions

A 37-year-old client with a nontender palpable breast mass has a questionable mammogram. She is undergoing further diagnostic tests to determine whether the mass is malignant. What information should the nurse take into consideration before planning health teaching for this client?

Results of a biopsy are necessary before a specific form of therapy is selected.

Rho(D) immune globulin (RhoGAM) is prescribed for an Rh-negative client who has just given birth. Before giving the medication, the nurse verifies the newborn's Rh factor and reaction to the Coombs test. Which combination of newborn Rh factor and Coombs test result confirms the need to give Rho(D) immune globulin ?

Rh positive with a negative Coombs result

Before the administration of Rho(D) immune globulin (RhoGAM) the nurse reviews the laboratory data of a pregnant client. Which blood type and Coombs test result must a pregnant woman have to receive RhoGAM after giving birth?

Rh-negative and Coombs negative -Rho(D) immune globulin (RhoGAM) is given to an Rh-negative mother after birth if the infant is Rh positive and the Coombs test reveals that the mother was not previously sensitized (negative). An Rh-positive mother will not develop antibodies to a fetus who is either Rh positive or Rh negative; therefore the Coombs test is not performed.

A newborn has congenital cataracts, microcephaly, deafness, and cardiac anomalies. Which infection does the nurse suspect that the newborn's mother contracted during her pregnancy?

Rubella -Congenital rubella (German measles) syndrome results in abnormalities that vary, depending on the gestational age of the fetus when the maternal infection was contracted; the most severe results occur if the mother was infected during the first trimester, when organogenesis is taking place

A nurse is teaching a primigravida about how she can identify the onset of labor. What clinical indicator of labor would necessitate the client to call her health care provider?

Rupture of membranes or contractions 5 minutes apart

What should the nurse tell a new mother will be delayed until her newborn is 36 to 48 hours old?

Screening for phenylketonuria

A nurse is caring for a client who has severe preeclampsia. For which characteristic of eclampsia should the nurse monitor the client?

Seizures

A 36-year-old client undergoes a modified radical mastectomy. The nurse determines that the client understands the schedule for self-examination of her remaining breast when she states that she will examine her breast:

Seven days after each menstrual period

A 16-year-old primigravida at 32 weeks' gestation is admitted to the high-risk unit. Her blood pressure is 170/110 mm Hg and she has 4+ proteinuria. She gained 50 lb during the pregnancy, and her face and extremities are edematous. What complication, which occurs in the latter part of pregnancy, does the nurse identify?

Severe preeclampsia

A client being prepared for surgery because of a ruptured tubal pregnancy complains of feeling lightheaded. Her pulse is rapid, and her color is pale. What condition does the nurse anticipate as a common complication of a ruptured tubal pregnancy?

Shock

What does the nurse teach a client to do when performing breast self-examination?

Squeeze the nipples to examine for discharge

The transmission of which microorganism that causes maternal mastitis is minimized by frequent handwashing by nursing staff members?

Staphylococcus aureus

A woman comes into the clinic and states that she is thinking about becoming pregnant. What can the woman do to improve the health of her baby before she becomes pregnant?

Start taking prenatal vitamins -Folic acid is important for the pregnant woman; a lack of folic acid can result in neural tube defects, including spina bifida. The time during fetal development when this occurs is very early in the pregnancy, when the woman may not even realize that she is pregnant. Taking prenatal vitamins with adequate folic acid can greatly reduce this birth defect.

The nurse is caring for a client whose labor is to be induced. What is the nurse's responsibility when a client's labor is being stimulated with an oxytocin (Pitocin) infusion?

Stopping the infusion if contractions become hypertonic

A nurse observes a laboring client's amniotic fluid and decides that it is the expected color. What finding supports this conclusion?

Straw-colored, clear, and containing little white specks

What is the most appropriate way for the nurse to elicit the Moro reflex in an infant?

Striking the surface of the infant's crib

A client states that she wishes to use the calendar method of birth control. The nurse concludes that the client understands how to calculate the beginning of the fertile period when she states, "I will:

Subtract 18 days from the length of my shortest cycle" -The fertile period is determined by subtracting 18 days from the length of the shortest cycle to determine the first unsafe day and subtracting 11 days from the length of the longest cycle to determine the last unsafe day.

During a vertex vaginal birth the nurse notes meconium-stained amniotic fluid. What is the priority nursing intervention for the newborn?

Suctioning the airway

At 10 hours of age a newborn has a large amount of mucus in the nasopharynx and becomes cyanotic. What is the nurse's initial action?

Suctioning the mouth

What clinical manifestation requires immediate intervention in a woman with a probable ruptured tubal pregnancy?

Sudden onset of knifelike pain in one of the lower quadrants

A nurse is caring for a client with type 1 diabetes on her first postpartum day. When planning care for this client, what changes in the client's insulin requirements does the nurse expect?

Suddenly decrease

On the second day of life, minutes after drinking 2½ ounces of formula, a newborn regurgitates about half an ounce. The mother remarks, "My baby spits up after every feeding." What should the nurse do next?

Suggest that she hold her baby upright for 30 minutes after feeding

A nurse is caring for a newborn with a cephalohematoma. What is the priority nursing action?

Supporting the parents

During assessment of a newborn, a practitioner diagnoses cephalhematoma and informs the parents. The mother asks why her baby's head looks different. What does the nurse take into consideration before responding in terms that the mother will understand?

Swelling that is confined to one part of the scalp is caused by hemorrhage beneath the periosteum.

Jaundice develops in a newborn 72 hours after birth. What should the nurse tell the parents is the probable cause of the jaundice?

The physiological destruction of fetal red blood cells -After birth, fetal erythrocytes hemolyze, releasing into the circulation bilirubin , which the immature liver cannot metabolize as rapidly as it is produced, resulting in physiological jaundice.

A nurse places a newly admitted client with worsening preeclampsia in a private room. Why is it important for this client to be in a nonstimulating environment?

The probability of tonic-clonic seizures is reduced.

Vitamin K (Aquamephyton) is to be given to a newborn to aid clotting. Click on the area where the injection will be administered.

The reason for using the vastus lateralis is that the site does not contain major nerves or blood vessels. Other sites, such as the ventrogluteal, which is not used until a child has been walking for 1 year, carry risks. The vastus lateralis muscle, middle third, is the site of choice. Either leg may be used.

A client is receiving an oxytocin (Pitocin) infusion for induction of labor. The uterine graph on the electronic monitor indicates no rest period between contractions, and this is confirmed on palpation. What should the nurse do first?

Turn the oxytocin infusion off

The nurse reads the history of a neonate admitted to the nursery and discovers that the infant's mother was listed as gravida 1 para 1 before the baby was born. How should the nurse use these data to gather more information?

To consider that someone recorded the gravida and para incorrectly -Gravida refers to pregnancies, including this one, and para refers to pregnancies terminated (by whatever means) after the age of viability. If this is the client's only pregnancy (gravida 1) she could not have had a previous pregnancy that ended after the age of fetal viability. Para will not exceed gravida. One pregnancy is gravida 1. A twin pregnancy is still one pregnancy terminated after the age of viability. Because the documentation of the client indicates that she is gravida 1, it cannot be assumed that it is the woman's second pregnancy.

A client at 16 weeks' gestation is scheduled for a sonogram followed by amniocentesis. The nurse instructs the client to drink 8 oz of fluid and not void before the sonogram. The nurse explains that this is done:

To improve visualization of the fetus

The healthcare provider of a woman who had a mastectomy has prescribed a Reach to Recovery visit. What does the nurse identify as the primary reason for the referral?

To prevent social isolation -The Reach to Recovery visit helps the client meet her need to remain within her social milieu and informs her about available community resources. Teaching arm exercises, meeting the client's physical needs, and viewing her surgical incision are all responsibilities of health care professionals.

A woman arrives at the prenatal clinic stating that her pregnancy test is positive. She asks the nurse for information about an abortion. After verifying that the woman is at 8 weeks' gestation, the nurse counsels her that having an abortion is controversial and that many women have long-lasting feelings of guilt after an abortion. What is the nurse's legal responsibility?

To provide the client with correct, unbiased information

A nurse notes that a healthy newborn is lying in the supine position with the head turned to the side with the legs and arms extended on the same side and flexed on the opposite side. Which reflex does the nurse identify?

Tonic neck

In what disorder is stimulation of labor contraindicated?

Total placenta previa -A total placenta previa requires a cesarean birth ; early intervention helps ensure a healthy neonate and mother

A nurse is teaching a client how to self-administer a medicated douche. In which direction should the nurse instruct the client to direct the douche nozzle?

Toward the sacrum -Toward the sacrum is the anatomic direction of the vaginal tract in the back-lying position. The vaginal tract may be injured if the douche nozzle is directed without consideration of the pelvic anatomy.

A male born at 28 weeks' gestation weighs 2 lb 12 oz. What characteristic does the nurse expect to observe?

Transparent red skin

When discussing dietary needs during pregnancy, a client tells the nurse that milk causes her to be constipated at times. What should the nurse teach the client?

Treat constipation when it occurs and continue drinking milk.

What is the safest position for a woman in labor when the nurse notes a prolapsed cord?

Trendelenburg -A position in which the mother's head is below the level of the hips helps decrease compression of the cord and therefore maintains the blood supply to the fetus. The prone position is impossible to maintain and will not relieve the pressure of the oncoming head on the cord.A position in which the mother's head is below the level of the hips helps decrease compression of the cord and therefore maintains the blood supply to the fetus. The prone position is impossible to maintain and will not relieve the pressure of the oncoming head on the cord.

During labor a client who has been receiving epidural anesthesia has a sudden episode of severe nausea, and her skin becomes pale and clammy. What is the nurse's immediate reaction?

Turning the client on her side -Maternal hypotension is a common complication of epidural anesthesia during labor, and nausea is one of the first clues that it has occurred. Turning the client on her side will keep the uterus from putting pressure on the inferior vena cava, which causes a decrease in blood flow. If signs and symptoms do not abate after the client is turned on her side, the health care provider should be notified. Checking the vaginal area for bleeding is not an assessment specific to epidural anesthesia; it is part of the general nursing care during labor. Fetal heart rate monitoring is a continuous process, and the rate should be recorded every 15 minutes; if this monitoring is not being performed, the rate should be checked and recorded every 15 minutes.

A client with osteoporosis has been receiving dietary information from the nurse. Which food selection by the client indicates that the nurse's dietary instruction was effective?

Turnip greens -Turnip greens are high in calcium and vitamins. A high level of nitrogen from protein breakdown may increase the release of calcium from bone to serve as a buffer of the nitrogen

The nurse is interpreting the results of a nonstress test (NST) on a client at 41 weeks' gestation. Which result after 20 minutes is suggestive of fetal reactivity?

Two accelerations of 15 beats/min lasting 15 seconds

A client is admitted with a marginal placenta previa. What should the nurse have available?

Two units of typed and screened blood

Laboratory studies reveal that a pregnant client's blood type is O and she is Rh-positive. Problems related to incompatibility may develop in her infant if the infant is:

Type A or B

Laboratory studies reveal that a pregnant client's blood type is O, and she is Rh positive. The client asks whether her newborn will have a problem with blood incompatibility. Before responding, the nurse must remember that fetal problems may develop if the fetus is:

Type A or B

Before a postpartum client is discharged, the nurse advises her about problems that should be reported and then asks her to recall these problems. Identification of which problem identified by the client indicates that the teaching has been effective?

Urgency, frequency, and burning on urination

A preterm infant is started on digoxin (Lanoxin) and furosemide (Lasix) for persistent patent ductus arteriosus. Which clinical finding provides the best indication of the effectiveness of the furosemide?

Urine output exceeds fluid intake.

A nurse is caring for a client who is being given intravenous magnesium sulfate to treat preeclampsia. Which adverse side effect alerts the nurse to notify the health care provider?

Urine output of less than 100 mL in 4 hours

A woman questions the nurse about the effectiveness of oral contraceptives. What most important factor about the effectiveness of oral contraceptives should be included in the reply to this question?

User motivation -Conception will not be prevented unless the user is motivated to use the method correctly and consistently.

Phototherapy is prescribed for a neonate with hyperbilirubinemia. Which nursing intervention is appropriate to reduce the potentially harmful side effect of the phototherapy?

Using shields on the eyes to protect them from the light

For what complication should the nurse specifically monitor a grand multipara who has just given birth?

Uterine atony

What complication should a nurse be alert for in a client receiving an oxytocin (Pitocin) infusion to induce labor?

Uterine tetany

A nurse places fetal and uterine monitors on the abdomen of a client in labor. While observing the relationship between the fetal heart rate and uterine contractions, the nurse identifies four late decelerations. What condition is most frequently associated with late decelerations?

Uteroplacental insufficiency

Supplemental oxygen is ordered for a preterm neonate with respiratory distress syndrome (RDS). What action does the nurse take to reduce the possibility of retinopathy of prematurity?

Verifying oxygen saturation frequently to adjust flow on the basis of need

A pregnant client with iron-deficiency anemia is prescribed a daily iron supplement. What nutrient should the nurse suggest that the client include in her diet to potentiate the effect of the iron supplement?

Vitamin C -Iron absorption is pH dependent; therefore iron should be taken with a source of ascorbic acid to enhance duodenal absorption. Biotin, lecithin, and vitamin B complex are all unrelated to the absorption of iron.

A client who has had recurrent infections before and during pregnancy should be instructed to eat a nutrient-rich diet as a means of supporting the body's natural defense mechanisms. What should the nurse encourage the client to include in her diet?

Vitamins A, C, and E and selenium -Vitamins A, C, and E and selenium are immune-stimulating nutrients.

While observing a newborn, the nurse notes that the skin is mottled. What should the nurse do first?

Warm the environment -Mottling results from hypothermia; the newborn should be wrapped, placed under a radiant warmer, or given to the mother for skin-to-skin contact.

A client in labor is receiving an oxytocin (Pitocin) infusion. For which adverse reaction resulting from prolonged administration should the nurse monitor the client?

Water intoxication

What should the nurse teach a formula-feeding mother about breast engorgement when it occurs?

Wear a tightly fitted brassiere.

A client with endometriosis asks the nurse what side effects to expect from leuprolide (Lupron). What should the nurse include in the response?

Weight gain

When is it most important for a female client to know that a fetus may be structurally damaged by the ingestion of drugs?

When she is planning to become pregnant -The greatest danger of drug-induced malformations is in the first trimester of pregnancy, during the period of organogenesis; because a woman may not know that she is pregnant, she should be aware of this possibility before becoming pregnant.

A client is to undergo a tuberculin test as part of her prenatal workup. Before administering the test, what information about the client should the nurse obtain?

Whether an earlier tuberculin test's result was positive

What is most important information to teach to a client who has had a mastectomy before she leaves the hospital?

Why self-examination of the remaining breast is important

When seeing her preterm infant son in the neonatal intensive care unit for the first time, a mother exclaims, "He's so little! How will I ever be able to take care of him?" The nurse explains to the mother that she:

Will be encouraged to participate in his care as much as possible

A nurse at a women's health clinic confirms that client teaching regarding the use of an oral contraceptive is understood when the client states, "I:

Will put a baby's picture on my bathroom mirror so I'll see it every morning" -Putting a baby's picture on the bathroom mirror serves as a reminder that the oral contraceptive must be taken every day. A woman should wait 2 to 3 months after stopping the oral contraceptive pill before attempting to become pregnant. If two consecutive menstrual cycles are missed, the client should stop the contraceptive pill and perform a pregnancy test. The client should use a barrier method of contraception for the first month of pill use and when a pill is missed to help prevent conception.


Related study sets

Culture Politics During the Time of Jesus

View Set

cell and molecular unit 1 practice problems

View Set

Preguntas Examen Teórico de Conducción 1

View Set

Human Communication: The Basic Course (Chapter 13)

View Set

Muscle Contraction and Types of Muscle

View Set

Interpersonal COM Exam 3: Conflict, Families, Romantic Relationships, and Friendships

View Set

Unit 10 : GRAMMAR [ { be going to , would like to } , { will , might , won't} , { making suggestions } ]

View Set

Chapter 11 - Childhood and Neurodevelopmental Disorders EAQs

View Set

Chapter 2: Historical Factors Public Health Nursing in Context

View Set